MCAT Biology

¡Supera tus tareas y exámenes ahora con Quizwiz!

Fetal Hemoglobin (HbF)

higher affinity for oxygen than adult hemoglobin (HbA) Left shifted curve compared to adult Fetal red blood cells pull O2 off maternal hemoglobin

Lacteals

Small lymphatic vessels found in small intestine/digestive tract that transport long-chain fatty acids into lymphatic system (by-passing liver)

All arteries transport:

Blood away from heart Veins carry blood towards the heart arteries are under the highest pressure The blood pressure in the aorta is always higher than the pressure in the superior vena cava.

Calcitonin

Calcitonin TONES down calcium levels in blood 2 cell populations within gland Follicular cels produce thyroid hormones and C-cells C Cells/ Parafollicular cells produce calcitonin Calcitonin decreases plasma calcium levels in 3 ways: increasing calcium excretion from kidneys, decreasing calcium absorption from gut, increasing storage of calcium in bone High levels of calcium in blood stimulate secretion of calcitonin from the C cells Calcium is important for bone structure and strength, release of neurotransmitters, regulation of muscle contractions, clotting of blood (calcium is cofactor), cell movement and exocytosis of cellular materials

Centriole vs. Centromere vs Centrosomes

Centriole: small organelle in cytoplasm of animal cells: Organizes spindle apparatus during mitosis/meiosis Centromere: Area of chromosome where sister chromatids are joined: Point of attachment to spindle fiber during mitosis/meiosis Centrosomes: paired cylindrical organelles, located in cytoplasm, that contain centrioles

Coagulation

Clots composed of both coagulation factors (proteins) and platelets (prevent/minimize blood loss) -Dammage to endothelium of blood vessel exposes connective tissue (contains collagen and tissue factor) -When platelets encounter collagen, it tells them there is injury, and they release their contents and clump/aggregate in response -Coaggulation factors also released by liver: Activation of prothrombin to form thrombin by thromboplastin -Thrombin can convert fibrinogen into fibrin (forms net that captures RBCs and platelets to clot) -Thrombus formation/blood clotting begins when platelets attach to exposed endothelial cells (activates quiescent integrin molecules -> forms bridge/ plug injury) Clot broken down by plasmin (generated from plasminogen)

Enteric Nervous System (ENS)

Collection of 100 million neurons that control function of GI system Present in GI walls, trigger peristalsis (rhythmic contractions of gut tube) -independent of CNS, but heavily dependent on Autonomic NS -Parasympathetic involved in stimulation of digestion, increases secretion from exocrine glands and promoting glands (sympathetic inhibits) -All glands of body (except sweat glands) are innervated by parasympathetic NS *Parasympathetic NS increases secretions from all digestive glands and promotes peristalsis, SNS slows peristalsis

Ductus arteriosus Ductus Venosus

Fetal structure that connects/shunts blood from the pulmonary artery to the aorta, bypassing the lungs/reducing blood flow to lungs This is important because the lungs are not fully developed early in gestation, and therefore are not capable of supporting high blood pressure Shunt from umbilical vein to inferior vena cava, allowing for oxygenated blood returning from placenta to bypass liver and enter the systemic circulation

Ligaments Tendons

Connect bone to bone Connects bone to muscle (too badmom)

Ductus Venous

Connects maternal and fetal circulations Reduce blood flow to liver

Anton van Leeuwenhoek Rudolph Virchow

First to view living cell under microscope (1674) Demonstrated that diseased cells could arise from normal cells in normal tissues

Right side of heart pumps blood into a ________ resistance circuit and must do so at ______ pressures, so it requires _______ muscle The left side of heart pumps blood into _________ resistance circuit at ________ presses and requires _____ muscle

Right side of heart pumps blood into a lower resistance circuit and must do so at lower pressures, so it requires less muscle The left side of heart pumps blood into higher resistance circuit at higher presses and requires more muscle

Pattern Recognition Receptors (PRRs)

Type of receptor on macrophages and dendritic cells that is able to recognize the nature of the invader (bacteria, virus, or fungi) and release the appropriate cytokines to attract the right immune cells to the area -Macrophages and DC checks have pattern recognition receptors (PRRs), including Toll-like receptors (TLRs) -PRRs recognize category (bacteria, fungi, virus, etc), which allows for production of appropriate cytokines

Regulation of Breathing

Ventilation primarily regulated by neurons in medulla oblongata (Ventilation center) that fire -Cerebrum can consciously control ventilation, but can be overridden by medulla -Neurons contain chemoreceptors (sensitive to carbon dioxide concentration) -Partial Pressure of CO2 increases (hypercarbia/hypercapnia) = increase in respiratory rate (more CO2 exhaled = levels in blood fall) -Cells also respond to O2 concentration (only during hypoxemia/ low O2 concentration in blood) -Hypoventilaition = increased CO2 levels (override medulla oblongata = jumpstart breathing) -Hyperventilation = blow off too much CO2 and inhibit ventilation ultimately (shift to the left in the bicarbonate buffer equation, and the blood becomes more alkaline. Breathing into the bag allows some of this carbon dioxide to be returned to the bloodstream in order to maintain the proper pH.)

Leukocytes

WBC Comprise less than 1% of total blood volume Production of antibodies and defense against infection 4500-11000 per cubic millimeter of blood (can increase during infection) Five Types in 2 classes : All come from hematopoietic stem cells: Granulocytes and Agranulocytes

Fertilization (Joining of sperm and ovum) occurs in the:

Widest part of the Fallopian tubes, Called the ampulla

Peroxisomes

breakdown of very long chain fatty acids through beta-oxidation -Inability to form hydrogen peroxide = inability to digest long chain fatty acids

Which hormone elevates calcium blood concentrations in response to low blood calcium levels? What antagonizes this hormone, causing calcium levels to decrease in blood?

Parathyroid Hormone (PTH) causes osteoclasts in bone to break down bone, referred to as bone resorption, so that calcium is released in blood Calcitonin antagonizes PTH and causes calcium levels to decrease in blood

Mendel's Laws

law of segregation (first law): Disjunction: Each chromosome of paternal origin separates from its homologue of maternal origin, and either chromosome can end up in either daughter cell (random) law of independent assortment: Second Law: Explained by crossing over: Inheritance of one allele has no effect on likelihood of inheriting certain alleles for other groups law of dominance

SRY gene

the sex determining region of the Y chromosome codes for a transcription factor that initiates testis differentiation and this formation of male gonads

Refractory Period

the time following an action potential during which a new action potential cannot be initiated Cell is depolarized No response can be elicited during this time

Lung Membranes

-Pleurae surround each lung, form sac against lung -Surface adjacent to lung = visceral pleura, outer part = parietal pleura -Side that touches lung = visceral, outer layer/associated with chest wall = parietal -Intrapleural space = space within sac (contains thin layer of fluid) potential space (normally empty, pleural effusion = fluid there, pneumothorax = air there, both bad) -Pressure differentials between the intrapleural space and lungs drive air into lungs

cyclic adenosine monophosphate (cAMP)

An intracellular second messenger in the signaling cascade initiated by a peptide hormone; synthesized from ATP by adenylate cyclase.

Cell migration, death and regeneration

Apoptosis: Programmed cell death -Apoptotic signals or pre-programming Cell divides into many self contained protrusions called apoptotic blebs -> broken into apoptotic bodies -> digested by other cells/recycling materials (prevents release of harmful stuff, differs from necrosis) Necrosis: Cell dies as a result of injury, internal substances released Regeneration: Incomplete regeneration = newly formed tissue not identical in structure or function (most of humans, but liver has high regenerative capacity) Senescence: Biological aging (at cellular level, failure of cells to divide, normally after 50 divisions in vitro) -May be due to shortened telomeres (ends of chromosome, prevent unraveling/ loss of genetic info, shorten during each round of synthesis) Telomerase enzyme prevents senescence (divide indefinitely) (Found in germ, fetal, and tumor cells)

Hilum

Area of an organ where large vessels or other structures enter or exit; the renal hilum is where the renal artery enters the kidney, the renal vein leaves the kidney, and the ureter exits the kidney to transport urine to the bladder.

Metaphase 2

Chromosomes line up at the metaphase plate

Plasmids

Circular DNA molecules that can replicate independently of the main chromosomes of bacteria -DNA acquired from external sources -Not necessary for survival -Extrachromosomal (extragenomic) material -May carry additional virulence factors/traits that increase pathogenicity -Subset of plasmids called episomes are capable of integrating into the genome of bacterium

3 Main effects of circulating antibodies on pathogen

Circulating antibodies can mark a pathogen for destruction by phagocytic cells (opsonization), cause agglutination of pathogen into insoluble complexes that can be taken up by phagocytic cells, or neutralize pathogen by preventing it from invading tissues

Which of the following hormones directly stimulates a target tissue that is NOT an endocrine organ? A. ACTH B. TSH C. LH D. GH

D A hormone that directly stimulates a non-endocrine target tissue is referred to as a direct hormone. Glancing at the answer choices, we notice that all of the hormones are secreted by the anterior pituitary gland. The direct hormones secreted by the anterior pituitary are prolactin, endorphins, and growth hormone (GH). All of the other answer choices are tropic hormones; the tropic hormones of the anterior pituitary include follicle-stimulating hormone (FSH), luteinizing hormone (LH), adrenocorticotropic hormone (ACTH), and thyroid-stimulating hormone (TSH).

Complement System

Has a classical pathway that requires antigen binding, and an alternative pathway that is antibody independent

Transposons

Jumping Genes Genetic elements capable of inserting and removing themselves from genome Do not require a vector

Cervix

Lower end of the uterus that marks the transition between vagina and uterus

List the vessels in the renal vascular pathway, starting from the renal artery and ending at the renal vein.

Renal artery, afferent arteriole, glomerulus, efferent arteriole, vasa recta, renal vein

The Nervous System

Responsible for control of muscular movement, neuromuscular reflexes, and granular secretions, high-level thinking/mental functions

Sertoli cells and Leydig cells

Sertoli cells are supportive cells in the tubules Leydig cells secrete testosterone

Aldosterone Aldosterone increases BP by: Low perfusion of juxtaglomerular cells of kidney stimulate aldosterone release, which:

Steroid hormone secreted by adrenal cortex in response to decreased BP -Alters ability of distal convoluted tubule and collecting duct to reabsorb sodium Increasing reabsorption of sodium -We use drugs that block angiotensin-converting enzyme/ angiotensin 2 receptor -angiotensin 2 receptor blockers end with -sartan -Blocking receptor limits aldosterone release -> limits salt and water reabsorption -> lowered BP *Low perfusion of juxtaglomerular cells of kidney stimulate aldosterone release, which increases reabsorption of sodium/water = increase in BP and blood volume

Heart Contraction

Systole: Ventricular contraction and closure of AV valve, blood pumped out of ventricles (pressure increase) Diastole: Ventricles relax, semilunar valve closed, blood from atria fills ventricles (pressure decrease) *Tachycardia: (V-tach): heart can't properly fill with blood

Dense Connective Tissues: __________ attach muscle to bone and _________ hold bones together at joint

Tendons attach muscle to bone and ligands holds bones together at joints *ligands link bones

Cytoskeleton (Eukaryotes)

Three Components: Microfilaments, microtubules, and intermediate filaments Microfilaments: Made up of solid polymerized rods of actin, provide protection -Actin filaments can use ATP to generate force for movement by interacting with mystic, such as in muscle contraction -Play role in cytokinesis, form cleavage furrow -MICROFILAMENTS DO NOT CONTAIN TUBULIN Microtubules: Hollow polymers of tubulin proteins -Primary pathway along which motor proteins like kinesis and dynein carry vesicles (non enzymatic protein functions) -Cilia and flagella are mobile structures composed of microtubules (Both composed of 9 pairs of microtubules forming outer ring and 2 in center, called 9+2 structure only seen in eukaryotic organelles of motility) -Centrioles: Found in centrosome, organizing centers for microtubules, structured as 9 triplets of microtubules with hollow center, organize mitotic spindle, attach to chromosomes via kinetochores and pull sister chromatids apart during mitosis Intermediate Filaments -Includes keratin, desmin, vimentin, lamins -Many involve in cell-cell adhesion or maintenance of overall integrity of cytoskeleton -Can withstand tremendous tension, increasing structural rigidity -Anchor organelles

Tidal Volume vs Residual Volume

Tidal: The amount of air exchanged in a normal breath Residual: The minimum amount of air that remains in the lungs at all times

Cells capable of becoming any cell type in the human body or even the placenta are called:

Totipotent: Capable of becoming any cell type Pluripotent: Capable of becoming many, but not all, cell types Differentiated cells have undergone differential and are no longer capable of becoming all cell types Determined/Committed cells are tied to one lineage

Sex-linked

X-linked Males are homozygous (only one copy of x-chromomse) Females express disorders less frequently (the can be carriers, males can't Y chromosome contains very little genetic info Presence of Y chromosome = male, absence of Y = female

Spermatogenesis

formation of haploid sperm through meiosis, occurs in seminiferous tubules Diploid cells = spermatogonia -After replicating genetic material (s phase) they develop into diploid primary spermatocytes First meiotic division results in haploid secondary spermatocytes Meiosis 11 = haploid spermatids (undergo maturation to become spermatozoa) Results in 4 functional sperm for each spermatogonium

Structure of mature sperm

head (containing the nucleus/genetic material) - midpiece (mitochondria that releases ATP as energy source) - tail (motility) -Each head is covered by acrosome, structure derived from Golgi apparatus and necessary to penetrate ovum *Around 3 million sperm produced every day

Three types of T-Cells

helper T-cells: CD4+ T cells: Coordinate response by secreting lymphokines (recruit and increase activity of immune cells like plasma/cytotoxic/macrohpages -Loss of these cells = HIV/AIDS -CD4+ respond to antigens presented on MHC 2 (exogenous, most affective against bacteria, fungi, parasite) -BETTER AT EXTRACELLULAR INFECTIONS killer (cytotoxic) T-cells/ Tc/CTL: CD8+ -Capable of directly killing virally infected cell -Respond to antigens on MHC 1 (Presents endogenous antigens, so CD8 most effective against viral/intracellular) Think: CD X MHC = 8 CD4+ respond to MHC 2 = 8 CD8 respond to MHC 1 = 8 Suppressor/Regulatory T cells (Treg): -Also express CD4 -Express protein called Foxp3 to tone down immune response Self Tolerance: Turn off self-reactive lymphocytes to prevent autoimmune disease *self-reactive T-cells eliminated in thymus -Many were formally self reactive T cells that have been turned off Memory T cells: Lie in wait for next exposure to same antigen

Pineal Glan

helps regulate sleep-wake cycle (circadian rhythm), deep within brain, secretes melatonin -Receives projections from retina

Bacteria Growth Phases

lag phase (first adapt to new conditions), log/exponential phase (bacteria adapt, rate of division increases, exponential increase in number), stationary phase (reduction of resources slows reproduction), death phase (Bacteria exceed ability of environment to support the number of bacteria/depletion of resources) Semilog plot graph

Allantois Amnion

one of four embryonic membranes; it contains the growing embryo's waste products Allantois: involved in early fluid change from embryo to yolk sac (forms umbilical cord) Surrounded by amnion (thin tough membrane filled with amniotic fluid)- shock barrier Chorion also forms outer membrane around amnion (more protection) Amnionthe innermost fluid-filled embryonic membrane; it forms a protective sac surrounding the embryos of birds, reptiles, and mammals

Osmotic Pressure VS Oncotic Pressure

osmotic = "sucking" pressure that draws water into vasculature caused by all dissolved particles (maintained at 290 milliosmoles per liter) oncotic pressure = osmotic pressue that is attributable to dissolved proteins specifically. osmolarity of blood must be tightly controlled to ensure correct ONCOTIC pressure w/in vasculature

Stomach

*Highly muscular organ *can hold about 2 liters *Upper left quadrant, underneath diaphragm *Uses hydrochloric acid and enzymes *thick mucosa to prevent auto-digestion *Minimal digestion occurs before stomach and small intestine, even though chemical digestion of carbs and fats is initiated in mouth *divided into 4 parts: Fundus/body (mostly gastric glands), Antrum/pylorus (mostly pyloric glands), internal curvature of stomach/lesser curvature, and external curvature/greater curvature Lining of stomach thrown into folds called rugae Mucosa of stomach contains gastric glands and pyloric glands NSAIDS = gastritis/gastric ulcers = damage to stomach mucus Few things absorbed directly by stomach (alcohol and aspirin are), stomach mainly organ of digestion

Function of Lymphatic System:

*Net pressure drawing fluid in at venule end is slightly less than pressure pushing fluid out at arterial end, so small amount of fluid remains in tissue: Lymphatic vessels drain tissues and return fluid to bloodstream *Offers some protection (Example: if blood has low albumin (plasma protein) indicates liver failure, the oncotic pressure is decreased so less water is driven back into bloodstream at venule end = fluid collects in tissues, and much of the fluid eventually returns to blood stream via lymphatics Edema: Swelling due to fluid collecting in tissue (Lymphatics overwhelmed) Transportation of Biomolecules: Transports fats from digestive system to blood Immunity: Lymph nodes are place where antigen presenting cells and lymphocytes interact -B cells proliferate and mature in lymph nodes and collections called germinal centers

Penetrance and Expressivity

*Penetrance*—the proportion of the population with a given genotype who actually express the phenotype -Huntington's *Expressivity*—the different manifestations of the same genotype across the population -Differences in expression (severity, location, etc) of phenotype -If expressivity is constant, all individuals express same phenotype, but if expressivity is variable, individuals with same genotype may have different phenotypes Penetrance is a population parameter (percent of individuals with genotype that express the phenotype) Expressivity reflects gray area: More commonly considered at individual level -Variable expressivity: Presentations of disease ranging from no effects to severe

What two main enzymes are found in saliva? Function?

*Salivary amylase (ptyalin) hydrolyzes/digests starch into smaller sugars (maltose and dextrins) *lipase catalyzes hydrolysis of lipids/digests fats Both start chemical digestion of food

Lung Capacities and Volumes

- Determined using spirometer - Can't measure amount of air remaining in lungs after complete exhalation (residual volume), can provide number of measures that are useful Total Lung Capacity: TLC: Max volume of air in lungs when one inhales completely (6-7 liters) Residual Volume (RV): Volume of air remaining in lungs when one exhales completely Vital Capacity: VC = TLC-RV: Difference between max and min volume of air in lungs Tidal Volume (TV): Volume of air inhaled or exhaled during normal breath Expiratory reserve volume (ERV): Volume of additional air that can be forcibly exhaled after normal exhalation Inspiratory Reserve Volume (IRV): Volume of additional air that can be forcibly inhaled after normal inhalation

Inhalation and Exhalation

- Inhalation = active process -Exhalation = does not have to be active process (active or passive) -Inhalation: uses diaphragm & external intercostal muscles (between ribs) to expand the thoracic cavity (increases volume of the intrapleural space = decrease in intrapleural pressure) - ultimately expands lungs, dropping pressure, drawing air in (negative pressure breathing) *Diaphragm contacts during inhalation and relaxes during exhalation -Diaphragm relaxes = gets bigger and pushes on lungs = exhalation -As diaphragm contracts (gets smaller)/ flattens and chest wall expands out, intrathoracic volume increases = decrease in intrapleural pressure (inhalation) -Pressure in lungs drops, air drawn into lungs (negative pressure) -Gas in lungs initially at atmospheric pressure, now higher than pressure in intrapleural space = lungs expand = pressure drop = air sucked in from high pressure (outside) = negative-pressure breathing (driving force is lower pressure in intrapleural space compared to lungs) Exhalation: relax external intercostal muscles = decrease in volume = increase in intrapleural space pressure (Boyles Law)(pressure higher than in lungs, which are still at atmospheric pressure) = air pushed out -We can speed this up by using internal intercostal muscles (oppose intercostals, decrease thoracic volume) *Passive exhalation: relax muscles, elastic recoil of lungs allow chest cavity to decrease in volume, reverse pressure differentials *Active exhalation: internal intercostal muscles and abdominal muscles can forcibly decrease volume of thoracic cavity, pushing out air *Surfactant prevents complete collapse of alveoli during exhalation by reducing surface tension *Chest wall expands = pulls air in *Chest walls relaxes = lung volume becomes smaller - pressure in lungs higher than pressure outside, so exhalation occurs *Emphysema = difficulty exhaling

Aerobes and Anaerobes

- obligate aerobes are bacteria that require oxygen to grow - anaerobic bacteria do not use oxygen for growth or metabolism -Obligate anaerobes can't survive in O2 (causes production of reactive oxygen species) -Facultative Anaerobes: Can switch between aerobic and anaerobic -Aerotolerant anaerobes: Unable to use O2 for metabolism but can survive in its presence

Fetus constructs 3 shunts to actively direct blood away from organ and lungs while they develop:

-2 shunts used to reroute blood from lungs: -foramen ovale (one way valve connects right atrium (gets blood from inferior VC) to left atrium -allows blood pressure to be higher on right side of heart - ductus arteriosus (shunts leftover blood from pulmonary artery to aorta (connecting them) Liver bypassed via ductus venous (shunts from placenta straight into inferior vena cava/ connects umbilical vein to inferior vena cava)

Skin (Integument)

-Largest organ of the body (16% of total body weight) -Derived from ECTODERM - Protects us from elements/microbes, UV (melanin), transduction of sensory info, thermoregulation, osmolarity (skin is relatively impermeable to water, keeps water out/in tissues) -Provides function for thermoregulation and innate immunity Epidermis: Subdivided into 5 strata Dermis: Divided into upper layer (Papillary layer)(Right below epidermis, loose connective tissue), Reticular layer (denser) -Sweat glands, blood vessels, and hair follicles originate in dermis -Many sensory cells here, like Merkel cells (deep pressure and texture), free nerve endings (pain), Meissner's corpuscles (light touch), Ruffini endings (stretch), and Pacinian Corpuscles (deep pressure and vibration) Hypodermis (subcutaneous layer): Connects skin to rest of body -Contains fat and fibrous tissue

Amino acid derivative hormones

-Less common than peptide and steroid hormones -derived from 1 or 2 amino acids w/ additional modifications -Include epinephrine, norepinephrine, triiodothyronine, and thyroxine Less predictable -Catecholamines (epinephrine and norepinephrine) bind to G protein coupled receptors --> fast onset & short lived -Thyroid hormones bind intracellularly (slower onset & longer duration) -Have names that end in -in or -ine (thyroxine, triiodothyronine)

Absorption of Lipids/Vitamins in Small Intestine

-Short chain fatty acids follow same process as carbs and amino acids (diffuse directly into intestinal capillaries) *Do not require transporters because they are non polar/can easily transverse cellular membrane -Larger fats, glycerol, and cholesterol move separately into intestinal cells but then reform into triglycerides (triglycerides and esterified cholesterol packaged into chylomicrons-> enter lymphatic circulation through lacteals (small vessels that form beginning of lymphatic system)) Lacteals converge and enter venous circulation though thoracic duct (base of neck) which empties into left subclavian vein Vitamins: *Fat soluble vitamins: A, D, E, and K *dissolve directly into chylomicrons to enter lymphatic circulation *Water soluble: B complex and C, and all others *Taken up with water, amino acids, carbs across endothelial cells of SI, passing directly into plasma *In addition to fats, carbs, amino acids, and vitamins, small intestine also absorbs water (mostly from secretions: up to 7 liters/day, reabsorbed by osmosis) *water passes transcellulary (across cell membrane) and paracellularly (squeezing between cells) to reach blood

Cartilage

-Softer and more flexible than bone -Avascular (no blood or lymph vessels) and not innervated -Consists of firm but elastic matrix called CHONDRIN that is secreted by chondrocytes *CHONDROCYTES FORM CARTILAGE -Composes most of fetal skeleton -Adults only have cartilage in external ear, nose, walls of larynx and trachea, intervertebral discs, and joints (need extra cushion) Most bones created by hardening of cartilage: Endochondral ossification (responsible for long bone formation) Bones may also be formed by intramembranous ossification (undifferentiated embryonic connective tissue/mesenchymal tissue transformed into and replaced by bone): Occurs in skull

Macrophages

-Type of agranulocyte -Eats everything: Engulfs and consumes pathogens -Reside within tissues (activated in tissues), come from blood-borne monocytes Residential Macrophages with specific names: microglia in CNS, Langerhans cells in skin, or osteoclasts in bone) When activated, Macrophage does 3 things: -Phagocytizes invader through endocytosis -Digests invader with enzymes -Presents little pieces of the invader (mostly peptide) to cells using protein called Major histocompatibility complex (MHC) -Binds to pathogenic peptide/antigen and carries it to cell surface to recognized by cells of adaptive immune system Macrophages release cytokines (chemicals that stimulate inflammation and recruit immune cells)

Cytotoxic Immunity (cell-mediated)

-centered on the function of T-cells (mature in thymus) T cells undergo positive and negative selection Positive selection: Allowing maturation of cells that can respond to presentation of antigen on MHC (cells that cant respond to MHC undergo apoptosis) Negative selection: Apoptosis of cells that self reactive/activated by self proteins Maturation of T cells facilitated by THYMOSIN (peptide hormone secreted by Thymic cells) When T cell leaves thymus, its mature but naive Exposure to antigen = clonal selection so that only those with highest affinity for antigen proliferate *When the adaptive immune system encounters an antigen, only the cells with receptors (antibodies or T-cell receptors) specific to that antigen are activated.

Thyroid

-controlled by thyroid stimulating hormone from anterior pituitary -Thyroid hormones decrease the sensitivity of the anterior pituitary to TRH, thereby decreasing TSH secretion -located on front surface of trachea (base of neck) -sets the basal metabolic rate (by releasing T3 and T4) and maintains calcium homeostasis (By releasing calcitonin) T3 (Triiodothronine) and T4 (Thyroxine) produced by iodination of amino acid tyrosine in follicular cells of thyroid Thyroid can reset/ increase basal metabolic rate by making energy production more/less efficient and by using glucose/fatty acids -Increased T3 and T4 = increased cellular respiration = increased protein and fatty acid turnover *T3 and T4 stimulate metabolic activity -High plasma levels of thyroid hormones will lead to decreased TSH and TRH synthesis -Negative feedback prevents excessive secretions of T3 and T4

Steroid Hormones

-derived from cholesterol -produced by gonads and adrenal cortex -Derived from non polar molecules, can easily cross the cell membrane -Bind to intracellular (in the cytosol) receptors -function by binding to DNA to alter gene transcription -Common form of conformational change = dimerization (pairing of two receptor hormone complexes) -Effects are Slower and longer lived (participate in gene regulation, cause alterations in amount of mRNA, have direct actions on DNA) -Lipid soluble, Not water soluble --> must be carried by proteins in bloodstream --> inactive while attached to carrier protein (must dissociate from carrier to function)(must be carried by specific protein) -Some are very specific and carry only one hormone (sex hormone binding globulin) -Have names that end in -one, -ol, or -oid (testosterone, estrogen, cortisol, glucocorticoids)

Endocrine Glands (Hypothalamus and interactions with anterior and posterior pituitary)

-hypothalamus, pituitary, thyroid, parathyroid glands, adrenal glands, pancreas, gonads (testes and ovaries), pineal glands -Capable of synthesizing and secreting hormones -Produce hormones that are secreted into the bloodstream to act on distant target tissues Hypothalamus: Bridge between nervous and endocrine systems -Regulates pituitary though tropic hormones -Located in forebrain, above pituitary (Which is controls through paracrine release of hormones) -Contanis nuclei in 3 sections: lateral, ventromedial, and anterior -Release of hormones by hypothalamus is regulated by negative feedback (Occurs when hormones/product later in pathway inhibits hormones/enzymes earlier in pathway- maintains homeostasis./ conserves energy) Hypothalamus interactions with the anterior pituitary: -hypothalamus secretes compounds through paracrine signaling into hypophyseal portal system (blood vessel that connects hypothalamus with anterior pituitary), then travel down pituitary stalk and bind to receptors in anterior, stimulating release of other hormones -Hypophysis = term for pituitary -Image shown: Hypophyseal portal system (System of blood vessels that connects hypothalamus to pituitary Hypothalamus secretes several tropic hormones (list is hormone secreted by hypothalamus and hormone released by anterior pituitary in response) - Gonadotropin-releasing hormone (GnRH) promotes release of follicle-stimulating hormone (FSH) and luteinizing hormone (LH) - Growth hormone releasing hormone (GHRH) -> Growth hormone (GH) -Thyroid releasing hormone (TRH) promotes release of Thyroid stimulating hormone (TSH) -Corticotropin-releasing factor (CRF) promotes release of adrenocorticotropic hormone (ACTH) -Exception: Prolactin inhibiting factor (PIF) (AKA dopamine) is released by hypothalamus but causes DECREASE in prolactin secretion (ABSENCE of PIF allows prolactin to be released) -Each of the tropic hormones has negative feedback effects -Release of CRF from hypo stmlautes anterior to secrete ACTH, which causes adrenal cortex to increase level of cortisol being secreted (detrimental in high levels, so cortisol inhibits hypothalamus and anterior pituaitary from releasing CRF and ACTH) -Three organ systems referred to as axes (like hypothalamic-pituitary-adrenal (HPA) axis) *Hypothalamus -> CRF-> Anterior pituitary -> ACTH -> Adrenal cortex -> cortisol Hypothalamus with the posterior pituitary: Does not recieve tropic hormones through hypophyseal postal system -Neurons in hypothalamus send their axons down pituitary stalk directly, which can release oxytocin (stimulate uterine contractions/lactation) and antidiuretic hormone(ADH/ Vasopressin)(Increases reabsorption of water, secreted in repose to plasma osmolarity/ encased concentrations of solutes within the blood)

Peptide Hormones

-made up of amino acids -Examples: ADH (anti-diuretic homeome = small, insulin = large) -derived from larger polypeptides that are cleaved into smaller units during post-translational modification -transported to Golgi apparatus for modification (activation) -packaged into vesicles and released by exocytosis -charged/Polar so they cannot pass through plasma membrane -Must bind to an extracellular receptor (first messenger) and trigger transmission of a second signal (second messenger) Signaling cascade: Connection between hormone at the surface and the effect brought about by second messengers within the cell -At each step there is the possibility of amplification (1 hormone bind to multiple receptors/one receptor may activate multiple enzymes) Common second messengers: -Cyclic adenosine monophosphate (cAMP) -Inositol triphosphate (IP3) -Calcium -Activation of G-protein coupled receptor: Binding of peptide hormone triggers receptor to either activate or inhibit enzyme adenylate cyclase, raising or lowing levels of cAMP, which can bind to intracellular targets, like protein kinase A, which phosphorylates transcription factors like cAMP response element binding protein (CREB) to exert hormones ultimate effect. Protein kinase A can modify other enzymes as well as transcription factors, can have rapid or slow effect on cell (Image shown) -Rapid and short lived effects -Water soluble so they travel freely in bloodstream (do not require carriers) (Contrast to steroid hormones, which are lipid soluble) Have names that end in -in or -ine (insulin, vasopressin) Key Concept: Peptide hormones have surface receptors and act via second messenger systems.

Gas and Solute Exchange

-occurs at level of the capillaries -relies on the existence of concentration gradients to facilitate diffusion across the capillary walls -capillaries are also leaky, which aids in transport of gases and solutes Oxygen: carried by hemoglobin (protein made of 4 subunits, each with heme group with central iron that binds to O2 -Goes from T to R state when bound to O2: each O bound increase affinity for other 3 sites = (Cooperative binding/ Allosteric/ Sigmoidal (s-shaped) curve) -Normal Partial pressure of O2 = 70-100 mmHg (blood from artery) -Oxygen saturation = % of hemoglobin carrying O2 (usually above 97%) -Tissues at lower partial pressure of O2 (around 40 mmHg at rest, 80% saturated, so 20% of O2 has been released from hemoglobin to tissues) -O2 saturation of hemoglobin at 30% (20 mmHg on X-axis) means that about 70% of O2 has been released to tissues during exercise -Curve shift to right during exercise in response to increased arterial CO2, increased H+, decreased pH, and increased temp (represents decreased affinity for O2, allowing it to be unloaded at tissues) Where should you look on the oxyhemoglobin dissociation curve to determine theamount of oxygen that has been delivered to tissues? Y value = % hemoglobin saturation

Carbon Dioxide

-primary waste product of cellular respiration -Non polar (like O2) = low solubility in aqueous plasma -Hemoglobin has much lower affinity to CO2 than O2 -most CO2 in blood exists as bicarbonate ions (HCO3-) -When CO2 enters red blood cells, carbon anhydrase catalyzes a combination reaction b/w CO2 & water to form carbonic acid (H2CO3)(weak acid) which dissociates into a proton and bicarbonate anion (both have high solubility in water making them a more effective method of transporting metabolic waste products to the lungs for excretion) -At alveolar capillaries in lungs, this reaction is reversed allowing us to breathe out CO2 -Concentration of free protons in blood affect pH (allosteric effect on oxyhemoglobin dissociation curve) -Increased CO2 = right shift in bicarbonate buffer equation = increased H+ = Decreased pH -Protons bind to hemoglobin, reducing hemoglobin affinity to O2 (shift to right = Bohr effect) -Higher rates of metabolism = increased CO2 production and lactic acid = decrease pH = decreased affinity = more O2 unloaded at tissues

Viral Progeny Release

-viral invasion may initiate cell death --> spills viral progeny -Host cell may lyse as a result of being filled with extremely large numbers of virions (Lysis bad for virus because virus can no longer use cell) -virus can leave the cell by fusing with its plasma membrane (process known as extrusion)--> allows survival of host cell and continued use of cell by virus (Productive Cycle) Lytic Cycle: Bacteriophage maximizes the use of the cell's machinery with little regard to survival of host cell -Cell swells and lyses (virulent viruses) Lysogenic Cycle: Virus integrates into host genome as provirus or prophage -Virus replicated as the bacterium reproduces because it is now part of host genome -May leave genome and revert to lytic cycle -Some phages can benefit bacteria (makes them less susceptible to superinfection (simultaneous infection)

Summary of Menstrual Cycle

1) Low levels of estrogen allows small increases in LH and FSH lead to follicular maturation, including an increase in the synthesis and secretion of estrogen 2) As follicle matures, it secretes more estrogen which exerts a negative feedback on FSH and LH → this leads to an initial dip in FSH and a flat lining of LH 3) As the amount of estrogen continues to grow, it crosses a threshold where it flips to exerting a positive feedback on LH and FSH, causing a huge spike in LH and FSH around day 14 4) This LH and FSH spike causes the rupturing of the primary follicle and the release of the ovum 5) Now that the primary follicle has ruptured, there is a dip in estrogen, which also removes the positive feedback on LH and FSH, allowing them to drop back down 6) The primary follicle turns into the corpus luteum, which starts secreting estrogen and progesterone for 10 days. This estrogen and progesterone exert negative pressure on FSH and LH, so no new follicles can mature 7) Eventually if no pregnancy occurs, the corpus luteum degrades, menstruation occurs, and the cycle starts over again with low estrogen levels releasing their inhibition of LH and FSH, allowing their concentrations to slightly increasing in the beginning of the month Follicular: Egg develops, endometrial lining becomes vascularized and glandularised, FSH levels increase, LH levels stay same, estrogen decreases and then increases, and progesterone decreases Ovulation: Egg released from follicle into peritoneal cavity, FSH levels increase, LH go way up, estrogen increases, and progesterone decreases Luteal: Corpus lute produces progesterone to maintain endometrium: FSH levels decrease, LH levels stay same, estrogen increases, and progesterone increases Menses: Shedding of endometrial lining: FSH levels and LH levels decrease, estrogen decreases and progesterone decreases

Prokaryotic Cells

1-10 micrometers (Eukaryotes 10X larger) 2/3 of overarching domains of life contain prokaryotes (bacteria and archaea, initially both classified as Monera) Always single-celled Do not contain a nucleus/membrane bound organelles Singular, circular molecule of DNA in nucleic region Cell wall + cell membrane/plasma membrane composed of phospholipid = envelope Cell wall provides structure and controls movement of solutes, also maintains concentration gradient (2 main types, gram positive and gram negative) -Lack mitochondria, so cell membrane is used for electron transport chain and generation of ATP -Contain primitive cytoskeleton -Contain smaller ribosomes (30-50S vs 40-60S in eukaryotes) (neither ribosome is surrounded by membrane) Reproduce via asexual reproduction in the form of binary fission -Capable of acquiring and using genetic material from outside the cell

Four fundamental tenets of cell theory

1. All living things are composed of cells 2. The cell is the basic fundamental unit of life 3. Cells arise only from preexisting cells 4. Cells carry genetic information in the form of deoxyribonucleic acid (DNA). This genetic material is passed on from parent to daughter cell. *Viruses are not living because they violate rule 3 and 4 (They can't replicate on their own and use RNA instead of DNA)

Three main mechanisms to remove neurotransmitters from synaptic cleft:

1. Neurotransmitters can be broken down by enzymatic reactions -Breakdown of acetylcholine 2. Neurotransmitters brought back into presynaptic neuron using reuptake carriers -Reuptake of serotonin (5-HT), Dopamine (DA) and norepinephrine (NE) reuptake *Cocaine acts by blocking neuronal reuptake carriers 3. Neurotransmitter may diffuse out of synaptic cleft -Nitric oxide

5 Types of infectious pathogens

1. bacteria (extracellular) 2. viruses (intracellular) 3. fungi 4. parasites (protozoa, worms, insects) 5. prions (no immune defense) *Imperfect, TB and listeria are intracellular bacteria

Innate Immune System (Non-Specific)

1st line of defense: Skin (Integument) -Physical Barrier -Defensins (antibacterial enzymes) found on surface -Sweat has antimicrobial properties Respiratory System mucous membranes lined with cilia trap and push stuff towards oropharynx Mucous membranes produce lysozyme (secreted in tears and saliva) Gastrointestinal Tract: -Stomach secretes acid -Gut colonized by bacteria that out-compete invaders Macrophage: Immune defender that engulfs and consumes pathogens Mast Cell: Releases histamine and other chemicals that promote inflammation (reside in (and are activated in) the skin and mucous membranes) Granulocytes: Participate in inflammatory response Dendritic Cells: Presents antigen fragments of protein/molecule from pathogen to adaptive immune cells, inducing them to attack cells with displayed antigen Natural Killer Cell (type of non-specific lymphocyte): Destroys body's own cells when infected/goes after cancer cells -Detect down regulation of MHC by infected cells

Bone Remodeling

2 cells responsible for building and maintaining strong bones: OsteoBlasts build Bone using organic and inorganic materials, osteoclasts (polynucleated macrophages of bone) reabsorb it *Osteoclasts release enzymes that dissolve bone, releasing calcium into blood stream *Bones are vascular and innervated *osteoporosis: Most common bone disease in US: increased osteoclast resorption and slowing bone formation = loss of bone mass -During bone formation, calcium and phosphate are obtained from blood, during bone resorption they are released back *Bone remodeling occurs in response to stress -Endocrine hormones may affect bone metabolism: Parathyroid hormone (peptide hormone released by parathyroid glands in response to low blood calcium) promotes resorption of bone, increasing concentration of calcium and phosphate in blood *Parathyroid hormone also activates Vitamin D, promotes resorption, promotes new bone growth, thus overcompensating for resorption - Calcitonin, peptide hormone released by parafollicular cells in thyroid in response to high Ca2+, promotes bone formation (lowering blood calcium levels)

Mitosis (M Stage)

2 identical daughter cells from single cell Occurs in somatic cells (not involved in sexual reproduction 4 phases: Prophase (2 chromosomes) -Chromosomes condense, spindle forms, centriole pair separate and move toward opposite pole (located in centrosome, responsible for correct division of DNA, form spindle fibers from microtubules) *Asters: Anchor centrioles to cell membrane (star shaped structures that form around centrosome during mitosis -Nuclear membrane dissolves so spindle fibers can reach chromosomes, nucleoli become less distinct/disappear -Kinetochores: Protein structures located on centromeres that serve as attachment points Metaphase (2 chromosomes) -Chromosomes align at metaphase/equatorial plate, centriole pair at opposite ends Anaphase (4 chromosomes) -Sister chromatids separate from pulling of shortening kinetochore fibers Telophase (4 chromosomes) -Reverse of prophase -Spindle apparatus disappears, nuclear membrane reforms, nucleoli reappear, chromosomes uncoil Cytokinesis: Splitting of the cytoplasm and organelles b/n 2 daughter cells (2 cells with 2 chromosomes in each) *Each cell undergoes finite amount of divisions (between 20-50)

Avery, MacLeod, and McCarty (1944)

3 Americans: Avery, MacLeod, and McCarty identified material underlying transformation principle as DNA (degradation of DNA led to cessation of bacterial transformation) Took heat killed virulent bacteria and separated sub cellular components into different extracts -Found that addition of 1 particular extract to non-virulent enabled killing , and when this substance was treated with enzymes known to degrade DNA, the bacteria was not transformed and the mice lived

Heart

4 chambers, 2 pumps Right side accepts deoxygenated blood retuning from body and moves it to lungs by pulmonary arteries (pump 1: Pulmonary circulation) Left side of heart receives oxygenated blood from lungs by pulmonary veins and forces it out to body through aorta (second pump: systemic circulation) Each side of heart has one atrium and one ventricle Atria: Thin walled, blood received from venue cave (deoxygenated blood entering right) or pulmonary veins (oxygenated blood entering left) -Atria push blood into ventricles (contract and send blood to lungs (right) and systemic circulation (left)) *Ventricles more powerful/muscular (father to pump blood) Atria separated by atrioventricular valves, ventricles separated from vasculature by semilunar valves (both 3 leaflets, prevent back flow, valve that separates right ventricle from pulmonary circulation = pulmonary valve and valve that seperates left ventricle from aorta is aortic valve) *Atrioventricular Valves: LAB RAT (valve between Left Atrium and left ventricle: Bicuspid/mitral (2 leaflets), Right Atrium = Tricuspid (three leaflets)) *Left heart more muscular than right side (farther to pump) *Under autonomic control: Sympathetic signal speeds up heart and increases contractility, parasympathetic provided by VAGUS NERVE slow down HR)

Parathyroid Glands

4 small pea-like organs that sit on posterior surface of thyroid -Increases blood/calcium concentration Produces hormone PTH (Parathyroid hormone) -Antagonstic hormone to calcitonin, raising blood calcium levels (decreases excretion of calcium by kidneys, increase absorption of calcium in gut, increases bone resorption, freeing up calcium) -PTH: Plasma calcium rises, PTH secretion decreased -Promotes phosphorous homeostasis by increasing reabsorption of phosphate in kidney (excretion through urine) PTH activates vitamin D (required for absorption of calcium and phosphate in gut/kidneys) Overall, PTH causes significant increase in blood calcium levels, with little effect on phosphate (absorption of phosphate in gut and excretion in kidneys cancel each other out) *just like glucagon and insulin, PTH and calcitonin are antagonistic to each other (PTH increases serum calcium levels, whereas calcitonin decreases calcium levels

Anatomy of the Cardiovascular System

4-chambered heart, blood vessels, and blood -Vasculature consists of arteries, veins, and capillaries -After blood travels through veins, it returns to right side of heart (pumped into lungs to be reoxygenated) -Oxygenated blood returns to left side of heart to be pumped into body again

Blood

55% liquid, 45% cells Plasma: Liquid portion of blood, aqueous mixture of nutrients, salts, respiratory gases, hormones, and blood proteins (Can be further refined via removal of clotting factors into serum) Blood consists of: Erythrocytes, leukocytes, and platelets (only leukocytes have nuclei, RBC and platelets do not) All blood cells formed from hematopoietic stem cells (originate from bone marrow) -Production of blood cells and platelets = hematopoiesis (triggered by hormones, growth factors, cytokines: most notable = erythropoietin (secreted by kidney, stimulates mainly RBC development) and thrombopoetin (secreted by liver/kdineys and stimulates mostly platelet development) Blood Composition: Plasma proteins, salts, and water Plasma Proteins: Albumin (Osmotic balance, pH buffering), fibrinogen (clotting), immunoglobulins (defense (antibodies)) Salts: -Sodium, potassium, and calcium (Osmotic balance, pH buffering) -Magnesium, chloride, bicarbonate (Regulation of membrane potential) Water: Solvent for carrying other substances

A B+ person could receive blood from _______________ and donate blood to _____________ A B- person could receive blood from _______________ and donate blood to _____________

A B+ person could receive blood from B+, B-, O+, or O- A B+ person could donate blood to B+ or AB+ *No blood antigen is foreign with AB, no A or B antibodies present in plasma of AB (AB doesn't produce antibodies, so it doesn't attack any antigens, so even though type B produces B antigens and anti-A antibodies, these anti-A antibodies don't get produced by recipient/transferred from donor *+ is producing D, so they can't donate to someone who isn't A B- person could receive blood from B- or O- A B+ person could donate blood to B-, B+, AB- or AB+

Epithelial tissue

A body tissue that covers the surfaces of the body, inside and out Topmost layer of skin Barrier to protect the internal organs

Classification of hormones by target tissue

A hormone that directly stimulates a non-endocrine target tissue is referred to as a direct hormone Direct hormones (secreted and act directly on a target tissue) -Major effects on non-endocrine tissues -Insulin release by pancreas causes increased uptake of glucose by muscles The direct hormones secreted by the anterior pituitary are prolactin, endorphins, and growth hormone (GH). Tropic hormones: Hormone secreted and travels to target cell.organ, where it ttriggers release of another hormone, which causes changes in the physiological activity of target cells -Require intermediary to act (Gonadotropin-releasing hormone (GnRH) form the hypothalamus stimulates the release of luteinizing hormone (LH) and follicle simulating hormone (FSH). LH acts on gonads to stimulate testosterone/estrogen production. GnRH and LH do not cause direct changes, but stimulate production of another hormone -Usually originate in brain and anterior pituitary gland, allowing for coordination of multiple processes -Major effects on other endocrine tissues

Telophase 2

A nuclear membrane forms around the chromosomes in each new nucleus. Cytokinesis follows, and two daughter cells are formed Up to four daughter cells are produced per gametocyte (Oogenesis produces fewer than 4)

Bacteriophage A virologist wants to modify her virus so that it can infect more cell types. Which of the following viral genes should she change?

A virus that infects bacteria Don't enter bacteria, inject genetic material Have capsid, tail sheath (like a syringe that injects genetic material), and tail fibers (help phage recognize and connect to correct host cell) A virologist wants to modify her virus so that it can infect more cell types. Which of the following viral genes should she change? Tail fibers

Which of the following reasons best explains why infants have more brown fat than adults? (can be more than 1) A. It is less efficient at the electron transport chain. B. Infants cannot shiver to maintain body heat. C. A greater ratio of surface area to volume makes them more susceptible to cold temperatures. D. It helps infants cool themselves.

A, B, and C

Muscle Contraction: Relaxation (follows initiation and shortening of sarcomere)

ACh degraded in synapse. by acetylcholinesterase = Termination of single at neuromuscular junction and slows sarcolemma to repolarize -Signal decays, calcium release ceases, SR takes up calcium from sarcoplasm *SR controls intracellular Ca2+ concentration so muscles only contract when needed -ATP binds to myosin heads (frees them from actin -> Sarcomere returns to original width) -No calcium = myosin binding sites covered by tropomyosin = no contraction Rigor Mortis: ATP production stops, so myosin heads can't detach from actin, so muscles can't relax and lengthen

Hormones involved in feeding: ADH (Antidiuretic hormone/vasopressin), aldosterone, glucagon, ghrelin, leptin, and cholecystokinin

ADH (Antidiuretic hormone/vasopressin), aldosterone, glucagon, ghrelin, leptin, and cholecystokinin ADH and aldosterone trigger sensation of thirst (encourage fluid consumption) Glucagon (secreted by pancreas) and ghrelin (secreted by stomach and pancreas) stimulate feelings of hunger Leptin and cholecystokinin: stimulate feeling full/satiety

Antidiuretic hormone (ADH) and Aldosterone

ADH: A peptide hormone, also known as vasopressin, that promotes water retention by the kidneys. Produced in the hypothalamus and released from the posterior pituitary in response to high blood osmolarity -DIRECTLY alters permeability of collecting duct (Increases permeability) = more water reabsorbed = cell junctions leaky = increased blood volume/pressure and decreased blood osmolarity -Inhibit ADH = more urine (alcohol/caffeine) ADH also functions in the brain. -Release stimulated by low blood volume and high blood osmolarity Aldosterone: Steroid hormone regulated by renin-angiontensin-aldosterone system -Increases sodium reabsorption in distal convoluted tubule and collecting duct = increase water reabsorption = increased blood volume and pressure but no change in blood osmolarity

Thermoregulation (Skin)

Achieved by sweating, piloerection, vasodilation, vasoconstriction *Body cools self through sweating and vasodilation *Body warms self through piloerection, shivering, and vasoconstriction Sweating: -(controlled by autonomic NS), -Postganglionic sympathetic neurons that use acetylcholine innervate sweat glands and promote water secretion -> evaporation of water from skin absorbs body heat *Unlike all other Postganglionic sympathetic neurons, these neurons are cholinergic (release acetylcholine, like all other preganglionic neurons in Autonomic NS and postganglionic in parasympathetic NS), not noradrenergic (release norepinephrine ) -Arteriolar vasodilation maximizes heat loss (more blood to skin -> speeds process up) In cold conditions, arrestor pili muscles contract (Hair stands up = piloerection) -Traps heated air near skin -Arterioles that feed skin capillaries contract (limiting quantity of blood) = shivering (requires alot of ATP, converted to thermal energy) Fat insulation: Brown fat: less efficient electron transport chain = more heat energy released as fuel is burned

Passive and Active Immunity

Active: Immune system stimulated to produce antibodies against specific pathogen (natural: Antibodies generated by infected individual, or artificial: Vaccine = injection of antigen that activates B cells to produce antibodies (antigen = weakened or killed or part of microbe protein structure) Passive: Transfer of antibodies (transient/temporary: only antibodies, not plasma cells that produce them, are given)(placental/ breast milk) Risks of vaccines: Encephalitis, Gullain Barre syndrome (autoimmune: peripheral myelin attacked) 1998: Lancet article on autism and vaccines Active immunity requires weeks to build, whereas passive immunity is acquired immediately.

Adrenal Cortex

Adrenal glands on top of kidneys: Has cortex and medulla -Cortex secretes corticosteroids (Steroid hormones produced by adrenal cortex, includes glucocorticoids, mineralocorticoids, cortical sex hormones) Glucocorticoids: Steroid hormones, regulate glucose levels, affect protein metabolism -Cortisol and cortisone raise blood glucose, decrease immflamuation /immunity and protein synthesis -Release controlled by ACTH (CRF promotes release of ACTH from anterior pituitary, which promotes release of glucocorticoids from adrenal cortex Mineralocorticoids: salt and water homeostasis, aldosterone (increases BP and promotes sodium reabsorption)(under control of renin-angiotensin-aldosternone system) Cortical sex hormones (androgens and estrogens) Functions of corticosteroids: Salt (mineralocorticosteroids), sex (cortical sex hormones), sugar (glucocorticoids)

female reproductive system

All internal Gonads (ovaries) produce estrogen and progesterone -Located in pelvic cavity -Consists of thousands of follicles (multilayer sacs that contain, nourishment and protect immature ova/eggs) -Puberty to menopause, one egg/month is ovulated into peritoneal sac (lines abdominal cavity) -Then drawn into Fallopian tubes/oviduct that has cilia to propel egg forward -Tubes connected to uterus (sight of fetal development), ends in cervix, connects to vaginal canal (where sperm are deposited) External female anatomy = vulva

The Muscular System

All muscles rely on calcium ions for contractions All muscles are innervated/has nerves (by different parts of NS) Both smooth and cardiac muscles exhibit myogenic activity (will respond to nervous input, but dont require external signals) Skeletal Muscle: Responsible for voluntary movement -Innervated by Somatic nervous system -Repeating units of actin and myosin called sarcomeres: Appears striped/striated -multinucleated/Polynucleated -Contraction compresses venous structures and helps propel blood through low pressure venous system to heart (also lymph through lymphatic system) -Rapid contraction = shivering -Muscles of support = dark meat, red fibers (slow-twitch): higher myoglobin content, aerobic, lots of mitochondria -Muscles for quick bursts (breast in chickens) = white fibers (fast twitch) = less myoglobin = less iron = less red Smooth muscle: Involuntary movement -Controlled by autonomic NS -Digestion/peristalsis, respiratory tree, bladder, blood vessel walls, uterus... -Single nucleus -Contain actin and myosin, not well organized = not stripes -Capable of more sustained contractions than skeletal -Tonus: Constant state of low level contraction (blood vessels) -Myogenic activity: contraction without input from NS (respond to stretch/stimuli) -Regulation of BP by constructing/relaxing vasculature Cardiac muscle: Maintains rhythmic contraction of heart without nervous system input -Primarily uninucleated (can have 2) -Involuntary, autonomic NS -Striated like skeletal -Cardiac muscle cells connected by intercalated discs (contain gap junctions: connections between cytoplasm of adjacent cells, allow for direct ion flow) - allows for rapid/coordinated depolarization and efficient contraction -Cells define/maintain their own rhythm (Myogenic activity) -SA node to AV node -> depolarization spreads to bundle of His and branches -> Purkinje fibers -Vagus nerve provides parasympathetic outflow to slow HR -Norepinephrine (sympathetic neurons) and epi (adrenal medulla) can increase HR by increasing Ca2+

Action Potentials

All or nothing Relay electrical impulses down axon to synaptic bouton Cause release of neurotransmitters into synaptic cleft Resting membrane potential: -70 mV (inside of neuron negative relative to outside) k+ concentration inside cell averages 140 mM, outside cell 4 mM (favorable to move k+ out)(Potassium leak channels = loss of positive charge from cell makes outside slightly positively charged)(Pushed out due to concentration gradient) -As negative charge builds up inside cell, k+ drawn back in (pulled in due to electrical gradient) k+ equilibrium potential = -90 mV Sodium: 12 mM inside, 145 mM outside (driving force pushing sodium into cell, facilitated by sodium leak channels: Causes build up of electrical potential (60 mV)(positive because sodium going INTO cell)) Resting membrane potential (-70) closer to potassium value because membrane is more penetrable to potassium --70 dependent onintra/extra cellular ion concentrations, relative permeability of membrane to different ions, and charges of ions To remember the direction of ion movement by Na/K ATPase, think pump- K -in (pumpkin) (and Na2+ out) -Tons of ATP spent to maintain these gradients -ATPase restores resting potential *Excitatory inputs: Depolarization (raising membrane potential) Inhibitory Inputs: Hyperpolarization (lowering membrane potential) If axon hillock receives enough excitatory input to be depolarized to the threshold value (-55- -40 mV) an action potential will be triggered -Not every stimulus generates response If the cell is brought to threshold, voltage gated sodium channels open (sodium rushes in) -Strong electrochemical gradient that promote Na+ into cell (also, cell interior is more negative) Electrical gradient: Cell negative inside, Na positive Chemical gradient: Lower concentration of Na+ inside When Vm reaches +35, Na+ channels inactivated (Sodium channels now inactive from +35 to resting potential) Positive potential inside cell not only triggers voltage gated Na to inactivate, but also opens voltage gated potassium channels -Na depolarized cell, now electrochemical gradient favoring K driven out (overshoots, causing hyper polarization) -Functions of hyper polarization: Neuron refractory to further action potentials (Absolute refractory period = no amount of stimulation will cause firing, relative refractory period = greater than normal stimulation can cause Action potential)

Electrical conduction in the Heart

An ordinary cardiac contraction originates in, and is regulated by, the sinoatrial (SA) node. The impulse travels through both atria, stimulating them to contract simultaneously. The impulse then arrives at the atrioventricular (AV) node, which momentarily slows conduction, allowing for completion of atrial contraction and ventricular filling. The impulse is then carried by the bundle of His and its branches through the Purkinje fibers in the walls of both ventricles, generating a strong contraction. *SA node → atria → AV node → bundle of His → Purkinje fibers → ventricles *A person has a heart attack that primarily affects the wall between the two ventricles: Bundle of His most likely affected Impulse initiation: SA node (Intrinsic rhythm: 60-100 signals/minute w/o neuro input) (small collection of cells in right atrium) -Depolarization wave spreads from SA to atria (both contract simultaneously) -Atria to Ventricular filling is passive, but atrial systole (contraction) results in increase in atrial pressure that forces more blood into ventricles (additional blood = AV kick (accounts for 5-30% of cardiac output) -Next, signal goes to AV node (At junction of atria and ventricles) (delayed here to allow ventricles to fill completely before contraction) -Next, signal travels to bundle of His and branches (in the inter ventricular septum/wall) and to purkinje fibers (both distribute signal through ventricular muscle) -Muscle cells connected by intercalated discs (contain gap junctions b/n cytoplasm and cells: allows for coordinated ventricular contraction) *depolarization precedes cardiac muscle contraction, so electrical spikes on EKG occur before cardiac contractile event (P waves before atria contract, QRS complex occurs just before ventricles contract, T-waves = ventricular repolarization) *If autonomic innervation to heart were lost, heart would keep beating at intrinsic rate (SA node rate), but you wouldn't be able to change HR via parasympathetic/sympathetic NS

Antigens and Antibodies

Antigen: A substance (usually a foreign pathogenic protein) that can be targeted by an antibody -Displayed on surface of cell or can float, chyle (lymphatic fluid), air An antibody is a protein that acts against a specific antigen; Immune or protective protein whose synthesis is induced by the presence of foreign substances (antigens) in the body -AKA immunoglobulin Antibodies also called immunoglobulins (Ig) -Y shared molecules composed of two identical heavy chains and 2 identical light chains -Heavy and light chains held together by disulfide linkages and non-covalent interactions -Each antibody has antigen binding region and variable region (domain) at the tips of the Y (In this region, there are specific polypeptide sequences that bind to ONE specific antigen sequence) -B cell undergoes hypermutation of antigen binding region, trying to find best match (only B or T cells that bind antigen with high affinity survive, generating specificity called CLONAL SELECTION) -Constant region: Region that NK cells, macrophages, monocytes, eosinophils, etc have receptor for, and that can initiate complement cascade -Each B cell makes only 1 type of antibody (5 different subtypes: IgM, IgD, IgG, IgE, and IgA) -Cells can change isotope of antibody when stimulated by specific cytokines in ISOTYPE SWITCHING -Antibody production is energetically expensive, so to save energy, naive B cells (not yet exposed to antigen), wait in lymph node, where on exposure to correct antigen, B cell proliferates into 2 types of daughter cells: Plasma and memory B cells -Plasma cells: Produce large amounts of antibodies -Memory B cells: Stay in lymph node awaiting re-exposure to same antigen

Which of the following domains of life utilizes histones to wrap DNA and is noted for its unique ability to utilize alternative sources of energy such as sulfur and ammonia?

Archaea Archaea and eukaryotes use histones to condense their DNA, but only archaea are noted for their ability to use alternative sources of energy, such as chemosynthesis. -utilizes histones to wrap DNA and is noted for its unique ability to utilize alternative sources of energy such as sulfur and ammonia Reproduction: binary fission or budding Single, circular chromosome Only have one type of RNA polymerase, no nucleus,

Patterns of Evolution

Are similarities the result of sharing a common ancestor or sharing common environment? 3 patterns of evolution emerge: Divergent, parallel, and convergent Divergent: Independent development of dissimilar characteristics in 2 or more lineages sharing common ancestor (seals and cats both in order Carnivora, but very different) Parallel: Related species evolve in similar ways for a long period of time in response to analogous environmental selection pressures Convergent: Independent development of similar characteristics in 2 or more lineages not sharing common ancestor (fish and dolphins: Evolved similar features in adapting to conditions of water, but belong to different class) Rate of evolution measured by rate of change of genotype over period of time (related to severity of the evolutionary pressures on species) -Even if species is perfectly suited, there will still be a super slow rate of evolution from mutation -If organism lives in rapidly changing environment, rate of evolution will be greater *As species become more taxonomically distant, proportion of shared genome will decrease *Molecular clock model: Degree of difference in genome between 2 species is related tot he amount of time since the 2 species broke off from common ancestor -Correlate degree of genomic similarity with amount of time since two species split off from common ancestor; more similar the genomes = more recently the species separated

Bacteria

Around 5X10^30 on earth (more than plants and animals combined) Contain cell membrane and cytoplasm Some have flagella fimbriae (like cilia) Bacterial ribosome much smaller than eukaryotic ribosome (neither ribosome is surrounded by membrane) Some bacteria are mutualistic symbioses (both humans and bacteria benefit from relationship, like in Vitamin K and biotin, prevent overgrowth of harmful bacteria) 3 shapes: -Spherical/Cocci -Rod-shaped/Bacilli (like E.coli) -Spiral-shaped/spirilli (Rare, but three common ones are treponema pallidum (causes syphillis), borrelia burgdorferi (causes Lyme disease), and Weil's disease)

Vasculature

Arteries: Blood away from heart (thick) -Most contain oxygenated blood (pulmonary and umbilical arteries carry deoxygenated) -Largest is aorta, branches into common carotids, subclavian -Branch into arterioles, which become capillaries -Capillaries join together to form venules, which become veins -Venous blood empties into superior and inferior venae cave for entry into right side of heart -All blood vessels lined with endothelial cells (allow vasodilatation/vasoconstriction, white blood cell passage, release chemicals to form clots) *Arteries have much more smooth muscle than veins *Arteries are highly muscular and elastic *Capillaries are vessels with single endothelial cell layer (tiny, red blood cells pass in single-file) -Allow easy diffusion of CO2 and O2 (gases), nutrients (glucose), and waste (ammonia, urea), endocrine signals/hormones -Can be delicate, damage = blood in interstitial space = bruise *No smooth muscle Veins: Thin walled, inelastic, transport deoxygenated blood to heart (except for pulmonary and umbilical veins) -Venules connect capillaries to veins -Less smooth muscle = less recoil - veins rely on external force to propel blood (Veins surrounded by skeletal muscle, squeeze veins as muscles contract) -Blood clot in deep legs veins = DVT (may travel to heart/lungs = pulmonary embolus) -More stretch = able to accommodate larger volumes of blood (3/4 of blood volume in veins, even though CO is same on both side of heart) -Most veins flow up from lower body to heart against gravity (High pressure in lower legs, can exceed 200 mmHg) -Veins have valves to prevent back flow, valve failure = varicose veins (distended) *ONLY VEINS CONTAIN VALVES *Heparin and warfarin prevent blood clots

Autosomal Cells vs Germ Cells

Autosomal: Diploid (2N) -2 copies of each chromosome -Division results in 2 genetically identical daughter cells Germ Cells: Haploid (N) -Only one copy of each chromosome -Daughter cells are not equivalent We inherit 23 chromosomes from each parent

Which of the following is the correct sequence of the development of a mature sperm cell? A. Spermatid → spermatocyte → spermatogonium → 2° spermatocyte → spermatozoan B. Spermatogonium → 1° spermatocyte → 2° spermatocyte → spermatid → spermatozoan C. Spermatozoan → 1° spermatocyte → 2° spermatocyte → spermatogonium → spermatid D. Spermatogonium → 1° spermatocyte → 2° spermatocyte → spermatozoan → spermatid

B Diploid cells called spermatogonia differentiate into primary spermatocytes, which undergo the first meiotic division to yield two haploid secondary spermatocytes. These undergo a second meiotic division to become immature spermatids. The spermatids then undergo a series of changes leading to the production of mature sperm, or spermatozoa.

Type B- blood will produce both _ antigens and ___ antibodies (referring to negative sign)

B antigens and anti-Rh antibodies Antigens are markers that identify cells as self Antibodies detect cells that are non-self

How do antibodies become specific for a given antigen?

B cells mature in bone marrow and have some specificity, but then antibodies that respond undergo hypermutation/rapid mutation of antigen binding sites, and only B cells with highest affinity for antigen survive, increasing specificity over time

B cells VS T Cells Site of Development: Site of Maturation: Major Functions: Specific or non-specific: Humoral or cell-mediated:

B cells: Antigens stimulate cell to divide and produce antibodies that neutralize invaders or tag for killing T cells: Killer T cells destroy infected cell in which it detects presence of antigens (other T cells, like helper and regulatory T cells, coordinate immune response) Site of Development: -B cells: Bone Marrow -T cells: Bone Marrow Site of Maturation: -B cells: Bone marrow, but activated in lymph nodes and spleen -T cells: Thymus Major Functions: -B cells: Produce antibodies -T cells: Coordinate immune system and directly kill infected cells Specific or non-specific: -B cells: Specific -T cells: Specific Humoral or cell-mediated: -B cells: Humoral -T cells: Cell-Mediated

Anaphase 2

Centromere divides; sister chromatids separate; chromatids pulled to opposite poles by spindle fibers

Fluid Balance

Balance between fluid volume and solute concentration between blood and interstitium (cells surrounding blood vessels) -Hydrostatic Pressure: Force per unit area that the blood exerts against the vessel walls -Generated by contraction of heart and elasticity of arteries -Measured upstream in large arteries as BP -Pushes fluid out of bloodstream into interstitium through capillary walls -Leaky -Osmotic (oncotic) pressures:(think on-constant pressure) -sucking pressure generated by solutes attempting to suck water into bloodstream -Mostly attributable to plasma proteins -At the arteriole end of a capillary bed, hydrostatic pressure (pushing fluid out) is larger than oncotic pressure (drawing fluid in) -Net influx of water from circulation -As fluid moves out, hydrostatic pressure drops significantly, osmotic pressure stays same. Therefore, at venule end of capillary bed, hydrostatic pressure (pushing fluid out) has dropped below oncotic pressure (drawing fluid in), and there is net influx of water back into circulation -Hydrostatic pressure pushed fluid out of vessels and is dependent on blood pressure generated by the heart and elastic arteries. Osmotic pressure pulls fluid back into the vessels and is dependent on the number of particles dissolved in the plasma Most are proteins, often refer to this as oncotic pressure Balance of opposing forces: Starling forces Accumulation of too much fluid: edema *Lymphatic fluid (lymph) returned to circulatory system by thoracic duct, but blockage can cause edema

Sarcomere

Basic contractive unit of skeletal muscle The sarcomere is the contractile unit in striated muscle cells. One sarcomere is represented by the area between the two vertical lines, referred to as the Z-lines. In addition, the Z-lines anchor the thin filaments. -Made of thick (myosin) and thin (actin and 2 other proteins (troponin and tropomyosin)) filaments *Think acTHIN = thin filaments -troponin and tropomyosin help regulate interaction b/n actin and myosin fibers -Another protein, titin, acts as spring and anchors filaments together (prevents excess stretch) -Each sarcomere divided into lines, zones and bands Z lines define boundaries (end of alphabet, end of sarcomere) M line runs down center, through middle of myosin filaments I band = only thin filaments (no myosin/thick filaments ) H band = only thick filaments A bands contains whole thick filaments , including that which overlap with thin filaments (remains constant during contraction, unlike H and I bands, because it spans entire length (filaments slide over, don't change length) *During contraction, H zone, I band, distance between Z lines, and distance between M lines all become smaller

How do bile and pancreatic lipase work together to digest fats?

Bile = mechanical digestion of fats (emulsifies and increases surface area by placing in micelles) Pancreatic lipase = chemical digestion of fats (breaks ester bonds/hydrolyzes bonds holding lipids together) *Bile produced in liver *Bile composed of pigments (especially bilirubin from hemoglobin breakdown), cholesterol, and BILE SALTS Bile Salts: Derived from cholesterol, not enzymes (dont break bonds, but hydrophobic and hydrophilic regions help EMULSIFY FATS and cholesterol into micelles, prevents spontaneous separation, increases surface area = increases rate lipase can act)

Dendrotoxins are proteinaceous antagonists of voltage-gated potassium channels in motor neurons. A dendrotoxin-laced bite from the Black Mamba snake would most likely be dangerous because: over-stimulating muscles in the respiratory system will cause difficulty breathing. Blocking voltage-gated potassium channels will make a motor neuron unable to repolarize and, thus, they will continually innervate muscles. the toxin will render pacemaker cells in the heart unable to initiate action potentials. blocking voltage-gated potassium channels abolishes all resting membrane potentials hyper-exciting the sleep center of the brain can induce delirium or coma.

Blocking voltage-gated potassium channels will render a neuron unable to repolarize. This will cause extended periods of depolarization, which results in copious neurotransmitter release. If the target cell is a muscle, this could effectively induce tetanic contraction. While this would have widespread effects throughout the body, paralysis of the respiratory system is particularly life-threatening Action potentials are initiated by an increase in sodium coming into the cell. Blocking potassium channels would not cause initiation to stop.

Portal systems in the body

Blood passes through 2 capillary beds in series before returning to the heart Three portal systems: Hepatic portal system: Connects gut tube and liver Hypophyseal portal system: Connects hypothalamus to anterior pituitary Renal portal system: Renal artery branches out, passes through medulla, enters afferent arterioles (away from artery towards glomeruli, just like afferent neurons carry info away from skin towards CNS) -> Capillary tufts from afferent arterioles = glomeruli in Bowman's capsule (first capillary bed -> go to efferent arterioles that form secondary capillary bed (these surround the loop of Henle and are known as vasa recta) which surround nephron, before leaving kidney through renal vein *Image: Microanatomy of the renal vascular system and the nephron Vasa Recta: Second capillary bed within the kidney that removes substances from the interstitium of the kidney to be returned to systemic circulation

Bone Composition

Bone is a connective tissue derived from embryonic mesoderm. Bone is much harder than cartilage, relatively lightweight -Strength comes from COMPACT bone: dense and strong (forms most of outer layer) -Spongy/Cancellous bone: Lattice structure, consists of many spaces between bony spicules (points) known as trabeculae (filled with either red or yellow bone marrow)(site of marrow production)(found in interior core of one, helps distribute force) -Red marrow is filled with hematopoietic stem cells (involved in erythrocyte formation) -Yellow marrow is composed primarily of fat (relatively inactive)(infiltrated by adipose tissue) Three parts of a bone: Diaphysis, metaphysis, and epiphysis Long bones: Characterized by cylindrical shafts called diaphyses that swell at each end to form metaphysis, that terminate in epiphyses -Diaphysis and metaphysis are full of bone marrow, but epiphyses use spongey core instead -At internal edge of epiphysis epiphyseal (growth) plate: Cartilaginous structure, site of longitudinal growth, filled with mitotic cells but then starts to close during puberty -Periosteum: Surrounds long bone to protect/serve as site of muscle attachment (needed for growth and repair) Outermost portion of bones = compact bone, internal core is spongy bone

For each segment of nephron listed, what are major functions? Bowman's capsule: Proximal Convoluted tubule: Descending limb of the loop of Henle: Ascending limb of the loop of Henle: Distal convoluted tubule Collecting duct

Bowman's capsule: -Site of filtration through which water, ions, amino acids, vitamins and glucose pass (everything but cells and proteins) Proximal Convoluted tubule: Controls solute identity, Site of bulk reabsorption for vitamins/amino acids/glucose while secreting potassium and hydrogen ions, ammonia, and urea Descending limb of the loop of Henle: Water reabsorption, uses medullary concentration gradient *permeable to water, not salt, so as filtrate moves into more osmotically concentrated renal medulla, water is reabsorbed from filtrate *Vasa recta and nephron flow in opposite ways = countercurrent multiplier system that allows max reabsorption of water Ascending limb of the loop of Henle: Salt reabsorption (both active and passive), not permeable to water -dilution of urine in diluting segment in outer medulla *Because salt is actively absorbed here, filtrate becomes hypotonic compared to blood Distal convoluted tubule: like PCT, important for solute identity by reabsorbing salts while secreting potassium and hydrogen ions, ammonia, and urea -Responsive to aldosterone -Drug that inhibits the Na+/Cl-symporter in the distal convoluted tubule Collecting duct: Urine concentration, variable permeability allows water to be reabsorbed as needed -Responsive to aldosterone and ADH

List the structures in the excretory pathway from filtrate entering nephron to excretion of urine

Bowman's space -> proximal convoluted tube - descending limp of the loop of Henle -> ascending limb of the loop of Henle - Distal convoluted tubule -> collecting duct -> renal pelvis - ureter - bladder -> urethra

Central Nervous System

Brain and spinal cord Brain consists of white matter and grey matter -White matter consists of axons encased in myelin sheaths (deeper than grey matter) -Grey matter consists of unmyelinated cell bodies and dendrites Spinal cord: Extends downward from brainstem, divided into 4 regions (cervical, thoracic, lumbar, sacral) Spinal cord protected by vertebral column, which transmits nerves at the space between adjacent vertebrae -White matter lies on outside of cord, grey matter deep inside -Axons of motor and sensory neurons are in spinal cord (sensory neurons bring info in from periphery and enter on the dorsal (backside) of the spinal cord -Cell bodies of sensory neurons found in the dorsal root ganglia -Motor nueorns exit the spinal cord ventrally (side closest to from) -Sensory neurons transmit info about pain, temp, and vibration up to brain and have cell bodies int he dorsal root ganglia towards back -Motor neurons run from brain along the opposite side of spinal cord and in the ventral root to control movements of skeletal muscles and glandular secretions

Digestive System

Breaks down food into absorbable units that enter the blood for distribution to body cells Two types of digestion that occur: 1. Intracellular (part of metabolism): involves oxidation of glucose/fatty acids for energy 2. Extracellular Digestion: Process of getting nutrients from food occurs in lumen of the alimentary canal (runs from mouth to anus, sectioned by sphincters) Digestion: Breakdown of food into constituent organic molecules (Carbs and starches into monosaccharides, lipids (Fats) into fatty acids and glycerol, protein into amino acids) *3 main sources of energy = carbs, fats, and proteins Digestion divided into mechanical (physical breakdown, not breaking chemical bonds) and chemical (enzymatic cleavage, like peptide bonds of proteins) *Chemical digestion of carbs occurs in mouth and small intestine (salivary amylase/ptyalin/lipase: higher specificity for rapidly dissolving starches and pancreatic amylase: higher specificity for less soluble/more non-polar states in SI) *Salivary amylase hydrolyzes starch into smaller sugars (maltose and dextrins) while lipase catalyzes hydrolysis of lipids *minimal chemical digestion occurs in mouth (doesn't stay there long) *Protein digestion occurs in stomach and small intestine *Digestion occurs primarily in oral cavity, stomach and duodenum, while absorption occurs primarily in jejunum Absorption: Transfer of products of digestion into circulatory system for distribution *the process by which substances are taken up, into, or across tissues Digestive Tract begins with oral cavity (mouth: Mastication increases surface area to volume ratio), bolus enters pharynx (food and air), then esophagus transports to stomach *Pharynx connects mouth and posterior nasal cavity to esophagus Esophagus: Top third is composed of skeletal (Under somatic/voluntary control), bottom composed of smooth muscle (Controlled, along with rest of GI tract, under autonomic/involuntary control *Swallowing initiated in oropharynx (constitutes upper esophageal sphincter). As bolus approaches stomach, a muscular ring (lower esophageal sphincter/cardiac sphincter) relaxes and opens The cardiac sphincter lies between the esophagus and the stomach. *Damaged/weakened lower esophageal sphincter = GERD From stomach to small intestine, then large intestine, then rectum *Salivary glands, pancreas, liver, and gallbladder help produce enzymes/lubrication *3 pairs of major salivary glands: parotid, submandibular, and sublingual (Mostly promoted by paraSNS, but SNS has some input, like increasing viscosity) *First part of small intestine = duodenum

Which of the following correctly pairs the stage of development of an egg cell with the relevant point in a woman's life cycle? A. From birth to menarche—prophase II B. At ovulation—metaphase I C. At ovulation—metaphase II D. At fertilization—prophase II

C From the time of birth until shortly before ovulation, all egg cells are arrested at the prophase stage of meiosis I. These cells are referred to as primary oocytes. At ovulation, the egg cell has completed meiosis I and is now arrested in metaphase II as a haploid cell called a secondary oocyte (Ovulated from follicle at day 14 of menstraul cycle). When a sperm penetrates the outer layers of the secondary oocyte, it completes meiosis II to become a mature ovum

Which of the following associations between a hormone and its category is INCORRECT? A. Aldosterone—mineralocorticoid B. Testosterone—cortical sex hormone C. ADH—mineralocorticoid D. Cortisone—glucocorticoid

C Unlike the other hormones listed here, ADH is not secreted by the adrenal cortex and is therefore not a -corticoid. Rather, ADH is a peptide hormone produced by the hypothalamus and released by the posterior pituitary that promotes water reabsorption.

Eukaryotic Cells

Can be unicellular or multicellular Contain true nucleus enclosed in membrane Nucleus encoded genetic material in DNA (organized into chromosomes) DNA contains coding regions called genes Linear DNA is wound around organizing proteins called histones, then further wound into linear strand called chromosomes Nuclear envelope/membrane (double membrane) surrounds/separates nucleus from cytoplasm Nucleolus: Ribosomal RNA (rRNA) synthesized (takes up 25% volume of nucleus, looks like dark spot) Membranes consist of phospholipid bilayer Membrane surfaces are hydrophilic, inner portion is hydrophobic (highly selective barrier) Cytosol allows for diffusion of molecules throughout cell Contains mitocondria, lysosomes, rough and smooth ER, peroxisomes, Golgi apparatus Unique characteristic: Tissues with division of labor (Four types: Epithelial, connective, muscle, and nervous) Reproduce through mitosis

Cardiovascular System: Nutrients, waste, and hormones

Carbs and amino acids absorbed into capillaries of small intestines and enter systemic circulation via hepatic portal system -Fats absorbed into lacteals in small intestines/bypass hepatic portal circulation to enter via thoracic duct , then packaged into water soluble lipoproteins Once hormones reach their target tissues, they can activate cells surface receptors (peptide hormones) or diffuse into cell to activate intracellular/intranuclear receptors (steroid hormones)

Cardiac Output (CO)

Cardiac Output (CO) = Total blood volume pumped by ventricle in one minute = Heart Rate (HR in bpm) X Stroke Volume (SV= volume of blood pumped per beat) *About 5 liters per minute (increase with sympathetic system)

Humoral Immunity

Centered on antibody production by plasma cells, which are activated B cells (Based on activity of B cells): Production of antibodies Can take up to a week Antibodies specific to microbe/produced by B cells (lymphocytes that originate in bone marrow and activated in spleen and lymph nodes) -When antibody binds to antigen, response depends on location -Antibody secreted into bodily fluids = 3 possibilities 1. Once bound to antigen, antibodies attract other WBCs to phagocytize (opsonization)(Opsonize = mark for destruction) 2. Antibodies cause pathogen to clump together/agglutinate (forms large insoluble complexes that can be phagocytized) 3. Antibodies can block ability of pathogen to invade tissue (Neutralizing it)

Right and Left shift of bicarbonate buffer system/Oxyhemoglobin binding curve

Causes of right shift of oxyhemoglobin curve: -Exercise = increase Pa CO2 (Partial pressure of CO2 in mmHg) - Increased H+/decreased pH -Increased Temp -Increased 2,3-biphosphoglycerate (2,3 - BPG), a side product of glycolysis in RBCs Left shift: Decreased PaCO2, decreased H+ concentration, increased pH, decreased temp, decreased BPG, hyperventilation resulting in excess CO2 blowoff (respiratory alkalosis) -Kidneys can compensate by increasing excretion to bicarbonate -Renal tubular acidosis type 1 = kidney cant excrete acid effectively = buildup of protons in blood (metabolic acidosis) = buffer system shift to left (Increase respiratory rate/blow off more CO2 to compensate) Bicarbonate buffer system links respiratory and renal system

Interferons

Cells that have been infected with viruses produce interferons, that prevent viral replication and dispersion Cause nearby cells to decrease production and decrease permeability Upregulate MHC class 1 and 2 molecules, resulting in increased antigen presentation *responsible for "flu-like" symptoms seen in viral infection

In the nephron, increasing the permeability of the collecting duct to water has the effect of: Which of the following statements is FALSE? A. ADH increases water reabsorption in the kidney. B. Aldosterone indirectly increases water reabsorption in the kidney. C. ADH acts directly on the proximal convoluted tubule. D. Aldosterone stimulates reabsorption of sodium from the collecting duct.

Concentrating urine Majority of water reabsorbed in collecting duct, which is under control of antidiuretic hormone (ADH)(Vasopressin) ADH causes filtrate to become more concentrated by increasing permeability of collecting duct so more water is absorbed Correct Answer: C Explanation: All of the answer choices describe ADH or aldosterone. These two hormones ultimately act to increase water reabsorption in the kidney; their respective mechanisms of action, however, are different. ADH increases water reabsorption by increasing the permeability of the collecting duct to water, whereas aldosterone stimulates reabsorption of sodium from the distal convoluted tubule and collecting duct. Using this knowledge, we can now attack the answer choices. Choice (C) is the correct answer because ADH does not act on the proximal convoluted tubule, but rather the collecting duct.

The Endocrine System

Consists of organs, known as glands, that secrete hormones Hormones: Signaling molecules that are secreted directly into the bloodstream to distant target tissues -Bind to receptors, inducing change in gene expression or cellular functioinng -messangers to the endocrine system (Essential role in communication) -Pancreas produces insulin and glucagon -Insulin induces the transport of glucose into organs and the storage of excess glucose when blood glucose concentrations are high -Glucagon triggers the release of sugar stores (raise blood glucose concentrations) Diabetes 1: Autoimmune disease in which insulin producing cells in the islets of Langerhans are destroyed Type 2 Diabetes: End organ insensitivity to insulin

Posterior Pituitary

Contains the nerve terminals of neurons with cell bodies in the hypothalamus -Receives/releases and stores two hormones produced by hypothalamus: Antidiuretic hormone (ADH) and Oxytocin (posterior pituitary does not actually synthesize any hormones itself) -ADH secreted in response to low blood volume (sensed by baroreceptors) or increased blood osmolarity (sensed by osmoreceptors) -Action at level of collecting duct (increases permeability to water, increases reabsorption in nephron = greater retention = increased blood volume and higher BP) -Oxytocin: Stimulates smooth muscle contraction and milk let down: Secreted during childbirth (allows for coordinated contraction of uterine smooth muscle, stimulated by suckling, involved in bonding behavior) -Unusual in that it has positive feedback loop: Release of oxytocin promotes uterine contraction, which promotes more oxytocin release, which promotes more uterine contractions..... spiraling forward

A bacterial cell is noted to be resistant to penicillin. The bacterium is transferred to a colony that lacks the fertility factor, and the rest of the colony does not become resistant to penicillin. However, the penicillin-resistant cell has also started to exhibit other phenotypic characteristics, including secretion of a novel protein. Which of the following methods of bacterial recombination is NOT likely to account for this change? A. Conjugation B. Transformation C. Transduction D. Infection with a bacteriophage

Correct Answer: A Explanation: A bacterial cell that does not rapidly cause a phenotypic change in the rest of the colony is likely not F+, meaning that this cell is not able to form a sex pilus for conjugation. The expression of new phenotypic characteristics indicate that this bacterium may have acquired some quantity of genetic information, which can be acquired by either transformation, choice (B), or transduction (which occurs via bacteriophage infection), choices (C) and (D).

After infection of a cell, a viral particle must transport itself to the nucleus in order to produce viral proteins. What is the likely genomic content of the virus? A. Double-stranded DNA B. Double-stranded RNA C. Positive-sense RNA D. Negative-sense RNA

Correct Answer: A Explanation: A virus that requires transport to the nucleus in order to produce viral proteins likely requires use of nuclear RNA polymerase in order to create mRNA that can be translated to protein. Only DNA viruses require this transport before any protein can be synthesized.

Which of the following neurotransmitters is used in the ganglia of both the sympathetic and parasympathetic nervous systems? A. Acetylcholine B. Dopamine C. Norepinephrine D. Serotonin

Correct Answer: A Explanation: Acetylcholine is the neurotransmitter released by the preganglionic neuron in both the sympathetic and parasympathetic nervous systems. The postganglionic neuron in the sympathetic nervous system usually releases norepinephrine, while the postganglionic neuron in the parasympathetic nervous system releases acetylcholine. A neurotransmitter found throughout NS (somatic neurons, preganglionic parasympathetic and sympathetic nerves, and postganglionic parasympathetic neurons) -Metabolized by acetylcholinesterase

Sarin is a potent organophosphate that can be used in chemical warfare. As an inhibitor of acetylcholinesterase, sarin causes excessive buildup of acetylcholine in all synapses where it is the neurotransmitter. Which of the following symptoms would be most likely to be seen in an individual with sarin poisoning? A. Increased urination and increased sweating B. Increased urination and decreased sweating C. Decreased urination and increased sweating D. Decreased urination and decreased sweating

Correct Answer: A Explanation: An excess of acetylcholine will lead to activation of all parasympathetic neurons, preganglionic sympathetic neurons, and the postganglionic sympathetic neurons that innervate sweat glands. Because the parasympathetic nervous system causes contractions of the bladder, one would expect increased urination. The increased activation of sweat glands would lead to increased sweating as well.

Which of the following correctly pairs the molecule with its primary site of absorption? A. Chylomicrons—lacteals B. Amino acids—large intestine C. Vitamins A and E—stomach D. Cholesterol—ascending colon

Correct Answer: A Explanation: Chylomicrons contain triacylglycerols, cholesteryl esters, and fat-soluble vitamins and are secreted by intestinal cells into lacteals. Amino acids, fat-soluble vitamins (like vitamins A and E), and cholesterol are all absorbed in the small intestine. Chylomicron: soluble lipid molecule that consists of triglycerides and esterfied cholesterol molecules; absorbed into lacteals from he digestive tract

At the venous end of a capillary bed, the osmotic pressure: A. is greater than the hydrostatic pressure. B. results in a net outflow of fluid. C. is significantly higher than the osmotic pressure at the arterial end. D. causes proteins to enter the interstitium.

Correct Answer: A Explanation: The exchange of fluid is greatly influenced by the relative balance between the hydrostatic and osmotic pressures of blood and tissues. The osmotic (oncotic) pressure remains relatively constant; however, the hydrostatic pressure at the arterial end is greater than the hydrostatic pressure at the venous end. As a result, fluid moves out of the capillaries at the arterial end and back in at the venous end. Fluid is reabsorbed at the venous end because the osmotic pressure exceeds the hydrostatic pressure. Proteins should not cross the capillary wall under normal circumstances.

In emphysema, the alveolar walls are destroyed, decreasing the recoil of the lung tissue. Which of the following changes may be seen in a patient with emphysema? A. Increased residual volume B. Decreased total lung capacity C. Increased blood concentration of oxygen D. Decreased blood concentration of carbon dioxide

Correct Answer: A Explanation: The intrinsic elastic properties of the lung are important during exhalation as the passive recoil of lung tissue helps decrease lung volume. With decreased recoil, the patient will have difficulty exhaling completely, increasing the residual volume. The total lung capacity would be expected to increase in this case because there would be less recoil opposing inhalation, eliminating choice (B). With decreased alveolar surface area, one would expect decreased gas exchange, which would decrease blood concentrations of oxygen while increasing blood concentrations of carbon dioxide, eliminating choices (C) and (D).

Under normal physiological circumstances, the primary function of the nephron is to create urine that is: A. hypertonic to the blood. B. hypotonic to the blood. C. isotonic to the filtrate. D. hypotonic to the vasa recta.

Correct Answer: A Explanation: The kidneys function to eliminate wastes such as urea, while reabsorbing various important substances such as glucose and amino acids for reuse by the body. Generation of a solute concentration gradient from the cortex to the medulla allows a considerable amount of water to be reabsorbed. Excretion of concentrated urine serves to limit water losses from the body and helps to preserve blood volume. Thus, the primary function of the nephron is to create urine that is hypertonic to the blood, making choice (A) the correct answer and eliminating choices (B) and (D). Water should be reabsorbed from the filtrate, so urine should be hypertonic to the filtrate, eliminating choice (C). *A hypotonic solution is one in which the concentration of solutes is greater inside the cell than outside of it, and a hypertonic solution is one where the concentration of solutes is greater outside the cell than inside it.

Increased synthetic activity of the parathyroid glands would lead to: A. an increase in renal calcium reabsorption. B. a decrease in the rate of bone resorption. C. a decrease in basal metabolic rate. D. a decrease in blood glucose concentration.

Correct Answer: A Explanation: The parathyroid glands secrete parathyroid hormone (PTH), a hormone that functions to increase blood calcium levels. An increase in synthetic activity of the parathyroid glands would lead to an increase in PTH and, therefore, an increase in blood calcium levels through three mechanisms: increased calcium reabsorption in the kidneys (decreased excretion), increased bone resorption, and increased absorption of calcium from the gut (via activation of vitamin D).

Which region of the kidney has the lowest solute concentration under normal physiological circumstances? A. Cortex B. Outer medulla C. Inner medulla D. Renal pelvis

Correct Answer: A Explanation: The region of the kidney that has the lowest solute concentration is the cortex, where the proximal convoluted tubule and a part of the distal convoluted tubule are found. The solute concentration increases as one descends into the medulla, and concentrated urine can be found in the renal pelvis.

Some studies suggest that in patients with Alzheimer's disease, there is a defect in the way the spindle apparatus attaches to the kinetochore fibers. At which stage of mitotic division would one first expect to be able to visualize this problem? A. Prophase B. Metaphase C. Anaphase D. Telophase

Correct Answer: A Explanation: The spindle apparatus first interacts with the kinetochore fibers near the end of prophase. While the spindle apparatus aligns the chromosomes at the equatorial plate during metaphase, choice (B), the initial connection of the microtubule to the kinetochore occurs in prophase. Kinetochore: Protein structure, located at the centromere, that provides a place for spindle fibers to attach to the chromosome

Which of the following does NOT describe connective tissue cells? A. They account for most cells in muscles, bones, and tendons. B. They secrete substances to form the extracellular matrix. C. In organs, they tend to form the stroma. D. In organs, they provide support for epithelial cells.

Correct Answer: A Explanation: While bones and tendons are composed predominantly of connective tissue cells, muscle tissue is considered a different tissue type. Other examples of connective tissue include cartilage, ligaments, adipose tissue, and blood. Connective tissue often secretes substances to form the extracellular matrix, such as collagen and elastin, eliminating choice (B). Choices (C) and (D) are essentially identical and can both be eliminated: in organs, connective tissue often forms the support structure for epithelial cells, called the stroma.

Which of the following biomolecules does NOT drain to the liver before arriving at the right side of the heart? A. Cholecalciferol (Vitamin D) B. Threonine (an amino acid) C. Fructose (a monosaccharide) D. Pantothenic acid (vitamin B5)

Correct Answer: A Explanation: While the capillaries from the intestine come together to form the portal vein, which drains to the liver, the lacteals come together to form the thoracic duct, which drains directly into the left subclavian vein. Therefore, fat-soluble compounds do not pass through the liver before reaching the right heart. Only choice (A), vitamin D, is fat-soluble.

In an experiment, enteropeptidase secretion was blocked. As a direct result, levels of all of the following active enzymes would likely be affected EXCEPT: A. trypsin. B. aminopeptidase. C. chymotrypsin. D. carboxypeptidase A.

Correct Answer: B Explanation: Aminopeptidase is a brush-border peptidase secreted by the cells lining the duodenum; it does not require enteropeptidase for activation. Both trypsinogen and procarboxypeptidases A and B are activated by enteropeptidase, eliminating choices (A) and (D). Once activated, trypsin can activate chymotrypsinogen; if trypsinogen cannot be activated, then chymotrypsinogen will not be activated either, eliminating choice (C).

To facilitate the process of birth, an infant's head is somewhat flexible. This flexibility is given in part by the two fontanelles, which are soft spots of connective tissue in the infant's skull. With time, the fontanelles will close through a process known as: A. endochondral ossification. B. intramembranous ossification. C. bone resorption. D. longitudinal growth.

Correct Answer: B Explanation: Bones form in one of two ways: endochondral ossification and intramembranous ossification. Endochondral ossification is the replacement of a cartilaginous skeleton with bone and occurs mostly in long bones, eliminating choice (A). Intramembranous ossification is the formation of bone from undifferentiated connective tissue cells (mesenchyme) and occurs mostly in the skull, making choice (B) the correct answer. Bone resorption is the breakdown of bone, not its formation, eliminating choice (C). Longitudinal growth occurs in long bones and is responsible for increasing height over time, but does not play a role in fontanelle ossification, eliminating choice (D).

The intrapleural pressure is necessarily lower than the atmospheric pressure during: A. inhalation, because the expansion of the chest cavity causes compression of the intrapleural space, decreasing its pressure. B. inhalation, because the expansion of the chest cavity causes expansion of the intrapleural space, decreasing its pressure. C. exhalation, because the compression of the chest cavity causes compression of the intrapleural space, decreasing its pressure. D. exhalation, because the compression of the chest cavity causes expansion of the intrapleural space, decreasing its pressure.

Correct Answer: B Explanation: During inhalation, the chest cavity expands, causing expansion of the intrapleural space. According to Boyle's law, an increase in volume is accompanied by a decrease in pressure. When the intrapleural pressure (and, by extension, the alveolar pressure) is less than atmospheric pressure, air enters the lungs. During exhalation, these pressure gradients reverse; thus, during exhalation, intrapleural pressure is higher than atmospheric pressure, not lower.

12. An autoimmune disease attacks the voltage-gated calcium channels in the nerve terminal. What is a likely symptom of this condition? A. Spastic paralysis (inability to relax the muscles) B. Flaccid paralysis (inability to contract the muscles) C. Inability to reuptake neurotransmitters once released D. Retrograde flow of action potentials

Correct Answer: B Explanation: First, consider the function of voltage-gated calcium channels. When the nerve terminal depolarizes, voltage-gated calcium channels open, allowing for influx of calcium. This influx of calcium triggers fusion of the synaptic vesicles containing neurotransmitters with the membrane of the neuron at the nerve terminal. This allows for exocytosis of the neurotransmitters into the synapse. If a disease blocked the influx of calcium, there would be no release of neurotransmitters. A lack of neurotransmitters means that the neuron cannot send signals. Thus, any symptoms resulting from this disease would be due to an inability of neurons to communicate. If neurons cannot communicate, flaccid paralysis may be one of the results.

An autoimmune disease attacks the voltage-gated calcium channels in the nerve terminal. What is a likely symptom of this condition? A. Spastic paralysis (inability to relax the muscles) B. Flaccid paralysis (inability to contract the muscles) C. Inability to reuptake neurotransmitters once released D. Retrograde flow of action potentials

Correct Answer: B Explanation: First, consider the function of voltage-gated calcium channels. When the nerve terminal depolarizes, voltage-gated calcium channels open, allowing for influx of calcium. This influx of calcium triggers fusion of the synaptic vesicles containing neurotransmitters with the membrane of the neuron at the nerve terminal. This allows for exocytosis of the neurotransmitters into the synapse. If a disease blocked the influx of calcium, there would be no release of neurotransmitters. A lack of neurotransmitters means that the neuron cannot send signals. Thus, any symptoms resulting from this disease would be due to an inability of neurons to communicate. If neurons cannot communicate, flaccid paralysis may be one of the results.

Idiopathic pulmonary fibrosis (IPF) is a disease in which scar tissue forms in the alveolar walls, making the lung tissue significantly more stiff. Which of the following findings would likely be detected through spirometry in a patient with IPF? Decreased total lung capacity Decreased inspiratory reserve volume Increased residual volume A. I only B. II only C. I and II only D. I, II, and III

Correct Answer: B Explanation: In a patient with IPF, the increased stiffness of the lungs would likely decrease the volume of air the individual could inhale, which would decrease both the total lung capacity and inspiratory reserve volume. However, spirometry cannot measure the total lung capacity accurately because it cannot determine the residual volume—the volume of air left in the lungs when an individual has maximally exhaled. Because the residual volume makes up a portion of the total lung capacity (total lung capacity = vital capacity + residual volume), a spirometer cannot be used to determine the total lung capacity. Therefore, while Statement I is a true statement, it cannot appear in the answer choice. Finally, increased stiffness of the lungs would be expected to decrease the residual volume, not increase it; further, residual volume, as described above, cannot be measured with a spirometer.

Iodine deficiency may result in: A. galactorrhea. B. cretinism. C. gigantism. D. hyperthyroidism.

Correct Answer: B Explanation: Inflammation of the thyroid or iodine deficiency can cause hypothyroidism, in which the thyroid hormones are undersecreted or not secreted at all. Hypothyroidism in newborn infants causes cretinism, which is characterized by poor neurological and physical development (including mental retardation, short stature, and coarse facial features). While iodine deficiency can result in a swelling of the thyroid gland (called a goiter), which can also be seen in causes of hyperthyroidism, iodine deficiency does not cause hyperthyroidism, eliminating choice (D). Galactorrhea, choice (A), is associated with prolactin; gigantism, choice (C), is associated with growth hormone.

A drug is used that prevents the conversion of angiotensin I to angiotensin II. What is a likely effect of this drug? A. Increased sodium reabsorption B. Increased potassium reabsorption C. Increased blood pressure D. Increased blood pH

Correct Answer: B Explanation: Normally, angiotensin II causes secretion of aldosterone from the adrenal cortex. Aldosterone serves to increase reabsorption of sodium, while promoting excretion of potassium and hydrogen ions. Thus, blocking the release of aldosterone should result in decreased reabsorption of sodium, while decreasing excretion of potassium and hydrogen ions. This eliminates choices (A) and (D) and makes choice (B) the correct answer. In the absence of aldosterone, less water reabsorption will occur, eliminating choice (C).

A patient presents with muscle weakness, slow movement, and calcium deposits in his tissues. A blood test reveals very low calcium levels in the blood. Administration of which the following would be an appropriate treatment for the blood test findings? A. Calcitonin B. Parathyroid hormone C. Aldosterone D. Thymosin

Correct Answer: B Explanation: Regardless of the cause, the low levels of calcium in the blood require treatment. While other therapies are more frequently used to treat hypocalcemia (low blood calcium levels), such as calcium gluconate or calcium chloride, administration of parathyroid hormone would also raise blood calcium concentration. Calcitonin would be a poor choice in this case, as this hormone lowers blood calcium concentrations, eliminating choice (A). Aldosterone and thymosin play no role in calcium homeostasis, eliminating choices (C) and (D).

In which of the following segments of the nephron is sodium NOT actively transported out of the nephron? A. Proximal convoluted tubule B. Thin portion of the ascending limb of the loop of Henle C. Distal convoluted tubule D. Thick portion of the ascending limb of the loop of Henle

Correct Answer: B Explanation: Sodium is actively transported out of the nephron in the proximal and distal convoluted tubules, where the concentration of sodium outside of the nephron is higher than inside; thus, energy is required to transport the sodium molecules against their concentration gradient, eliminating choices (A) and (C). In the inner medulla, however, sodium and other ions (such as chloride) diffuse passively down their concentration gradients from the thin ascending limb of the loop of Henle, making choice (B) the correct answer. The thick ascending limb of the loop of Henle is thick because its cells contain many mitochondria—which produce the ATP needed for active transport of sodium and chloride out of the filtrate, eliminating choice (D).

Starch is hydrolyzed into maltose by enzymes from the: salivary glands. brush border. pancreas. A. I only B. I and III only C. II and III only D. I, II, and III

Correct Answer: B Explanation: Starch is hydrolyzed to maltose by two enzymes: salivary amylase (secreted by the salivary glands) in the mouth and pancreatic amylase (secreted by the pancreas) in the duodenum. Brush-border disaccharidases can further break down maltose, but do not break down starch.

Which of the following definitions is FALSE? A. Penetrance—the percentage of individuals in the population carrying the allele who actually express the phenotype associated with it B. Expressivity—the percentage of individuals in the population carrying the allele who do not express the phenotype associated with it C. Incomplete dominance—occurs when the phenotype of the heterozygote is an intermediate of the phenotypes of the homozygotes D. Codominance—occurs when multiple alleles exist for a given gene and more than one of them is dominant

Correct Answer: B Explanation: The definition given here for expressivity is a much better match for defining penetrance (or, really, one minus the penetrance). Expressivity refers to the variable manifestations of a given genotype as different phenotypes; the degree to which various phenotypes are expressed. All of the other definitions given are accurate.

A surgeon accidentally clips a dorsal root ganglion during a spinal surgery. What is a likely consequence of this error? Loss of motor function at that level Loss of sensation at that level Loss of cognitive function A. I only B. II only C. I and II only D. I, II, and III

Correct Answer: B Explanation: The dorsal root ganglion contains cell bodies of sensory neurons only. If a dorsal root ganglion is disrupted at a certain level, there will be a loss of sensation at that level.

Which of the following INCORRECTLY pairs a structure of the male reproductive system with a feature of the structure? A. Seminal vesicles—produce alkaline fructose-containing secretions B. Epididymis—surrounded by muscle to raise and lower the testes C. Vas deferens—tube connecting the epididymis to the ejaculatory duct D. Cowper's glands—produce a fluid to clear traces of urine in the urethra

Correct Answer: B Explanation: The epididymis is the site of sperm maturation at the posterior side of the testis. In the epididymis, sperm gain mobility and are stored until ejaculation. It is the vas deferens (ductus deferens) that is surrounded by muscle that raises and lowers the testis to maintain a constant temperature suitable for sperm production, not the epididymis.

What is the last point in the meiotic cycle in which the cell has a diploid number of chromosomes? A. During interphase B. During telophase I C. During interkinesis D. During telophase II

Correct Answer: B Explanation: The first meiotic division (reductional division) separates homologous chromosomes to opposite poles of the cell during anaphase I. Near the end of telophase I, cytokinesis occurs, resulting in two haploid (n) daughter cells. Thus, during interkinesis and anaphase II, the daughter cells are already haploid, eliminating choices (C) and (D). The cell is diploid during interphase, choice (A), but remains diploid up until the end of telophase I.

2. Which of the following are NOT involved in cell-mediated immunity? A. Memory cells B. Plasma cells C. Cytotoxic cells D. Suppressor cells

Correct Answer: B Explanation: The lymphocytes involved in cell-mediated immunity are the T-lymphocytes, or T-cells. There are four types of T-cells, each playing a different role in cell-mediated immunity: cytotoxic T-cells, helper T-cells, memory T-cells, and suppressor T-cells. Thus, from the answer choices, the only cells not involved in cell-mediated immunity are the plasma cells, which are differentiated immunoglobulin-secreting B-lymphocytes involved in humoral immunity.

Many medications have anticholinergic side effects, which block the activity of parasympathetic neurons throughout the body. Older individuals may be on many such medications simultaneously, exacerbating the side effects. Which of the following would NOT be expected in an individual taking medications with anticholinergic activity? A. Dry mouth B. Diarrhea C. Slow gastric emptying D. Decreased gastric acid production

Correct Answer: B Explanation: The parasympathetic nervous system has many roles in the digestive system. It promotes motility of the gut tube and secretion from glands. Therefore, blocking the parasympathetic nervous system would likely result in dry mouth (from reduced secretion of saliva), slow gastric emptying (from decreased peristalsis), and decreased gastric acid production (from reduced HCl secretion from the parietal cells in the gastric glands), eliminating choices (A), (C), and (D). Choice (B) is the correct answer because we would be more likely to expect constipation in such an individual, not diarrhea: slowed motility through the colon would lead to increased water reabsorption, making the feces too firm and causing constipation.

A two-week-old male infant is brought to the emergency room. His mother reports that he has been unable to keep any milk down; shortly after he nurses, he has sudden projectile vomiting. During exam, an olive-shaped mass can be felt in his upper abdomen. It is determined that there is a constriction in the digestive system that prevents food from reaching the small intestine from the stomach. Which structure is most likely the site of the problem? A. Cardiac sphincter B. Pyloric sphincter C. Ileocecal valve D. Internal anal sphincter

Correct Answer: B Explanation: The question is basically asking us to identify the structure that lies between the stomach and the small intestine. This is the pyloric sphincter; the presentation given in the question is a classic example of what is called pyloric stenosis, in which the pyloric sphincter is thickened and cannot relax to permit chyme through. The cardiac sphincter, choice (A), lies between the esophagus and the stomach. The ileocecal valve, choice (C), lies between the ileum of the small intestine and the cecum of the large intestine. The internal anal sphincter, choice (D), lies at the end of the rectum.

A scientist discovers a new hormone that is relatively large in size and triggers the conversion of ATP to cAMP. Which of the following best describes the type of hormone that was discovered? A. Amino acid-derivative hormone B. Peptide hormone C. Steroid hormone D. Tropic hormone

Correct Answer: B Explanation: The question stem indicates that the newly discovered hormone functions as a first messenger, stimulating the conversion of ATP to cAMP; cAMP functions as a second messenger, triggering a signaling cascade in the cell. Hormones that act via secondary messengers and are relatively large in size (short peptides or complex polypeptides) are peptide hormones. This hormone could be a tropic hormone, but it is also entirely possible for it to be a direct hormone; thus, choice (D) can be eliminated.

As the climate became colder during the Ice Age, a particular species of mammal evolved a thicker layer of fur. What kind of selection occurred in this population? A. Stabilizing selection B. Directional selection C. Disruptive selection D. Speciation selection

Correct Answer: B Explanation: The situation described in the question stem is an example of directional selection. In directional selection, the phenotypic norm of a particular species shifts toward an extreme to adapt to a selective pressure, such as an increasingly colder environment. Only those individuals with a thicker layer of fur were able to survive during the Ice Age, thus shifting the phenotypic norm.

What type of immunity is likely to be affected by removal of the spleen? A. Cytotoxic immunity B. Humoral immunity C. Innate immunity D. Passive immunity

Correct Answer: B Explanation: The spleen is a location where B-cells mature and proliferate. Therefore, removal of the spleen is likely to result in a reduction of humoral immunity. In fact, many people receive vaccinations prior to the removal of the spleen in order to bolster their immunity.

In a particular Hardy-Weinberg population, there are only two eye colors: brown and blue. Of the population, 36% have blue eyes, the recessive trait. What percentage of the population is heterozygous? A. 24% B. 48% C. 60% D. 64%

Correct Answer: B Explanation: Using the information given in the question stem, we can determine that the percentage of the population with blue eyes (genotype bb) = 36% = 0.36 = q2; therefore, q = 0.6. Because this is a Hardy-Weinberg population, we can assume that p + q = 1, so p = 1 - 0.6 = 0.4. The frequency of heterozygous brown eyes is therefore 2pq = 2 × 0.4 × 0.6 = 0.48 = 48%. *If one letter (R) is given (frequency of R), use first equation. If the phenotype is given (RR, Rr, or rr)(__% have recessive trait), use second equation

Hemoglobin's affinity for O2: A. increases in exercising muscle tissue. B. decreases as blood PaCO2 decreases. C. decreases as blood pH decreases. D. is higher in maternal blood than in fetal blood.

Correct Answer: C Explanation: According to the Bohr effect, decreasing the pH in the blood decreases hemoglobin's affinity for O2. This makes choice (C) the correct answer. The affinity is generally lowered in exercising muscle to facilitate unloading of oxygen to tissues, eliminating choice (A). A decrease in the PaCO2 would cause a decrease in [H+] or increased pH—which increases hemoglobin's affinity for O2, eliminating choice (B). Finally, choice (D) is incorrect because hemoglobin's affinity for O2 is higher in fetal blood than in adult blood.

Which of the following shows the correct order of early developmental milestones during embryogenesis? A. Blastula → gastrula → morula B. Morula → gastrula → blastula C. Morula → blastula → gastrula D. Gastrula → blastula → morula

Correct Answer: C Explanation: After the first cell divisions occur, the embryo consists of a solid ball of cells known as a morula. Then a hollow center forms, creating the blastula. Finally, as the cells begin to differentiate into the three germ layers, the embryo is considered a gastrula. Choice (C) is therefore the correct answer.

A young woman presents to the emergency room with a broken hip. She denies any recent history of trauma to the joint. Blood tests reveal a calcium concentration of 11.5 mg/dL (normal: 8.4-10.2). Which tissue is likely responsible for these findings? A. Thyroid B. Cartilage C. Parathyroid D. Smooth muscle

Correct Answer: C Explanation: An unprovoked fracture of the hip is not a normal finding in a young woman. Given that she has a high calcium level, it is likely that she has an increased level of bone resorption that is causing her bones to be more fragile. Parathyroid hormone causes calcium release from bones. If this woman had an overactive parathyroid gland—or even cancer in this gland—then it is likely that calcium could still be resorbed from her bones even though her blood calcium levels are already high.

Resistance to antibiotics is a well-recognized medical problem. Which mechanism(s) can account for a bacterium's ability to increase its genetic variability and thus adapt itself to different antibiotics? Binary fission Conjugation Transduction A. I and II only B. I and III only C. II and III only D. I, II, and III

Correct Answer: C Explanation: Bacterial cells reproduce by binary fission, an asexual process in which the progeny is identical to the parent. Therefore, Statement I does not increase genetic variability. Conjugation can be described as a sexual mating in bacteria; it is the transfer of genetic material between two bacteria that are temporarily joined. Transduction occurs when fragments of the bacterial chromosome accidentally become packaged into viral progeny produced during a viral infection and can subsequently be introduced into another bacterium by the viral vector. Therefore, both Statements II and III increase bacterial genetic variability. *3 main ways bacteria increase genetic variability: Transformation, conjugation, and transduction

Which of the following statements regarding fetal circulation is FALSE? A. In the umbilical cord, there are more arteries than veins. B. The foramen ovale is the only shunt that connects two chambers of the heart. C. Blood flow in the ductus arteriosus is from the aorta to the pulmonary artery. D. The ductus venosus is the only shunt that bypasses the liver.

Correct Answer: C Explanation: Blood flow in the ductus arteriosus is from the pulmonary artery to the aorta. The direction of flow is determined by the pressure differential between the right side of the heart (and pulmonary circulation) vs. the left side of the heart (and systemic circulation). Unlike in adults, the right side of the heart is at a higher pressure during prenatal life than the left side, so blood will shunt from the pulmonary circulation to the systemic circulation through both the foramen ovale and ductus arteriosus.

Oxytocin and antidiuretic hormone are: A. peptide hormones produced and released by the pituitary. B. steroid hormones produced and released by the pituitary. C. peptide hormones produced by the hypothalamus and released by the pituitary. D. steroid hormones produced by the hypothalamus and released by the pituitary.

Correct Answer: C Explanation: Both oxytocin and vasopressin (another name for antidiuretic hormone) end with the suffix -in; this should hint that they are peptide or amino acid-derivative hormones. These two hormones are both synthesized by the hypothalamus, but released by the posterior pituitary. Remember that the posterior pituitary does not actually synthesize any hormones itself; rather, it contains the axons of cells originating in the hypothalamus and is the site of release for these hormones.

Which of the following is involved in the body's primary blood-buffering mechanism? A. Fluid intake B. Absorption of nutrients in the gastrointestinal system C. Carbon dioxide produced from metabolism D. Hormones released by the kidney

Correct Answer: C Explanation: Carbon dioxide is a byproduct of metabolism in cells that later combines with water to form bicarbonate in a reaction catalyzed by carbonic anhydrase. This system is blood plasma's most important buffer system. Food and fluid absorption are not significant sources of buffering, eliminating choices (A) and (B). While the kidney can be involved in acid-base balance, it is through its filtration, secretion, and reabsorption mechanisms, not hormones, eliminating choice (D). Note: Gastrointestinal cells need to be replaced every couple of days so they would not go into G0 phase because they are actively replicating.

Which of the following was NOT a belief of Darwin's? A. Evolution of species occurs gradually and evenly over time. B. There is a struggle for survival among organisms. C. Genetic recombination and mutation are the driving forces of evolution. D. Those individuals with fitter variations will survive and reproduce.

Correct Answer: C Explanation: Darwin's main argument was that natural selection is the driving force for evolution. Darwin did not consider the role of genetic mutation and recombination, which were unknown at the time. These aspects were added to Darwin's theory as part of the modern synthesis model, which did not occur until significantly after Darwin's time.

Excessive levels of dopamine in the brain are associated with psychosis. Accordingly, many antipsychotic medications block dopamine receptors. Which of the following effects may be seen in an individual taking antipsychotics? A. Increased secretion of growth hormone B. Decreased secretion of growth hormone C. Increased secretion of prolactin D. Decreased secretion of prolactin

Correct Answer: C Explanation: Dopamine is used in a number of neurological systems; most relevant to the endocrine system is the fact that dopamine secretion prevents prolactin release. Thus, an individual taking medications that block dopamine receptors would lose this inhibition on prolactin release and have elevated prolactin levels.

A child is born with webbed toes (or an imperforate anus, in which anal canal fails to form correctly). This pathology is most likely accounted for by a failure of: A. cell differentiation. B. cell determination. C. apoptosis. D. neurulation.

Correct Answer: C Explanation: During development, programmed cell death occurs in multiple locations in order to ensure development of the correct adult structures. One of the places in which this occurs is between fingers and toes, another is the digestive tract (where central lumen is formed). If apoptosis does not occur correctly in these locations, then an infant may be born with webbed fingers or toes. Failure of determination or differentiation would likely result in an absence of digits, eliminating choices (A) and (B). Failure of neurulation would lead to an absence of a nervous system and would not be compatible with life, eliminating choice (D).

Which of the following is likely to be found in maternal blood during pregnancy? A. Immunoglobulins produced by the fetus B. Fetal hemoglobin released from fetal red blood cells C. Progesterone produced by placental cells D. Carbon dioxide exhaled from fetal lungs

Correct Answer: C Explanation: During pregnancy, the placenta produces estrogen and progesterone to maintain the endometrium. These hormones are necessary for proper gestation of the fetus and should be measurable in maternal blood because they act on maternal organs. Prior to birth, the fetus is immunologically na?ve and does not yet produce immunoglobulins, eliminating choice (A). It is worth noting, though, that maternal immunoglobulins cross the placenta to enter fetal blood. Fetal hemoglobin is a large protein and, thus, cannot easily cross the placenta. Further, red blood cells are much too large to cross the barrier themselves, eliminating choice (B). Carbon dioxide from fetal metabolism can be found in maternal blood, but the lungs are nonfunctional prior to birth as the fetus is suspended in amniotic fluid. Carbon dioxide is transferred across the placenta directly from the fetal bloodstream, eliminating choice (D).

Which of the following would NOT likely lead to elevated levels of bilirubin in the blood? A. Cholangiocarcinoma, a cancer of the bile ducts that can ultimately lead to full occlusion of the duct lumen B. Autoimmune hemolytic anemia, a disease in which the red blood cells are attacked by antibodies and are lysed C. Ménétrier's disease, in which rugae thicken and overlying glands lose secretory ability D. Acetaminophen (Tylenol) overdose, in which the accumulation of toxic metabolites can cause rapid liver failure

Correct Answer: C Explanation: Elevated bilirubin implies a blockage to bile flow, increased production of bilirubin (from massive hemoglobin release), or an inability of the liver to produce bile. If the bile duct were occluded, as in choice (A), then bile would not be able to flow into the digestive tract and would build up, increasing bilirubin levels in the blood. If many red blood cells were lysed, as in choice (B), then bilirubin levels would rise in accordance with the increased hemoglobin release. If liver failure occurred, as in choice (D), then it would be unable to produce bile, and bilirubin would again build up. Choice (C) refers to a pathology in the stomach—the key word given here is rugae, which are the folds in the stomach wall. Lack of gastric function would have no effect on bilirubin levels, making this the correct choice.

Certain ovarian tumors called granulosa cell tumors are known to produce excessive levels of estrogen. A physician who diagnoses a granulosa cell tumor should look for a secondary cancer in which of the following parts of the reproductive tract? A. Fallopian tube B. Cervix C. Endometrium D. Vagina

Correct Answer: C Explanation: Estrogen is known to cause growth of the endometrial lining during the follicular phase of the menstrual cycle, and its levels stay high during the luteal phase to promote vascularization and glandularization of this tissue. Excessive levels of estrogen may provide a strong enough signal for cell growth to promote tumor formation, or even cancer. The other tissues listed in this question require estrogen for development, but are not strongly associated with rapid tissue growth due to estrogen.

What is the response of the immune system to downregulation of MHC molecules on somatic cells? A. B-cells are activated and antibodies are released. B. T-cells are activated, resulting in a cytotoxic response. C. Natural killer cells induce apoptosis of affected cells. D. Macrophages engulf the pathogen and display its antigens.

Correct Answer: C Explanation: Healthy cells exhibit MHC class I molecules. Natural killer cells monitor the expression of MHC molecules on the surface of cells. Viral infection and cancer often cause a reduction in the expression of MHC class I molecules on the cell surface. Natural killer cells detect this lack of MHC and induce apoptosis in the affected cells. Natural killer cells have evolved the ability to monitor cells for the absence of MHC

Due to kidney disease, a person is losing albumin into the urine. What effect is this likely to have within the capillaries? A. Increased blood pressure B. Increased hydrostatic pressure C. Decreased oncotic pressure D. Decreased hydrostatic pressure

Correct Answer: C Explanation: In circulation, plasma proteins play an important role in generating osmotic (oncotic) pressure. This allows water that is displaced at the arterial end of a capillary bed by hydrostatic pressure to be reabsorbed at the venule end. Loss of these plasma proteins would cause a decrease in the plasma osmotic (oncotic) pressure. ONCOTIC PRESSURE OF BLOOD DRAWS WATER BACK INTO VESSELS AT VENULE END ONCE HYDROSTATIC PRESSURE HAS DECREASED

Lymphoma is cancer of the cells of the lymphoid lineage. These cells often reside within lymph nodes. What type of cell is NOT likely to cause a lymphoma? A. CD8+ T-cells B. B-cells C. Macrophages D. Th1 cells

Correct Answer: C Explanation: Lymphocytes arise from the lymphoid lineage, which includes B-cells and T-cells. Thus, all types of B- and T-cells are capable of causing lymphoma. Macrophages, however, are not lymphocytes and are not likely to cause lymphoma.

An individual who is phenotypically female is found to have only one copy of a disease-carrying recessive allele on the X chromosome, yet she demonstrates all of the classic symptoms of the disease. Geneticists determine she has a genotype that likely arose from nondisjunction in one of her parents. What is the likely genotype of this individual? A. 46,XX (46 chromosomes, with XX for sex chromosomes) B. 46,XY C. 45,X D. 47,XXY

Correct Answer: C Explanation: Nondisjunction refers to the incorrect segregation of homologous chromosomes during anaphase I, or of sister chromatids during anaphase II. In either case, one daughter cell ends up with two copies of related genetic material, while the other receives zero. Immediately, this should eliminate choices (A) and (B), which show a normal complement of chromosomes (46). An individual who has only one recessive disease-carrying allele, and yet still expresses the disease, likely does not have a dominant allele for the given trait. This is seen in males, who are hemizygous for many X-linked genes, and can also be seen in women with Turner syndrome (45,X), who have only one X chromosome.

Which of the following hormones is NOT derived from cholesterol? A. Aldosterone B. Estrogen C. Oxytocin D. Progesterone

Correct Answer: C Explanation: Steroid hormones are derived from cholesterol. Many of their names end with -one, -ol, or -oid, and they include the hormones of the adrenal cortex and gonads. Aldosterone is a mineralocorticoid from the adrenal cortex, while estrogen and progesterone are sex hormones from the ovaries, eliminating choices (A), (B), and (D). Oxytocin is a peptide hormone and is thus not derived from cholesterol. Peptide and amino acid-derivative hormones often end with -in or -ine.

A patient has a very high TSH level. Which of the following would NOT cause a high TSH level? A. Autoimmune destruction of thyroid cells that produce T3 and T4. B. A tumor in the hypothalamus that secretes high levels of TRH. C. High levels of T4 from thyroid replacement medications. D. Cancerous growth of parafollicular cells in the thyroid, destroying other cell types in the organ.

Correct Answer: C Explanation: The hypothalamic-pituitary-thyroid axis includes the secretion of thyroid-releasing hormone (TRH) from the hypothalamus stimulating the secretion of thyroid-stimulating hormone (TSH) from the anterior pituitary, which stimulates the secretion of triiodothyronine (T3) and thyroxine (T4) from the thyroid. Overproduction of TRH would promote overproduction of TSH, eliminating choice (B). Destruction of the follicular cells that produce T3 and T4 would remove negative feedback, allowing TSH levels to rise, eliminating choices (A) and (D). High levels of T4 would cause too much negative feedback and lower TSH levels, making choice (C) the correct answer. TSH stimulates synthesis and release of thyroid hormones. The process of T3/T4 production involves iodine uptake and thyroglobulin synthesis. TSH stimulates hypertrophy (increase in cell size) /hyperplasia (increase in cell numbers) of thyroid tissues.

Which of the following developmental stages has the greatest nuclear-to-cytoplasmic ratio? A. Eight-cell embryo B. Morula C. Blastula D. Zygote

Correct Answer: C Explanation: The question is asking us to determine the developmental stage with the greatest nuclear-to-cytoplasmic material ratio. During the series of rapid mitotic divisions known as cleavage, the number of cells increases dramatically, but the overall volume of the embryo does not change. In fact, the volume of the embryo does not significantly increase until after the blastula has already implanted. As such, a high ratio of nuclear to cytoplasmic material will be found at the stage with the greatest amount of cells during early development. From the given choices, the stage with the greatest number of cells is the blastula. This is further supported by the fact that the cells are displaced from the center by the blastocoel.

An X-ray of the right femur in a child shows that it is shorter than the opposite femur, and below the average length for a child of this age. Which region of the bone is most likely to have an abnormality? A. Diaphysis B. Metaphysis C. Epiphysis D. Periosteum

Correct Answer: C Explanation: This question is essentially asking where longitudinal growth occurs in bones. The most likely site of abnormalities in this child's femur is the epiphyseal plate, a disk of cartilaginous cells at the internal border of the epiphysis, because the epiphyseal plate is the site of longitudinal growth. Damage to the epiphysis (with or without metaphysis involvement) can imply damage to the epiphyseal plate.

Upon ovulation, the oocyte is released into the: A. fallopian tube. B. follicle. C. abdominal cavity. D. uterus.

Correct Answer: C Explanation: This subtle point about ovulation eludes most students and remains hard to believe until the organs are examined in anatomy class in medical school. The ruptured ovarian follicle releases an oocyte into the abdominal cavity, close to the entrance of the fallopian tube. With the aid of beating cilia, the oocyte is drawn into the fallopian tube, through which it travels until it reaches the uterus. If it is fertilized in the fallopian tube, it will implant in the uterine wall. If fertilization does not occur, it will be expelled along with the uterine lining during menstruation.

Total lung capacity is equal to the vital capacity plus the: A. tidal volume. B. expiratory reserve volume. C. residual volume. D. inspiratory reserve volume.

Correct Answer: C Explanation: Total lung capacity is equal to the vital capacity (the maximum volume of air that can be forcibly inhaled and exhaled from the lungs) plus the residual volume (the air that always remains in the lungs, preventing the alveoli from collapsing).

Destruction of all β-cells in the pancreas would cause: A. glucagon secretion to stop and a decrease in blood glucose concentration. B. glucagon secretion to stop and an increase in blood glucose concentration. C. insulin secretion to stop and an increase in blood glucose concentration. D. insulin secretion to stop and a decrease in blood glucose concentration.

Correct Answer: C Explanation: β-cells are responsible for insulin production. The function of insulin is to lower blood glucose levels by promoting the influx of glucose into cells and by stimulating anabolic processes, such as glycogenesis or fat and protein synthesis. Thus, destruction of the β-cells would result in a cessation of insulin production, which would lead to hyperglycemia, or high blood glucose concentrations.

In a nonevolving population, there are two alleles, R and r, which code for the same trait. The frequency of R is 30 percent. What are the frequencies of all the possible genotypes? A. 49% RR, 42% Rr, 9% rr B. 30% RR, 21% Rr, 49% rr C. 0.09% RR, 0.42% Rr, 0.49% rr D. 9% RR, 42% Rr, 49% rr

Correct Answer: D Explanation: We are told that the frequency of R equals 30%, and as such, p = 0.30. The frequency of the recessive gene r = 100% - 30% = 70%; thus, q = 0.70. The frequency of the genotypes, according to the Hardy-Weinberg equilibrium, are given by p2 = RR, 2pq = Rr, and q2 = rr. Therefore, the frequency of the genotypes are (0.3)2 = 0.09 = 9% RR, 2 × 0.3 × 0.7 = 0.42 = 42% Rr, and (0.7)2 = 0.49 = 49% rr.

Which of the following is a correct sequence of passageways through which air travels during inhalation? A. Pharynx → trachea → bronchioles → bronchi → alveoli B. Pharynx → trachea → larynx → bronchi → alveoli C. Larynx → pharynx → trachea → bronchi → alveoli D. Pharynx → larynx → trachea → bronchi → alveoli

Correct Answer: D Explanation: Air enters the respiratory tract through the external nares (nostrils) and travels through the nasal cavities. It then passes through the pharynx and into the larynx. Ingested food also passes through the pharynx on its way to the esophagus; to ensure that food does not accidentally enter the larynx, the epiglottis covers the larynx during swallowing. After the larynx, air goes to the trachea, which eventually divides into two bronchi, one for each lung. The bronchi branch into smaller bronchioles, which terminate in clusters of alveoli. From the given sequences, only choice (D) correctly describes the sequence of the passages through which air travels.

Which of the following would NOT be seen during pregnancy? A. High levels of hCG in the first trimester B. High levels of progesterone throughout the pregnancy C. Low levels of FSH in the first trimester D. High levels of GnRH throughout the pregnancy What is the corpus luteum?

Correct Answer: D Explanation: During the first trimester of pregnancy, the corpus luteum is preserved by human chorionic gonadotropin (hCG); hence, progesterone secretion by the corpus luteum is maintained during the first trimester. This eliminates choice (A). During the second trimester, hCG levels decline, but progesterone levels rise because the hormone is now secreted by the placenta itself, eliminating choice (B). High levels of progesterone and estrogen inhibit GnRH secretion, thus preventing FSH and LH secretion and the onset of a new menstrual cycle. The remnant of the ovarian follicle, which after ovulation continues to secrete progesterone. Its degradation leads to menstruation -It also maintains the uterine lining during pregnancy

The world record for the longest-held breath is 22 minutes and 0 seconds. If a sample were taken from this individual during the last minute of breath-holding, which of the following might be observed? A. Increased hemoglobin affinity for oxygen B. Decreased PaCO2 C. Increased hematocrit D. Decreased pH

Correct Answer: D Explanation: Holding one's breath for a prolonged period would result in a drop of oxygenation and an increase in PaCO2. The increased carbon dioxide would associate with water to form carbonic acid, which would dissociate into a proton and bicarbonate anion. Further, the low oxygen saturation would eventually lead to anaerobic metabolism in some tissues, causing an increase in lactic acid. These would all lead to a decreased pH.

Which of the following INCORRECTLY pairs a digestive hormone with its function? A. Trypsin—hydrolyzes specific peptide bonds B. Lactase—hydrolyzes lactose to glucose and galactose C. Pancreatic amylase—hydrolyzes starch to maltose D. Lipase—emulsifies fats

Correct Answer: D Explanation: Lipase is involved in the digestion of fats, but its function is not to emulsify fats—this is the job of bile. Rather, lipase chemically digests fats in the duodenum, allowing them to be brought into duodenal cells and packaged into chylomicrons. Fat emulsification is the process of increasing the surface area of fats in the small intestine by grouping them into small clusters. This is the responsibility of bile, a liquid created by the liver and stored in the gallbladder. Actual digestion of the fats is then accomplished by lipase, an enzyme from the pancreas. Digested lipids are then transported into the lacteals of the microvilli for transportation through the lymphatic system before entering the blood.

Which of the following statements regarding the periosteum is INCORRECT? A. The periosteum serves as a site of attachment of bone to muscle. B. Cells of the periosteum may differentiate into osteoblasts. C. The periosteum is a fibrous sheath that surrounds long bones. D. The periosteum secretes fluid into the joint cavity.

Correct Answer: D Explanation: The periosteum, a fibrous sheath that surrounds long bones, is the site of attachment to muscle tissue. Some periosteum cells are capable of differentiating into bone-forming cells called osteoblasts. This eliminates choices (A), (B), and (C). It is the synovium that secretes fluid into the joint cavity (joint space), not the periosteum, making choice (D) the correct answer.

A genotypically female infant is born with ambiguous genitalia. Soon after birth, she suffers from hyponatremia, or low blood concentrations of sodium. Which endocrine organ is most likely to be affected? A. Hypothalamus B. Pituitary C. Kidneys D. Adrenal cortex

Correct Answer: D Explanation: The question stem states that an infant who is genotypically female is born with ambiguous genitalia, meaning that the genitalia do not appear to be specifically female or specifically male. In a genotypic female, this indicates that she was exposed to androgens during the fetal period. In addition, she is also losing sodium, causing hyponatremia. This indicates that she has two defects. First, she has excess androgens. Second, she is lacking in aldosterone, a hormone required for proper reuptake of sodium in the kidneys. Both of these hormones are synthesized in the adrenal cortex, making choice (D) the correct answer. Note that neither of these hormones is regulated by the hypothalamic-pituitary-adrenal axis, eliminating choices (A) and (B).

Which of the following does NOT likely contribute to genetic variability? A. Random fertilization of an egg by a sperm B. Random segregation of homologous chromosomes C. Crossing over between homologous chromosomes during meiosis D. Replication of the DNA during S stage

Correct Answer: D Explanation: The safest way to answer this question correctly is to go through each answer choice and eliminate the ones that contribute to genetic variability. The random fertilization of a sperm and an egg, the random segregation of homologous chromosomes during anaphase I, and crossing over between homologous chromosomes during prophase I all contribute to genetic variability during sexual reproduction because they result in novel combinations of genetic material, eliminating choices (A), (B), and (C). S stage, choice (D), should specifically not cause increased genetic variability; the DNA should be copied precisely, without error, meaning that both strands of DNA should be identical.

Which of the following statements concerning the somatic division of the peripheral nervous system is INCORRECT? A. Its pathways innervate skeletal muscle. B. Its pathways are usually voluntary. C. Some of its pathways are referred to as reflex arcs. D. Its pathways always involve more than two neurons.

Correct Answer: D Explanation: The somatic division of the peripheral nervous system innervates skeletal muscles and is responsible for voluntary movement. Some of the pathways in this part of the nervous system are reflex arcs, which are reflexive responses to certain stimuli that involve only a sensory and a motor neuron. These neurons synapse in the spinal cord and do not require signaling from the brain. The pathways of the somatic division can involve two, three, or more neurons, depending on the type of signal. The correct answer therefore is choice (D).

Which of the following is FALSE regarding aldosterone regulation? A. Renin converts the plasma protein angiotensinogen to angiotensin I. B. Angiotensin II stimulates the adrenal cortex to secrete aldosterone. C. Angiotensin I is converted to angiotensin II by angiotensin-converting enzyme. D. A decrease in blood oxygen concentrations stimulates renin production.

Correct Answer: D Explanation: The stimulus for renin production is low blood pressure, which causes the juxtaglomerular cells of the kidney to produce renin, an enzyme that converts the plasma protein angiotensinogen to angiotensin I. Angiotensin I is then converted to angiotensin II by an enzyme in the lungs; angiotensin II ultimately stimulates the adrenal cortex to secrete aldosterone. *secretion of aldosterone, decreases drive to release renin = negative feedback mechanism) Aldosterone helps to restore blood volume by increasing sodium reabsorption in the kidney, leading to an increase in water reabsorption. This removes the initial stimulus for renin production. Thus, choices (A), (B), and (C) correctly describe the renin-angiotensin-aldosterone system, while choice (D) describes the stimulus for erythropoietin secretion. -Decreased BP causes juxtaglomerular cells of kidney to secrete renin

A researcher wishes to incorporate a radiolabeled deoxyadenine into the genome of one of the two daughter cells that would arise as a result of mitosis. What is the latest stage of cellular development during which the radiolabeled deoxyadenine could be added to achieve this result? A. G1 B. G2 C. M D. S

Correct Answer: D Explanation: To ensure that the labeled deoxyadenine will be incorporated into the DNA of one of the daughter cells, we have to insert the nucleotide before DNA replication has been completed. Because replication occurs during S stage, we could introduce the deoxyadenine during G1 or S stage. Because G1 precedes S, the latest point the deoxyadenine could be added is the S stage.

When the pH of the blood is high, which substance is likely to be excreted in larger quantities in the urine? A. Urea B. Ammonia C. Hydrogen ions D. Bicarbonate ions

Correct Answer: D Explanation: When the pH of the blood is high, this indicates that the blood is alkalemic. In order to correct the pH of the blood, the kidney will increase the excretion of a base, namely bicarbonate. Excretion of urea would have little effect on the pH, eliminating choice (A). While ammonia is a base, it is quite toxic and is generally converted to urea before excretion, eliminating choice (B). Excretion of hydrogen ions would result in exacerbation of the alkalemia, eliminating choice (C).

__________ is understood to control long term (slow) stress response, while __________ are understood to control short-term (fast) stress response

Cortisol is understood to control long term (slow) stress response, while are understood to control short-term (fast) stress response Cortisol increase synthesis/release of catecholamines

Cancer cells are cells in which mitosis occurs continuously, without regard to quality or quantity of the cells produced. For this reason, most chemotherapies attack rapidly dividing cells. At which point(s) in the cell cycle could chemotherapy effectively prevent cancer cell division? S stage Prophase Metaphase A. I only B. I and II only C. II and III only D. I, II, and III

D Explanation: The question is asking us to determine at which points in the cell cycle we can prevent or at least lower the number of cells undergoing mitosis. One idea would be to prevent DNA synthesis during the S stage of the cell cycle. Without the DNA being replicated, two viable daughter cells could not be formed. Another idea would be preventing the mitotic cycle from occurring altogether in prophase by preventing spindle apparatus formation, preventing the nuclear membrane from dissolving, or other processes during this phase. Similarly, a treatment that would act on cells in the metaphase stage of the cell cycle would also interfere with the mitotic cycle. Therefore, any of the three solutions presented would be a viable option.

Which of the following associations correctly pairs a stage of respiration with the muscle actions occurring during that stage? A. Inhalation—diaphragm relaxes B. Inhalation—internal intercostal muscles contract C. Exhalation—diaphragm contracts D. Exhalation—external intercostal muscles relax

D The muscles involved in ventilation are the diaphragm, which separates the thoracic cavity from the abdominal cavity, and the intercostal muscles between the ribs. During inhalation, the diaphragm contracts and flattens, while the external intercostal muscles contract, pulling the rib cage up and out. These actions cause an overall increase in the volume of the thoracic cavity. During exhalation, both the diaphragm and the external intercostals relax, causing a decrease in the volume of the thoracic cavity because of recoil of these tissues. In forced exhalation, the internal intercostals and abdominal muscles may contract to force out air. Thus, the only correct association from the given answers is choice (D).

Cancer

Damaged cells undergo mitosis Usually mutation of gene that produces p53 (TP53) Cell divides continuously, creating tumors If cell produces things like proteases that can digest basement membranes/factors that encourage blood vessel formation, damaged cells can reach other tissues Distant spread = Metastasis

In severe chronic obstructive pulmonary disease (COPD) the lungs have lost the ability to adequately ventilate CO2, resulting in chronically elevated CO2 levels. The neurons in the brainstem eventually stop responding to increased CO2 as a drive for breathing. What stimulates the respiratory drive in these patients?

Decreased levels of O2 in the blood Breathing requires input from our nervous control center. Ventilation is primarily regulated by a collection of neurons in the medulla oblongata called the ventilation center that fire rhythmically to cause regular contraction of respiratory muscles. These neurons contain chemoreceptors that are primarily sensitive to carbon dioxide concentration. As the partial pressure of carbon dioxide in the blood rises (hypercarbia or hypercapnia), the respiratory rate will increase so that more carbon dioxide is exhaled, and carbon dioxide levels in the blood will fall. These cells also respond to changes in oxygen concentration, although this tends to have significance only during periods of significant hypoxia (low oxygen concentration in the blood).Loss of the hypercapnic (high CO2) drive is a common occurrence in late stage COPD. Other than high CO2, the chemoreceptors of the brain are only responsive to low oxygen concentration, known as the hypoxic drive (C).

Deoxygenated blood with high CO2 concentration is brought to lungs via _______________. Oxygenated blood with low CO2 leaves lungs via ________________.

Deoxygenated blood (from right) with high CO2 concentration is brought to lungs via pulmonary arteries *Arteries carry blood Away from the heart *Heart attack (MI) = lack of blood through coronary arteries (decreased oxygen to heart) Oxygenated blood with low CO2 leaves lungs via pulmonary veins -Oxygenated blood returns to left atrium of heart via pulmonary veins (then to left ventricle then to rest of body)

Muscle Contraction: Initiation (followed by shortening of sarcomere and relaxation)

Depends on ATP and calcium Initiation Contraction starts at neuromuscular junction (NS communicates with muscle cells via motor (efferent) neurons) -Travels down neuron to nerve terminal/motor end plate (synaptic bouton), where acetylcholine released into synapse -Each nerve terminal controls a group of myocytes (together with nerve terminal = motor unit) -ACh binds to receptors on sarcelomma = depolarization -> triggers action potential, spreads down sarcolemma to T tubules to muscle tissues to SR, releasing Ca2+ -Calcium ions bind to subunit in troponin -> triggers confirmation change in tropomyosin (to which troponin is bound) -Exposes myosin binding sites on actin thin filament *Calcium binds to troponin, leads to change in tropomyosin, exposes myosin binding sites of actin

Gram positive and Gram Negative Bacteria cell walls Based on structure and genome, which of the following most closely resembles mitochondria? Gram-negative bacteria Gram-positive bacteria Retroviruses Peroxisomes

Determined by crystal violet/safranin (pink) stain Gram Positive = violet Gram negative = pink Gram positive cell walls consist of a thick layer of peptidoglycan (polymeric substance made from amino acids and sugars) and lipoteichoic acid -Penicillina nd antibiotics with similar function can more easily reach and weaken the peptidoglycan layer of gram positive bacteria Gram Negative: Very thin, contain less peptidoglycan separated rom membrane by periplasmic space, Also have outer membrane containing phospholipids and lipopolysaccharides (triggers inflammation response in humans) Based on structure and genome, which of the following most closely resembles mitochondria? Gram-negative bacteria Mitochondria have a double membrane structure and a double-stranded circular DNA genome, as do gram-negative bacteria. Gram-positive bacteria have a single membrane while mitochondria have a double membrane. Retroviruses have an RNA genome while mitochondria have a circular DNA genome. Peroxisomes do not have a genome and only have a single membrane. Mitochondria are believed to have stemmed from gram-negative bacteria. They share many similar characteristics, including double membranes, ribosome structures, and a genetic code involving circular DNA.

Twins

Dizygotic: Two different eggs released during one ovulatory cycle fertilized by two different sperm -No more genetically similar than siblings Monozygotic: Identical Single zygote splits in two Identical genetic material Incomplete division = conjoined twins Monochorionic/monoamniotic share amnion and chorion Monochorionic/diamniotic twins each have their own amnion, share chorion More shared structures = more risks

Endoplasmic Reticulum

Double membrane contiguous with nuclear envelope folded into complex structures with central lumen Rough ER: Studded with ribosomes, translation of proteins destined for secretion directly into lumen Smooth ER: Lipid synthesis/detoxification of drugs and poisons/ transports proteins from RER to Golgi (Proteins from the rough ER can cross into the smooth ER, where they are secreted into cytoplasmic vesicles and transported to the Golgi apparatus)

The Bladder

Elastic, hollow, muscular organ that provides temporary storage for urine. Detrusor Muscle: Muscular lining, parasympathetic NS causes contraction, but in order to leave body, urine must pass through internal urethral sphincter (smooth muscle/involuntary control/ normally contracted) and external urethral sphincter (skeletal muscle/under voluntary control) -Micturition reflex: Stretch receptors say when bladder full, parasympathetic signaling, detrusor muscle contracts, internal relaxes (up to person when they want to relax external)

Retrovirus

Enveloped, single stranded RNA viruses Usually contains two identical RNA molecules Carry enzyme known as reverse transcriptase that synthesizes DNA from single stranded RNA DNA then integrates into host genome where it is transcribed as if it were host DNA (intergrade inserts copy into host DNA) Allows the cell to be infected indefinitely, only way to remove virus is to kill cell HIV (must bind to CD4 and CCR5 proteins on cell surface)(Enveloped virus that fuses with membrane and enters cell intact)

Pancreas

Exocrine and endocrine functions Glucose homeostasis Exocrine tissues (makes up bulk of pancreas): -Secretes enzymes directly into ducts -Made up of acinar cells (produce bicarbonate rich alkaline pancreatic juice) -Pancreatic amylase: Breaks polysaccharides into smaller disaccharides (responsible for carb digestion) -Pancreatic peptidases (Trypsinogen, chymotrypsin and carboxypeptidases A and B) released in zymogen form, digest proteins when activated *Enteropeptidase, produced by duodenum converts Trypsinogen to trypsin, which activates other zymogens and procarboxypeptidases A and B -Lastly, pancreas secretes pancreatic lipase (can break down fats into free fatty acids and glycerol -Pancreatic juices empty into ducts that empty into duodenum through major and minor duodenal papillae Endocrine: Clusters of hormone producing cells = islets of Langerhans (made up of alpha (secretes glucagon), beta (insulin), and delta cells (somatostatin)) Endocrine functions: Release of insulin, glucagon, and somatostatin (peptide hormones that maintain sugar levels in blood) Glucagon: Secreted during fasting (Glucagon is high when glucose is GONE) -When blood glucose is high, glucagon release is inhibited -Stimulates protein and fat degradation Insulin: Antagonsitic to glucagon -Secreted when blood glucose levels are high -Tells cells to take up glucose and store as glycogen for later -Stimulates anabolic processes (fat and protein synthesis) -Somatostatin: Inhibits insulin and glucagon secretion -Stimulated by high blood glucose and amino acid levels -Produced in hypothalamus -decreases growth hormone (GH)

Skeletal Structure

Exoskeleton: Able to protect soft tissue underneath, must be shed and regrown Endoskeleton: (humans): Better able to accommodate growth Skeletal system divided into axial and appendicular skeletons (both covered in other structures, like muscle, connective tissue, and vasculature) Skeleton is created from BONE and CARTILAGE Adult has 206 bones, over 100 of which are in hands and feet Axial: Skull, vertebral column, ribcages hyoid bone (small bone in anterior neck for swallowing) -Basic central framework for body -Includes sternum/breatbone Appendicular: Bones of the limbs -Typically long bones -Upper: humerus, radius, Ulam carpals, metacarpals phalanges Lower: Femur, tibia, fibula, tarsals, metatarsals, phalanges Pectoral girdle: scapula and clavicle Pelvis

3 processes by which solutes are exchanged between filtration and blood: In the nephron, amino acids enter the vasa recta via the process of: A. filtration. B. secretion. C. excretion. D. reabsorption.

Filtration = In nephron, movement of solutes from blood into filtrate at Bowman's capsule. Secretin is movement of solutes from blood into filtrate anywhere BUT Bowman's capsule. Reabsorption is movement of solutes from filtrate into blood *Blood plasma forced under high pressure out of glomerulus into Bowman's capsule Correct Answer: D Explanation: Essential substances, such as glucose, salts, amino acids, and water, are reabsorbed from the filtrate and returned to the blood in the vasa recta. In general, reabsorption refers to the movement of solutes from the filtrate back into the blood.

Gestation (280 days) and Birth

First trimester: Organogenesis (heart, eyes, goals, limbs, liver, brain) heart beat at 22 days 15 mm by 6 weeks Skeleton starts to form by 7 weeks 8 weeks: most organs formed, brain fairly developed, now called fetus 3rd month: 9 cm long Second trimester: 6 months = 30-36 cm long Third trimester: Lots of antibodies transported Birth: Vaginal birth = parturition (coordinated by prostaglandins and peptide hormone oxytocin) First cervix thins and amniotic sac ruptures, then strong uterine contractions = birth, then placenta and umbilical cord are expelled (afterbirth)

Muscle Contraction: Shortening of the Sarcomere (step 2) *Actin-Myosin Cross-Bridge Cycle

Free globular heads of myosin molecules bind to exposed actin sites -Forms bridge that allows myosin to pull on actin, draws thin filaments toward M line = shortening sarcomere Top of diagram: Myosin carrying hydrolyzed ATP (ADP and Inorganic Phosphate (Pi)) binds to myosin binding site. Release of Pi and ADP provide energy that results in sliding actin over myosin filaments *Its the disassociation of ADP and Pi from myosin that gives power not hydrolysis of ATP (this is required for releasing myosin head from actin filament) ATP binds to myosin head -> releases it from actin -ATP hydrolyzed to ADP and Pi -> recooks myosin head so it can initiate cross-bridge cycle -Repetitive binding/releasing on actin filaments allows thin to slide along thick, causing sequential shortening (sliding filament model) *ATP BINDING ALLOWS MYOSIN FILAMENT TO DISCONNECT FROM ACTIN, DISSOCIATION OF ADP AND INORGANIC PHOSPHATE FROM MYOSIN CAUSES POWERSTROKE

Assume that a population is in Hardy-Weinberg equilibrium. If 9% of population is homozygous dominant, solve for: Frequency of Dominant allele Frequency of recessive allele Portion of population that is heterozygous Portion of population with homozygous recessive genotype Portion of population with dominant phenotype

Frequency of Dominant allele (p): 0.3 Frequency of recessive allele (q): 0.7 Portion of population that is heterozygous: 2pq = 2X0.3X0.7= 0.42 or 42% Portion of population with homozygous recessive genotype: q^2 = 49% Portion of population with dominant phenotype: p^2 + 2pq = 51%

Cells can enter a quiescent state, also called G0, in which they do not actively divide. Typically a cell would be considered to be in which phase of the cell cycle immediately before entering G0?

G1 G1 can either go to G0 or S The cell cycle consist of 4 stages: G1, S, G2, and M

Interphase (mitosis)

G1 phase, S phase, G2 phase Longest part of cell cycle (90% of time) Individual chromosomes are not visible (in less condensed form known as chromatin because DNA must be available to RNA polymerase so genes can be transcribed) G0: Cell is living and carrying out functions without preparation for division G1: Presynthetic Gap Cells create organelles for energy and protein production (mitochondria, ribosomes, ER), increase size -Must contain proper complement of DNA is order to pass RESTRICTION POINT and enter synthesis stage -G1/S Chechkpoint: is condition good enough for synthesis (restriction point) -P53: Main Protein stopping damaged DNA from going through the cycle S Stage: Synthesis of DNA -Replicates genetic material so daughter cell has identical copies -After replication, each chromosome consist of 2 identical chromates bound at centromere -PLOIDY OF CELL DOES NOT CHANGE EVEN THOUGH CHROMATIDS HAVE DOUBLED G2: Post synthetic Gap *Cells entering G2 have twice as much DNA as cells in G1 -Cell check to ensure that there are enough organelles and cytoplasm for 2 daughter cells and make sure DNA replication proceeded correctly

Lungs (gas exchange)

Gas Exchange: Capillaries bring deoxygenated blood from pulmonary arteries (originate from right ventricle of heart) -Walls of alveoli are 1 cell thick: facilitates diffusion of CO2 from blood into lungs, and O2 into blood -Oxygenated blood returns to left atrium of heart via pulmonary veins -Driving force for gas exchange: blood has low partial pressure of O2 when it arrives at alveoli and high CO2 = moment down concentration gradient (No energy required) *O2 in alveoli flow down partial pressure gradient into pulmonary capillaries, where it binds to hemoglobin for transport -CO2 flows down gradient from capillaries into alveoli for expiration Higher altitude/less oxygen: rapid breathing to avoid hypoxia -Natural response of hemoglobin in decreased CO2 to decrease unloading of O2 to tissues, so other mechanisms must counteract this -Body make more red blood cells to ensure adequate delivery of O2 (short term) -Long term: Body develop more blood vessels (vascularization) = facilitate distribution of O2 to tissues

Other organs that release hormones (but aren't part of endocrine system)

Gastrointestinal Peptides: secretin, gastrin, cholecystokinin Kidneys produce erythropoietin (stimulates bone marrow to increase red blood cell production, responds to low O2 levels in blood) Heart release atrial natriuretic peptide (ANP)(Lowers BP): Antagonistic to aldosterone, promotes excretion in response to stretch -Aids in loss of salt within nephron, acts as natural diuretic (pretty weak though, its a lot easier for us to raise BP than lower it) Thymus releases thymosin (important for T cell development, levels drop with age)

Gene Pool: Mutations, Leakage, and Genetic Drift

Gene pool: All of the alleles that exist within a species -New genes introduced by mutation or genetic leakage Mutations: Change in DNA sequence -> results in mutant allele Ways new mutations may be introduced: -Ionizing radiation (UV rays) -Chemical exposures/damage DNA Substance that can cause mutations = mutagens DNA polymerase makes mistakes during DNA replication Transposons can insert/remove self from genome, and it it inserts in the middle of a coding sequence, mutation will disrupt gene Incorrect pairing of nucleotides during transcription or translation, or a tRNA molecule charged with incorrect amino acid for its anticodon can result in derangement of normal amino acid sequence Consequences of mutations: -Can be advantageous (sickle cell) -Can be detrimental/deleterious (One example: Inborn errors of metabolism, like PKU) Genetic Leakage: Flow of genes between species through hybrid offspring *Requires formation of hybrid organism that can then mate with members of one or the other parent species (fertile hybrids are rare) Genetic Drift: Changes in composition of gene pool due to chance -More pronounced in small populations -Founder effect: Small population in reproductive isolation from others -> inbreeding -> increased homozygous genotypes -Founder effect often results from bottlenecks -Genetic drift, founder effect, and inbreeding all cause reduction in genetic diversity -Outbreeding/outcrossing = introduction of unrelated individuals into breeding group -> increased variation/fittness Mutations can be subdivided into nucleotide and chromosomal mutations -Nucleotide includes point (substituting nucleotide) and frameshift mutations (moving 3-letter frame) -Point mutations split into silent, missense, nonsense, -Frameshift: insertion/deletion -Chromosomal are larger, include deletion, duplication, inversion, insertion, and translocation

In order to test whether three genes (A, B, and C) are linked, a scientist performs a number of crosses and calculates the following recombination frequencies. Based on the data which of the following conclusions is most reasonable? AB: 49% BC 45% AC 4%

Genes are organized in a linear fashion on chromosomes. Crossing over during prophase I of meiosis causes alleles to be swapped between homologous chromosomes, supporting Mendel's second law (of independent assortment). However, genes that are located very close together on a chromosome are less likely to be separated from each other during crossing over. In other words, the further apart two genes are, the more likely it is that there will be a point of crossing over, called a chiasma, between them. The likelihood that two alleles are separated from each other during crossing over, called the recombination frequency (θ), is roughly proportional to the distance between the genes on the chromosome. We can also describe the strength of linkage between genes based on the recombination frequency: tightly linked genes have recombination frequencies close to 0 percent; weakly linked genes have recombination frequencies approaching 50 percent, as expected from independent assortment. As stated, genes that are not linked will have a recombination frequency close to 50% because they follow the Law of Independent Assortment. Only AC has a frequency close to 0%, so the answer is (B).

Allele

Genes coding for alternative forms of a given trait

Gene Mapping

Genes organized in linear fashion on chromosome Crossing over during prophase 1 = alleles swapped between homologues -> supports Mendel's 2nd law (IA) -But, genes closer together more likely to be swapped -Point of crossover = chiasma -Recombination Frequency: Likelihood that 2 alleles are separated from each other: Proportional to distance between genes (can also describe strength of linkage -> tightly linked = 0%, weakly linked have recombination frequency approaching 50%, which is expected from independent assortment) Recombination frequencies can be used to construct genetic map (shows relative distance between genes on chromosome) -1 map unit/centimorgan corresponds to 1% chance of recombination (If 2 genes were 25 map units apart, expect 25% of total gametes examined to show recombination between two genes) *If recombination frequencies are known, one can deduce order of genes on chromosome

Founder effect

Genetic drift On one island, all plants of that genus lack a compound produced by all other members of that genus on other islands in island chain Change in gene frequencies that occurs as a result of chance Occurs when small population is isolated from larger one, so it has a substantially different allele distribution

Mitosis and Meiosis Similarities

Genetic material must be duplicated Chromatin is condensed to form chromosomes Microtubules emanating from centrioles are involved in dividing genetic material

Other cells in the Nervous System (Besides neurons)

Glial cells/neuroglia: structural and supportive role Long axons insulated with myelin carry signals faster than unmyelinated Oligodendycytes manufacture myelin and wrap axon with 10-150 layers Astrocytes: Nourish neurons, form blood brain barrier Ependymal Cells: Line ventricles, produce CSF Microglia: Phagocytic/ingest and break down waste in CNS Oligodendrocytes (CNS) and Schwann Cells (PNS) produce myelin around axons

Granulocytes (leukocytes) Ends in ~phil

Granulocytes: Basophils (closely related to mast cells), neutrophils, eosinophils (Think of parasites), mast cells -Contain cytoplasmic granules (contains toxic enzymes and chemicals released via exocytosis) -Inflammation, allergy, destruction, exocytosis Neutrophils: Most abundant leukocyte, very short lived (5-days), phagocytic, target bacteria -Follow bacteria by chemotaxis (moving up concentration gradient) -Can detect when bacteria have been opsonized (marked with antibody from B cell) -pus= dead neutrophil collection Eosinophils: Bright red/orange granules -Allergic reactions and invasive parasites -Release large amounts of histamine *inflammatory mediator) -Histamine causes inflammation by inducing vasoldilation and movement of fluid/cells (like macro and neutrophils) from blood to tissues / increased leakiness -Inflammation especially useful for extracellular pathogens (bacteria, fungi, parasites) Basophils: Large purple granules -Allergic response/reaction -Least produced leukocyte -Closely related to mast cells, both release lots of histamine, but mast cells have smaller granules and exist in tissues, mucosa, and epithelium

Which of the following statements concerning growth hormone is NOT true? A. Overproduction of growth hormone in adults results in acromegaly. B. It promotes growth of bone and muscle. C. It is produced by the hypothalamus but secreted by the pituitary. D. A childhood deficiency in growth hormone results in dwarfism.

Growth hormone is a direct hormone secreted by the anterior pituitary. Among its many functions, GH promotes growth in bone and muscle, eliminating choice (B). An overproduction of growth hormone in children results in gigantism, whereas in adults it results in acromegaly (enlargement of the small bones in the extremities and of certain facial bones, such as the jaw), eliminating choice (A). On the other hand, a childhood deficiency of growth hormone results in dwarfism, eliminating choice (D). GH is synthesized and secreted in the anterior pituitary; choice (C) describes antidiuretic hormone and oxytocin, not GH.

Notes

Human body contains 37 trillion cells, bacterial outnumbering eukaryotic by 10 to 1

Homologous Chromosomes and sister chromatids

Humans have 23 pairs of homologous chromosomes Whereas as homologous pairs are considered separate chromosomes (Such as maternal chromosome 15 and paternal chromosome 15), Sister chromatids are identical strands of DNA connected at the centromere After S phase, there are 92 chromatids organized into 46 chromosomes which are organized into 23 homologous pairs Sister chromatids: Each chromatid is composed of a complete double stranded molecule of DNA Identical copies of each other Chromosome can refer to single chromatid before S phase or the pair of chromatids attached at centromere after S phase In S phase, the policy of the cell does not change even though the # of chromatids has doubled

Hypothyroidism/ Cretinism/ Hyperthyroidism

Hypothyroidism: Deficiency of iodine or inflammation of the thyroid Thyroid hormones secreted in insufficient amounts/not at all Lehtargy, decreased body temp, slowed respiratory and Heart rate, cold intolerance, weight gain Cretinism: Intelelctual disability and developmental delay from inappropriate thyroid hormone levels (newborn infants) Hyperthyroidism: Excess of thyroid hormone, may result from tumor or thyroid (heightened activity level, increased body temp, increased respiratory and HR, heat intolerance, weight loss)

Adaptive Immune System

Identification of specific pathogen (takes time) Adaptive IM consists of B-cells/lymphocytes and T-cells/lymphocytes Divided into humoral immunity and cell-mediated (cytotoxic) immunity For cell-surface activation: Binding antigen to B cell causes activation = proliferation and formation of plasma and memory cells -In contrast, when antigen binds to antibodies on surface of mast cell, it causes degranulation (exocytosis of granule contents), releasing histamine Primary Response: Takes 7-10 days (initial activation) Secondary Response: More rapid and robust Development of lasting memory cells is basis of efficacy of vaccination

Joints and Movement

Immovable Joints: Fused together to form sutures/similar fibrous joints (in head, anchor skull bones together) Movable joints: -Hinge joints (elbow, knee) -Ball and socket joints (shoulder or hip) -Strengthened by ligaments (fibrous tissue that connects bone to bone, consists of synovial capsule, which encloses the actual joint cavity (articular cavity)) -Layer of soft tissue called synovium secretes synovial fluid (lubricates moveable joints) *Synovial fluid lubricates the movement in the joint space, but does not stop the bones from contacting one another; this is the job of articular cartilage -Articular cartilage coats surface of bones so impact is restricted to lubricated joint cartilage (rather than to bones) *The end of the muscle with larger attachment to bone (usually proximal connection) is called the origin -The end with smaller attachment (usually distal) = insertion Muscles can be classified by types of movements they coordinate -Flexor muscle decreases angle across joint (bicep brachii) -Extensor increases or straightens this angle (triceps brachii) -Abductor: A muscle that moves body away from midline (pectoralis major) -Medial and lateral rotation describes motions in limbs -Medial rotator rotates axis of limb toward midline (subscapularis) -Lateral rotator rotates axis away from midline (infraspinatus)

Hardy-Weinberg Principle

Implies that study population is not undergoing evolution, thus, allele frequency will remain stable Allele frequency = how often allele appears in population (doesn't tell us homozygous or heterozygous) -Evolution results from changes in gene frequencies, so when gene frequencies are not changing gene pool is sable and evolution is not occurring. Five criteria are mandatory for this to be possible (Hardy Weinberg equilibrium): 1. No mutations 2. Random mating (no sexual selection) 3. No natural selection/ genes are equally successful at being reproduced 4. Extremely large population size (no genetic drift) 5. No gene flow/ No migration into or out of population p is frequency of dominant allele q is frequency of recessive allele p+q = 1 (100%) (tells us frequency of alleles) p^2 + 2pq + q^2 = 1 (Tells us frequency of genotypes/phenotypes) where p^2 is frequency of homozygous dominant genotype, 2pq is heterozygous, and q^2 is frequency of homozygous recessive genotype p^2 + 2pq = frequency of dominant phenotype Lets you determine relative frequency of alleles in population and frequency of a given genotype or phenotype in population *Note: TWICE AS MANY ALLELES AS INDIVIDUALS IN POPULATION BECAUSE EACH INDIVIDUAL HAS 2 AUTOSOMAL COPIES OF EACH GENE Allele frequencies will remain constant at HW equilibrium

When an action potential is generated, which occurs first? Increased permeability to K+ Increased permeability to Na+ Increased permeability to Cl- Decreased permeability to Ca2+

Increased permeability to Na+ When action potential is generated, neuronal membrane is depolarized caused by influx of Na+ into neuron Reason other answers were incorrect: Increased permeability to K+ occurs when neuronal membrane is repolarized Negatively charged chloride ions are used to stabalize the membrane potential because negatively charged CL ions will cancel out Na+ ions (Increased permeability to Cl- will cancel out depolarization and no action potential will be propagated) Calcium ion has no effect on action potential. Influx of Ca causes release of neurotransmitter vesicles into the synaptic cleft *The release of Ca2+ from the sarcoplasmic reticulum is responsible for muscle contractions

Cell-Cell Communication

Inducer: Cell secreting signal (often growth factors: Peptides that promote differentiation and mitosis) -Induction is not a one way pathway (reciprocal development) Cell that is induced = responder (must be competent) Communication occurs via autocrine (acts on same cell that secreted signal), paracrine (acts on cells in local area), juxtacrine (don't usually involve diffusion, cell directly stimulating receptors of adjacent cell)(occurs between adjacent cells, but the signal does not spread by diffusion), or endocrine signals (secreted hormones through bloodstream to distant target) Signaling can occur via gradients

Innate and Adaptive Immunity

Innate: Composed of defenses that are always active, but non-specific, phagocytes (invest and destroy), DCs and macrophages (secrete cytokines (proteins) to trigger influx of immune cells like monocytes (mature in to macrophages) and neutrophils) Adaptive: Target specific pathogen, initially slower but memory allows for faster reactions to subsequent attacks -B cells and T cells: Both spawn memory cells -B cells secrete antibody molecules that bind to antigens and destroy/mark it (specific) -Activated B cell = plasma cell -T cells: Recognize antigens, activate B and T cells, attack directly -Agents of cell-mediated immunity: Coordinate immune system and directly kill infected cells

Robert Hooke

Invented crude microscope and was first to observe "small chambers" in cork and call them cells (1665) Also known for characterization of spring/Hooke's law (physics) F=-kx

MHC I (Major Histocompatibility Complex)

Joined with antigens: Then MHC-Antigen complex goes to cell surface to display antigen All nucleated cells in body display MHC class 1 (any protein produced can be loaded/presented) -Monitors cell health, because only infected cells display unfamiliar/non-self proteins on surface -Presents endogenous antigens (binds to antigens that come from inside the cell) -Cells invade by intracellular pathogen killed by cytotoxic T-cells/lymphocytes *Picture shows endogenous pathway for antigen presentation (MHC 1): IN ALL NUCLEATED CELLS Important to match donor/recipient MHC as closely as possible

Kidneys

Kidneys: Bean shaped, behind digestive organs at level of bottom rib -Filters about 180 Liters blood/day (36X our volume) -Entire volume of blood filtered every 40 minutes -Filters blood to form urine (Low blood volume/high osmolarity = retain water = less, more highly concentrated urine) -Kidneys regulate blood flow and osmolarity -Functional unit is nephron (each kidney has around 1 million) -All nephrons empty into renal pelvis, narrows to form ureter -Urine travels through ureter to be stored in urinary bladder to urethra -Each kidney subdivided into cortex and medulla Cortex is outermost layer, medulla within cortex Each kidney has renal hilum (deep slit in center) Renal Pelvis: Widest part, spans almost entire width of renal hilum Renal artery, vein, and ureter enter and exit through the renal hilum Bowman's capsule: Located around glomerulus, cupcake structure *20% of blood from glomerulus filtered into Bowman's space (collected fluid known as filtrate) *Filtrate: No cells/proteins (because glomerular pores are tiny), isotonic to blood so capsule/capillaries don't swell *If blood cells/proteins are found in urine, problem is glomerulus *amount of fluid moved in controlled by starling forces (pressure differential: Hydrostatic pressure in glomerulus higher than bowman = fluid moves into nephron (even though oncotic pressure pushes other way)) *FLUID FROM GLOMERULUS TO BOWMAN'S SPACE -Problems: obstruction (stone) = fluid buildup = increase in hydrostatic pressure in Bowmans space = no more filtration because of opposing movement Kidney function divided into 3 processes: Filtration (Movement of solutes from blood to filtrate at Bowman's capsule), secretion of salts, acids, bases, urea (Movement of solutes from blood to filtrate anywhere besides Bowman's capsule), and reabsorption (Movement of solutes from filtrate to blood) *Almost always reabsorbed: glucose, amino acids, vitamins (hormones like ADH/vasop, aldosterone can alter quantity reabsorbed to maintain BP) Major waste Products: H+, Urea, NH3, K+

Chromosomal Mutations

Large-scale mutations in which large segments of DNA are affected Deletion: Large segment of DNA lost (small deletion = frameshift) Duplication: Segment copied Inversion: Segment reversed Insertion: Segment moved from one chromosome to another (small insertion, including when inserted DNA is not from another chromosome = frameshift mutation instead) Translocation: Segment swapped with segment of DNA from another chromosome

Insulin and glucagon

Lots of insulin = hypoglycemia -Underproduction/insufficient/insensitivty = diabetes mellitus/hyperglycemia Kidneys overwhelmed by excess glucose = polyuria and polydipsia (increased thirst) Diabetes = autoimmune destruction of beta cells of pancreas resulting in low or absent insulin production (so glucose gets too high) *Insulin decreases glucose, glucagon increases glucose (So can GH, glucocorticoids, and epi)

During which phase of the menstraul cycle is progesterone at the highest concentration?

Luteal Phase About a week after ovulation (after this, levels of progesterone and estrogen drop) Progesterone peaks during the luteal phase, as it supports the endometrium for potential implantation of a blastula. Progesterone levels are relatively low during follicular phase and ovulation. Withdrawal of progesterone causes menses.

Anatomy of the Immune and Lymphatic Systems

Lymphatic System: Circulatory system, made up of one-way vessels that become larger as they move towards center of body -Join to form thoracic duct (posterior chest) which delivers fluid (lymph) into left subclavian vein (near heart) *Thoracic duct is primarily responsible for returning materials from lymphatic circulation to cardiovascular system Bone Marrow: Produces all leukocytes (WBC) through hematopoiesis/ site of immune cell production Spleen: Storage area for WBV and platelets, recycling center for RBC, filters blood and lymph/Location of blood storage and activation of B cells to plasma cells to produce antibodies (adaptive) -When B cells leave BM, they are mature but naive because they haven't been exposed to an antigen yet -Humoral Immunity: Antibodies dissolve and act in blood, not in cells (adaptive) T cells mature in thymus: Small gland in front of pericardium, sac that protects heart Lymph Nodes: Small, bean shaped structures: Contain lymphatic channel, artery, and vein -Filter lymph and place where immune system can be mounted/cells communicate/ B cells can be activated here -Provide space for cells of immune system to be exposed to possible pathogens *Cancers can spread via lymphatic channels Gut-Associated Lymphoid Tissue (GALT)/Immune tissue: Tonsils and adenoids (head), Peyer's patches (small intestine), lymphoid (aggregates in appendix)

MHC 2 (Major Histocompatibility Complex)

MHC-II is an antigen-presenting cell and presents exogenous antigens (Antigens originate outside of the cell) -Presents proteins that result from digestion of extracellular pathogens that have been brought in by endocytosis (Exogenous antigen) -Only displayed on antigen presenting cells -Professional antigen presenting cells: macrophages, dendritic cells in skin, some B-cells, and certain activated epithelial cells -Pick up pathogens from environment, process, and present -Antibody production is domain of adaptive immune system, but innate present antigens too

Osteoclasts

Macrophages Dissolve bone to release calcium Do not respond to infections (no B or T cells)

Bacterial (Extracellular pathogen) Infections

Macrophages engulf and release inflammatory mediators Cells digest bacteria and present antigens from pathogen on surface in conjunction with MHCII Cytokines attract more cells (neutrophils, macrophages) Mast cells activated and degranulate, releasing histamine and increasing capillary leakage DC cells travels to lymph node where it presents antigen to B cells B cells proliferate through clonal selection (can also occur with T cells) to create plasma and memory cells Antibodies travel through the bloodstream to the affected tissue to tag bacteria for destruction At the same time, Dendritic cells are also presenting the antigen to T-cells, activating a T-cell response with CD4+ T cells (TH1 and TH2). These release interferon gamma (IFN-Y) which activates macrophages/increases killing TH2 helps activate B cells (more common with parasites) Pathogen eliminated = plasma cells die, B and T cells remain

Peripheral Nervous System

Made up of nerve tissue and fibers outside the brain and spinal cord -Includes all 31 pairs of spinal nerves and 10 of 12 cranial nerves (olfactory and optic considered CNS) -PNS connects CNS to body -Subdivided into somatic and autonomic Somatic: Sensory and motor neurons distributed through out skin, joints, and muscles. Sensory neurons transmit info through afferent fibers Motor impulses travel along efferent fibers ANS regulates heart beat, digestion, respiration, and glandular secretions (involuntary muscles), regulate body temp by sweating/piloerection -Perihperal component of ANS contains 2 neurons (by contrast, motor neuron in SNS goes directly from spinal cord to muscle without snapping, but in ANS 2 neurons work in series to transmit messages from spinal cord) -First neuron = preganglionic neuron (soma in CNS, axon travels to ganglion in PNS), second = postganglionic neuron. Autonomic Nervous System has two subdivisions: Sympathetic (accelerate HR, inhibit digestion, redistributes blood to locomotion muscles, increases blood glucose, relaxes bronchi, decreases digestion/peristalsis, dilates eyes to maximize light intake, releases epinephrine, stimulates orgasm, inhibits bladder contraction and salivation, secretion of noradrenalin/adrenaline) and parasympathetic (promotes digestion, slows HR) *In sympathetic, preganglionic neurons release acetylcholine, postganglionic release norepinephrine Main role of parasympathetic is to conserve energy (associated with rest, constricts bronchi and pupils, increasing peristalsis and exocrine secretions and bile release, contracts bladder, neurotransmitter = acetylcholine, vagus nerve (cranial nerve X))

Lysosomes

Membrane-bound structures containing hydrolytic enzymes there can break stuff down Work with endosomes (transport, package, and sort cell material) Lysosomal membrane releases hydrolytic enzymes in process called autolysis which causes apoptosis (cell death) via degradation SURROUNDED BY SINGLE MEMBRANE

Menstrual Cycle

Menarche to menopause *Follicule stimulating hormone (FSH) facilities the maturation of single ovum The peak of luteinizing hormone at day 14 marks ovulation (release of oocyte from follicle) Endometrial ling of uterus reaches peak in luteal phase and is shed at beginning of next cycle Menstraul cycle divided into 4 phases: 1. Follicular phase: Menstraul flow begins, shed uterine lining, GnRH secretion from hypothalamus increases in response to decreased in estrogen and progesterone. More GnRH = more FSH and LH = production of estrogen (negative feedback effects, causes GnRH, LH, and FSH concentrations to level off) -Estrogen stimulates regrowth of lining, stimulating decidua 2. Developing follicles secrete more and more estrogen, reach threshold = positive feedback, and GnRH , LH, and FSH levels spike. LH spike induces ovulation (ovum from ovary to abdominal cavity) 3. Luteal Phase: LH causes ruptured follicle to form corpus lute, which secretes progesterone (levels rise, estrogen remains high) -High progesterone = negative feedback on GnRH, FSH, and LH, preventing ovulation of multiple eggs Menstruation: No implantation = corpus lute loses stimulation from LH, progesterone levels decline, uterine lining sloughed off. Loss of high levels of estrogen and progesterone removes block on GnRH so next cycle begins -If fertilization occurs: Zygote becomes blastocyst, implants in uterine lining and secretes human chorionic gonadotropin (hCG) (analog of LH) (Maintains corpus luteum) (Levels of hCG fall by second trimester because placenta can secrete estrogen and progesterone by itself, high levels of both continue to serve as negative feedback on GnRH secretion) Menopause: Ovaries less sensitive to FSH and LH -Negative feedback on FSH and LH removed = levels rise

Mendel's first law (of segregation) most aligns with: Mendel's second law (of Independent Assortment) most aligns with: (what phase?)

Mendel's first law (of segregation) most aligns with: anaphase 1 of meiosis Mendel's second law (of Independent Assortment) most aligns with: prophase 1

Mendelian Genetics (1860's)

Mendel's first law: Law of segregation *An organism has 2 alleles for each gene, which segregate during meiosis, resulting in gametes carrying one allele for a trait (4 basic tenets) *Genes exist in alternative forms (alleles) *An organism has 2 alleles for each gene (1 inherited from each parent) *The 2 alleles segregate during meiosis, resulting in gametes that carry only one allele for any inherited trait *If 2 alleles are different, only one will be fully expressed and the other will be silent. Expressed allele is domainant, silent allele is recessive Separation of homologous chromosomes during anaphase 1 of meiosis is what is important here: separating chromosomes so each gamete carries only one allele for trait Mendel's Second Law: Law of independent Assortment *Inheritance of one gene does not affect inheritance of another allele for a different trait During prophase 1, homologous chromosomes form tetrads (2 sister chromatids in two homologous chromosomes) and exchange small amounts of genetic material = novel combinations/recombination -This allows inheritance of one gene to be independent of the inheritance of all others Mendel did not know that DNA was genetic material

Mammals

Milk producing mammary glands, three bones in middle ear and one int he lower jaw, fur/hair, heterodont dentition (different kinds of teeth), and both sebaceous (oil producing) and sudoriferous (sweat) glans Prototherians: Monotremes, ovaparity Metatherians: Marsupials

cyclins and cyclin-dependent kinases (Cdks)

Molecules responsible for cell cycle CDKs require certain cyclins to become activated Activated CDK-Cyclin complex can phosphorylate transcription factors (Promote transcription of genes required for next stage)

Punnett Squares

Monohybrid: one trait being studied Offspring are filial (F) generation Grandparents = P generation, your parents = F1, you are F2 Crossing 2 heterozygotes for a trait with complete domainance results in 1:2:1 ratio of genotypes and 3:1 ratio for phenotypes *Usually, more offspring parents have = closer to phenotypic ratios are to expected -Test cross/back cross: unknown genotype crossed with homozygous recessive (if result is 1:1 dominant to recessive phenotypes, unknown likely heterozygous) *If a test cross on a species of plants reveals the appearance of a recessive phenotype in the offspring, dominant parent has to be heterozygous Dihybrid cross: 2 different genes -Mendels second law of independent assortment holds true for unlinked genes -Punnett square is 4X4 -Crossing 2 heterozygous gives phenotypic ratio of 9:3:3:1 Sex-Linked (X-Linked)(Assume X-linked recessive) -Females = XX (Can be carrier) -Males = XY (males can be hemizygous for disease, disease more common in males, having only 1 recessive allele is sufficient for expression of recessive phenotype) -Becuase egg always carries X, sperm determines the sex of child, so men with a sex-linked trait will have daughters who are either carriers or have it, and a diseased man can NEVER PASS A SEX LINKED TRAIT TO HIS SON)

Microscopic Bone Structure Most inorganic bone is composed of:

Most inorganic bone is composed of hydroxyapatite crystals - compact bone is a dense, hardened bone marix that contains organic and inorganic components - organic: collagen fibers & glycoproteins and other peptides - inorganic: calcium, phosphate, hydroxide (combine & harden to form hydroxyapatite crystals), and minerals like sodium, potassium and magnesium -Strong bones require uniform distribution of organic and inorganic -Bony matrix ordered into structural units called osteons/Haversian systems (each contains concentric circles of bony matrix called lamellae surrounding central microscopic channel) -Longitudinal channels (with axis parallel to bone) = Haversian canals -Transvere channels (axis perpendicular) = Volkmann's canals -Both canals contains blood vessels, nerve fibers, lymph vessels -Between lamellar rings are small spaces called lacunae (house mature bone cells = osteocytes) -Lacunae are interconnected by canaliculi (Small canals connecting lacunae within the bone matrix allow with Haversian canals, allow for exchange between osteocytes and Haversian/Volkmann's canals)

Gastric glands and pyloric glands of stomach What are the 6 products that the stomach secretes? (HPMBWI)

Mucosa of stomach contains gastric glands and pyloric glands *Gastric glands respond to signals from vagus nerve of paraSNS (activated by brain in response to senses): 3 different types: Mucous cells, chief cells, and parietal cells *Gastric juice is a combination of chief cells and parietal cells -Chief cells secrete pepsinogen -H+ ions in stomach secreted by parietal cells as hydrochloric acid cleave pepsinogen to pepsin (digests proteins, cleaves peptide bond by aromatic amino acids, most active at low pH) -Parietal cells also secrete intrinsic factor: Glycoprotein involved in proper absorption of B12 1. HCl (kills microbes, denatures proteins, converts pepsinogen to pepsin) 2. Pepsinogen (cleaved in stomach to pepsin (enzyme that partially digests proteins)) 3. Mucus (protects mucosa) 4. Bicarbonate (Protects mucosa) 5. Water (dissolves and dilutes ingested stuff) 6. Intrinsic Factor (required for normal absorption of B12) Pyloric Glands: Contain G cells that secrete GASTRIN (peptide hormone, induces parietal cells to release more HCl and signals contraction -> results in chyme)

List major secretions and functions of: Mucous Cell Chief Cell Parietal Cell G-cell

Mucous Cell: -Secretes mucus -Produce bicarbonate rich mucus -Protects lining of stomach -Increases pH (bicarbonate) Chief Cell: -Secrete pepsinogen (inactive, zymogen form of pepsin, a proteolytic enzyme) -Pepsinogen activated by acidic environment off stomach -Digests proteins, activated by H+ Parietal Cell: -Reduces pH by secreting HCl -Also secretes intrinsic factor: Absorbs B12 -Kills microbes, denatures proteins, carries out some chemical digestion G-cell: Contained in pyloric glands -Secrete gastrin (peptide hormone that induces parietal cells to secrete more HCl and signals stomach to contract) -Increases HCl production and increases gastric motility

Diaphragm

Muscle extending across bottom of ribcage Separates thoracic cavity (lungs and heart) from abdominal cavity In relaxed state, it is dome shaped (shortened thoracic cavity = exhale), but when it contracts/flattens, it causes thoracic cavity to enlarge (drawing in air) (reduces intra-thoracic pressure)

Stimulation, Summation, and Muscle Fatigue

Muscle fibers contract in all or nothing fashion because they are innervated by neurons that deliver signals using action potentials (Both muscle cells and neurons are all or nothing) -Either respond completely or not at all (strength of response cant be changed, so change comes from number of motor units recruited to respond) *Maximal response occurs when all fibers within muscle cell are stimulated to contract simultaneously Simple Twitch: Response from single muscle fiber to brief stimulus at or above threshold -Latent period (time between reaching threshold and onset of contraction): During this time action potential spreads along muscle and allows for calcium to be released from sarcoplasmic reticulum), contraction, and relaxation period (assuming calcium is cleared form sarcoplasm) Summation and Tetanus: -Frequent and prolonged stimulation = insufficient time to relax: Contractions will combine and become stronger/more prolonged = frequency summation) Summation of frequent simple twitches can lead to tetanus (contractions so frequent that there is no relaxation at all) *Tetanus is summation of multiple simple switches that occur too quickly for muscle to relax: Leads to stronger and more prolonged contraction -Tetanospasmin blocks release of GABA from neurons that inhibit motor neurons, making motor neurons over-excitable -> leads to super strong contraction/leads to muscle fatigue (can be prevented by tetanus immunoglobulin)

Oxygen Debt and Muscle Fatigue

Muscles require ATP Slow twitch (red) muscle fibers have high levels of mitochondria/ use oxidative phosphorylation to make ATP (lots of O2 needed) 2 supplemental energy reserves in muscle: 1. Creatine phosphate: created by transferring phosphate from ATP to creatine during rest (can be reversed during muscle use to get ATP quickly from ADP) 2. Myoglobin: binds O2 with high affinity -Exercise = muscles run out of O2 -> use myoglobin reserves -Fast twitch (white) have fewer mitochondria -> must rely on glycolysis and fermentation to make ATP -Exercise = HR and respiratory rate increase = oxyhemoglobin dissociation curve shifts to right (Increased CO2, H+, and temperature) -> when this gets overwhelmed and even red muscle switch to anaerobic, you get lactic acid/fatigue Oxygen debt: What muscles need vs what they have -Most lactic acid is converted back into pyruvate (enters citric acid cycle)(requires O2) *Amount of O2 required to recover from strenuous exercise = oxygen debt

Which of the following best describes what myelin's role is in terms of capacitance? Myelin decreases capacitance by blocking the accumulation of oppositely charged molecules along the axonal membrane Myelin decreases capacitance by generating a magnetic field to accelerate ions in accordance with Lorenz's Law Myelin increases capacitance by blocking the accumulation of oppositely charged molecules along the axonal membrane Myelin increases capacitance by generating a magnetic field to accelerate ions in accordance with Lorenz's Law

Myelin decreases capacitance by blocking the accumulation of oppositely charged molecules along the axonal membrane Capacitance is the ability of a system to store charge, or, in this case, sequestration of opposite charges along the cell membrane, which basically results in a neutralization of charge differences. Higher capacitance results in a lower potential difference across the membrane (because it doesn't take much energy to get strong opposing charges to come together, remember from physics that C = Q/V so as capacitance increases, voltage or potential difference decreases)(you could also think of it as less charge (Q) accumulation = less capacitance) . This weakens the electrical gradient, which is part of the electrochemical gradient seen during the firing of action potentials. Increased capacitance requires a greater ion concentration difference across the membrane. Myelin insulates the neuron, which decreases its capacitance (higher potential difference/ voltage). It also allows for the concentration of ion channels at the nodes of Ranvier. Both of these effects serve to increase conduction velocity; decreasing capacitance by decreasing the necessary concentration of ions to conduct the signal, concentrating ion channels by allowing the necessary concentration to be reached more rapidly. This matches (A).

All of the following are associated with the myelin sheath EXCEPT: A. faster conduction of nerve impulses. B. nodes of Ranvier forming gaps along the axon. C. increased magnitude of the potential difference during an action potential. D. saltatory conduction of action potentials.

Myelin is a white lipid-containing material surrounding the axons of many neurons in the central and peripheral nervous systems. It is arranged on the axon discontinuously; the gaps between the segments of myelin are called nodes of Ranvier, eliminating choice (B). Myelin increases the conduction velocity by insulating segments of the axon so that the membrane is permeable to ions only at the nodes of Ranvier, eliminating choice (A). The action potential jumps from node to node, a process known as saltatory conduction, eliminating choice (D). Action potentials are often described as being "all-or-nothing"; the magnitude of the potential difference in an action potential is constant, regardless of the intensity of the stimulus. Thus, myelin does not affect the magnitude of the potential difference in an action potential, making choice (C) the correct answer.

T cells that would react with self-proteins are normally inactivated in the thymus. This occurs through _________ selection

Negative selection: Eliminating cells that respond Positive selection means choosing/keeping the immune cells that would respond to a particular antigen

Which of the following correctly describes a difference between nerves and tracts? A. Nerves are seen in the central nervous system; tracts are seen in the peripheral nervous system. B. Nerves have cell bodies in nuclei; tracts have cell bodies in ganglia. C. Nerves may carry more than one type of information; tracts can only carry one type of information. D. Nerves contain only one neuron; tracts contain many neurons.

Nerves are collections of neurons in the peripheral nervous system and may contain multiple types of information (sensory or motor); they contain cell bodies in ganglia. Tracts are collections of neurons in the central nervous system and contain only one type of information; they contain cell bodies in nuclei. A relative lack of ribosomes in axons means that new proteins must be transported large distances-> Axons are devoted almost entirely to conduction of action potentials and for the most part lack the machinery required for cellular repair. The nucleus and various organelles are present in the cell body, so repair of a severed axon requires the transport of the necessary proteins over large distances In PNS, multiple neurons bundled together to form nerve: Sensory, motor, or mixed (Cell bodies of neurons of same type clustered together into ganglia) In CNS, axons may be bundled together to form tracts. Unlike nerves, tracts only carry one type of information. Cell bodies of neurons in the same tract are grouped into nuclei Neuronal Tract: Bundles of neurons in the central nervous system are commonly referred to as tracts. This includes the brain and spinal cord, but not peripheral neurons

Reflexes

Neural circuits called reflex arcs control reflexive behavior Sensory neurons connect with interneurons to get pain signal to brain, but also, Rather than wait for brain to send out signal interneurons in spinal cord send signals to muscles directly 2 Types of reflex arcs: Monosynaptic: Single synapse between sensory neuron that receives stimulus and motor neuron that responds to it (knee jerk reflex) Stretch on patellar tendon makes body think that muscle may be getting overstretched (muscle contracts to prevent injury) (info travels up sensory (afferent, presynaptic) neuron to spinal cord where it interfaces with motor (efferent, postsynaptic) neuron that causes contraction Polysynaptic: At least 1 interneuron between sensory and motor neurons -Stepping on nail/withdrawal reflex (monosynaptic reflex is pulling foot from nail, but other leg muscles stimulated/extended to maintain balance) -Interneurons in the spinal cord provides connections from the incoming sensory info to motor neurons in the supporting limb *In a monosynaptic relflex, a sensory (afferent, presynaptic) neuron fires directly onto a motor (efferent, postsynaptic) neuron. In a polysynaptic reflex, a sensory neuron may fire directly onto a motor neuron, but interneurons are used as well. These interneurons fire onto other motor neurons

Synapse

Neurons aren't in direct physical contact Presynaptic neuron (releases synaptic vesicles)-> synaptic cleft -> post synaptic neuron (Post-synaptic cell also called effector) *Within neuron, electricity is used to pass signals down the length of axon. Between neurons, chemicals (neurotransmitters) are used to pass signals to subsequent neuron/gland/muscle *Prior to release, neurotransmitters stored in nerve terminal , then when action potential reached, calcium channels open and calcium floods in -> exocytosis of the neurotransmitter *If receptor is ligand gated ion channel, postsynaptic cell will either be depolarized or hyper polarized. If it is a G protein-coupled receptor, it will cause either changes in levels of cyclic AMP (cAMP) or an influx of calcium

Oogenesis

No unending supply of eggs, woman born with all she will have By birth, oogonia have already undergone DNA replication = primary oocytes (2N) Menarche = first period (1 primary oocyte per month will complete meiosis 1, producing secondary oocyte and polar body)(Unequal cytokinesis) -Secondary oocyte remains arrested in meiosis 2 unless fertilization occurs -Oocytes surrounded by zona pellucida (surrounds oocyte, acellular mix of glycoproteins to protect and allow sperm binding) and corona radiata (outside soon, layer of cells adhered to oocyte during ovulation -meiosis 2 triggered when sperm penetrates these layers with help of acrosomal enzymes -Secondary oocyte split into mature ovum and polar body -Ovum contributes almost everything to zygote (sperm only contributes half of DNA) -After meiosis 2 = haploid pronuclei of sperm and ovum join = zygote

Anatomy and Mechanism of Breathing

Nose and mouth remove dirt from air, warm and humidify it before it reaches lungs Lungs in thoracic cavity (Also contains heart, cheat wall forms outside) Gas exchange occurs in lungs Air enters respiratory tract through external NARES of nose, passes through nasal cavity, filtered by mucous membranes and nasal hairs (vibrissae) -Then, air passes through pharynx and larynx Pharynx behind nasal cavity/back of mouth (pathway for air-> lungs and food -> esophagus) -Larynx below pharynx, used only for air (Opening/glottis covered by epiglottis during swallowing) -Larynx contains two vocal cords -From larynx, air passes into cartilaginous trachea into one of two bronchi -Ciliated epithelial cells in bronchi and trachea catch material -In lungs, bronchi divide into bronchioles, divide into alveoli (small sacs that interface with pulmonary capillaries, where gas exchange occurs) -Alveolus: Basic functional unit of lung (tiny sac specialized for passive gas exchange between lungs and blood) -Each alveolus surrounded by capillaries and coated with surfactant (lowers surface tension/prevents collapse) *premature infants often require ventilation using positive end-expiratory pressures. at the end of expiration, the ventilator will provide a higher pressure than normal, which forces extra air into the alveoli. This pressure must be used to prevent alveolar collapse, which should remind you that surfactant serves the same purpose by reducing surface tension. Thus, it makes sense that if premature babies lack surfactant, providing extra air pressure at the end of expiration would be beneficial. -Alveoli = large surface area for gas exchange (100m^2) -Left lung smaller with 2 lobes instead of three because of heart position -Lungs do not fill passively, require skeletal muscle to generate negative pressure -Needs diaphragm: thin/muscular, divides thoracic cavity from abdominal cavity, under somatic control

Telophase 1

Nuclear membrane forms around each new nucleus Each chromosome still consists of 2 sister chromatids joined at the centromere Cells are now haploid Once homologous chromosomes separate, only n chromosomes are found in each daughter cell (23 in humans) The cell divides into 2 daughter cells by cytokinesis Interkinesis: Between cell divisions, short rest period where chromosomes partially uncoil

Pathway of retroviral nucleic acids from infection of a host cell --> release of viral progeny:

Nucleic acid enters as single stranded RNA, which undergoes reverse transcription (using reverse transcriptase) to for, double stranded DNA This DNA enters host genome and replicates with host cell DNA transcribed to mRNA which can be used to make structural protins mRNA doubles as viral genome for are virions Once new virions are assembled from the structural proteins and mRNA (single stranded RNA) genome, the virions can be released to infect other cells

Meiosis

Occurs in gametocytes (germ cells) Results in 4 nonidentical sex cells (gametes) One round of replication and 2 rounds of division (versus 1 round of replication and one round of division in mitosis) Meiosis 1: Homologous chromosomes being separated into haploid daughter cells (Homologous chromosomes align on opposite sides of the metaphase plate) (reductional division) Meiosis 2: Seperation of sister chromatids without a change in policy (similar to mitosis) (Equational division) 2N->N

Fundamental Concepts of Genetics

Offspring of parents who were also related by blood (consanguinity)= greater similarity in genotypes *Genotype: Combination of alleles one has at a given genetic locus All genes organized into chromosomes Alternate forms of genes = alleles (3 alleles for same gene are A, B, and O in blood) Humans have two copies of each chromosome, called homologues (except for male sex chromosome) Each gene has locus (specific place on chromosome) Hemizygous genotypes: one allele is present for a given gene, like with parts of X in males Complete dominance: One domiant and one recessive alleles for given gene Codominance: More than one dominant allele exists (AB blood) Incomplete dominance: Heterozygote expresses phenotype that is intermediate between 2 homozygous genotypes (red and white given you pink) -Neither allele is dominant

Oligodendrocytes are found in the ______________ while the Schwann cells are found in the ____________________

Oligodendrocytes are found in the central nervous system while the Schwann cells are found in the peripheral nervous system *Both synthesize myelin to support the neurons

In bacterial sepsis (overwhelming bloodstream infection), a number of capillarybeds throughout the body open simultaneously. What effect would this have on theblood pressure? Besides the risk of infection, why might sepsis be dangerous for theheart?

Opening more capillary bed (in parallel) decrease overall resistance -CO will increase in attempt to maintain constant BP Increased demand on heart tires it, leading to heart attack or drop in BP

Accessory Organs of Digestion

Originate as outgrowths of endoderm from gut tube during development Includes cell lining alimentary canal, pancreas, liver, gallbladder, salivary glands Gallbladder located beneath liver, stores and concentrates bile *Upon release of CCK, it contracts and pushes bile out into biliary tree (bile duct and pancreatic duct merge before emptying into duodenum) -Common site of cholesterol and bilirubin stone formation

Blood Osmolarity

Osmolality is a measure of how much one substance has dissolved in another substance. The greater the concentration of the substance dissolved, the higher the osmolality. Very salty water has higher osmolality than water with just a hint of salt. When your body is functioning properly, it makes specific adjustments to maintain an appropriate osmolality. For example, you may need to urinate frequently if your blood osmolality is too low. This helps your body get rid of excess water, raising the osmolality of your blood.

Osteoblasts vs osteoclasts vs osteocytes vs chondrocytes

Osteoblasts are bone cells involved in the secretion of bone matrix. Osteoclasts are large, polynucleated cells involved in bone resorption. Osteocytes are mature bone cells that eventually become surrounded by their matrix; their primary role is bone maintenance. Finally, chondrocytes are cells that secrete chondrin, an elastic matrix that makes up cartilage.

Steatorrhea is the presence of excess fat in feces and can be a response to ineffective digestion or absorption of dietary fat in the small intestine. Excessive secretion of digestive juices from which organ will most likely lead to steatorrhea? Pancreas Stomach Liver Gallbladder

Pancreatic enzymes include lipase which would favor a stool without fat. If the stomach secretes an excess amount of acid, it can inactive pancreatic enzymes including lipase. More bile synthesized and secreted by the liver would assist in fat digestion. Bile synthesized by the liver is stored by the gallbladder and increased release would help fat digestion. Parietal cells in the stomach secrete HCl which has the ability, if released in more than normal physiological quantities, to inactivate pancreatic enzymes, especially lipase. One possible cause of this is a gastrin-secreting tumor, or a gastrinoma. Another way to look at this question is that excess secretion from organs in the three incorrect answer choices would all help digestion and absorption of fat, as they involve the release of more lipase (pancreas) or bile (liver and gallbladder). Thus, (B) is correct. More lipase from the pancreas would assist in fat digestion, making (A) incorrect. (C) is incorrect because more bile synthesized and secreted by the liver would assist in fat digestion, or at the very least do nothing to hurt it. Finally, (D) is incorrect because bile synthesized by the liver is stored by the gallbladder and increased release would help fat digestion.

Which of the following is true regarding pancreatic somatostatin? A. Its secretion is increased by low blood glucose. B. It is always inhibitory. C. It is regulated by cortisol levels. D. It stimulates insulin and glucagon secretion.

Pancreatic somatostatin secretion is increased by high blood glucose or amino acid levels, leading to both decreased insulin and glucagon secretion, eliminating choices (A) and (D). Somatostatin is thus always an inhibitory hormone, confirming choice (B). The stimuli for somatostatin release include high blood glucose or amino acids, as mentioned above, as well as certain gastrointestinal hormones, but not cortisol

Order of anatomical features encountered by oxygen during inhalation: Arrange bronchioles, larynx, pharynx, trachea (divided into 3 parts)

Pharynx (Nasopharynx (behind nasal cavity), oropharynx (back of mouth), and laryngopharynx (above vocal cords), larynx, trachea, then bronchioles Alveoli after bronchi

Fetal Circulation

Placenta: Organ where nutrient, gas, and waste exchange occur Crucial that maternal and fetal blood types don't mix (they may be different) Move nutrients and waste via diffusion (aided by fetal hemoglobin (HbF)) Placental barrier serves immunity function (antibodies across placenta)(TORCHES = bad things that can cross, like HIV and rubella) Placenta = endocrine organ because it produces progesterone, estrogen and hCG Umbilical arteries: blood away from fetus to placenta (deoxygenated) Umbilical vein: Blood to fetus from placenta (oxygenated Oxygentation/gas exchange occurs at placenta *Unlike most other arteries, umbilical arteries carry deoxygenated blood with waste products. Veins different too because they carry oxygenated blood with nutrients Lungs and liver (detoxification and metabolism from moms liver) don't serve function before birth *The last structure to become fully functional is the lungs

Main Functions of Lymphocytes: Plasma Cells Memory B-cell Helper T cell Cytotoxic T cell Suppressor (regulatory) T cell Memory T cell

Plasma Cells: Form from B cells, exposed to antigen, produce antibodies (Antibodies are produced by plasma cells derived from B-lymphocytes) Memory B-cell: Form from B cells, exposed, Saved for second exposure (so it will be faster/stronger) Helper T cell: Coordinate through lymphokines and respond to antigens bound to MHC2 Cytotoxic T cell: Directly kill virally infected cells, respond to antigen bound to MHC 1 Suppressor (regulatory) T cell: Stop immune response after pathogen cleared and promote self tolerance Memory T cell: Stored for more rapid/robust response upon second exposure

T cell passes positive selection, but then inappropriately passes through negative selection. What will it be reactive towards?

Positive selection occurs when T cells in thymus are able to respond to antigen presented on MHC (Those that don't respond undergo apoptosis) Negative selection: T cells that respond to self-antigens undergo apoptosis before leaving thymus A T cell that inappropriately passes negative selection will be reactive to self antigens

Mitochondria

Power plant of the cell (Provide energy and also capable of killing cells by releasing enzymes kick starting apoptosis) 2 layers (outer and inner membrane) Outer: Barrier Inner: Arranged into numerous foldings called crystal, contains the molecules and enzymes of electron transport chain Cristae increase surface area available for electron transport chain enzymes Space inside the inner membrane is called mitochondrial matrix Pumping of protons from mitochondrial matrix to inter membrane space establishes proton-motive force (Ultimately, these protons flow through ATP synthase to generate ATP during oxidative phosphorylation) Mitochondria are semi-autonomous (contain their own genes and replicate independently of the nucleus via binary fission) MITOCHONDRIAL DNA IS CIRCULAR AND SELF-REPLCIATING Both mDNA (mitochondrial DNA) and bacterial DNA are organized into a single circular chromosome of double-stranded DNA that can replicate during binary fission -Cytoplasmic/extranuclear inheritance (transmission of genetic material independent of nucleus) Serial Endosymbiosis: Theory that Mitochondria/chloroplasts/flagella came from engulfing prokaryotes resulting in symbiotic relationship

Liver

Produces bile, Process and synthesize nutrients (glycogenesis, storage and mobilization of fats), production of urea (ammonia converted to urea), detoxification of chemicals, synthesis of albumin (protein that maintains plasma oncotic pressure and serves as carrier for drugs, hormones), and clotting factor ) *A patient with liver failure would not be able to convert ammonia into urea and would have high concentrations of ammonia and low concentrations of urea in the blood. With decreased synthetic activity, both albumin and clotting factor concentrations would be low. Upper right quadrant of abdomen Contains 2 unique structure for communicating with digestive system: Bile ducts (connect liver with gallbladder and SI: Bile produced in liver, stored in GB, secreted into duodenum) and Hepatic portal vein (receives all nutrient rich blood draining from abdominal portion of digestive tract, processes it, and sends it to inferior VC -> right side of heart) *liver takes up extra sugar, stores glucose as glycogen, and excess fat as triacylglycerols *Liver can reverse process: Produces glucose through glycogenolysis and gluconeogensis and mobilizing fats in lipoproteins -Detoxifies endogenous (made in body) and exogenous (brought in compounds) -Liver also detoxifies/metabolizes alcohol/medications Cirrhosis = scarring of liver = increased resistence in portal vein (portal hypertension -> swollen veins, hematemesis, blood problems, ammonia buildup) *Jaundice from bilirubin buildup

Adrenal Medulla

Produces epinephrine and norepinephrine (catecholamines) *Epi can increase glycogenolysis (breakdown of glycogen to glucose) and increase basal metabolic rate *Both increase HR, dilate bronchi, increased blood flow to muscles, heart lungs and brain but decreased to gut, kidneys, skin

Protein synthesis occurs in ___________ and __________________. Lipid and carbohydrate breakdown takes place in the _____________ and _______________. ATP production occurs in the ______________

Protein synthesis occurs in ribosomes and rough endoplasmic reticulum. Lipid and carbohydrate breakdown takes place in the peroxisomes and cytoplasm. ATP production occurs in the mitochondria

Saliva can not break down:

Proteins (but it can break down lipids and polysaccharides) Saliva contains peptidases, amlyase, and lipase, among other enzymes. Saliva contains salivary amylase, also known as ptyalin, and lipase. Salivary amylase is capable of hydrolyzing starch into smaller sugars (maltose and dextrins), while lipase catalyzes the hydrolysis of lipids. The amount of chemical digestion that occurs in the mouth is minimal, though, because the food does not stay in the mouth for long. The oral cavity does not have any enzymes capable of digesting proteins, *The duodenum can digest pretty much everything

Blood Antigens

RBCs express surface proteins called antigens (target to which immune system reacts) -Two major families of antigens = ABO antigens and Rh factor A and B are codominant (One can be expressed on each chromosome, type AB) O allele is recessive (homozygous) -Expresses neither antigen variant, so it will not cause any immune response (Universal donor: Wont cause hemolysis) -O- blood are considered universal donors because their blood cells contain no surface antigens. Therefore, O- blood can be given to anyone without causing potentially life-threatening consequences from ABO incompatibility. -Produces Anti A and Anti B antibodies, so can only receive from O Type AB: Universal recipient: No blood antigen is foreign Type A blood will recognize type A protein as self but type B protein as foreign, and will make antibodies to types B and AB Type B produces B ANTIGENS and Anti-A ANTIBODIES so it can donate to B or AB and recieve from B or O -Most common blood type is O+, least common is AB- Rh Factor: Surface protein expressed on red blood cells -Exists as several variants -Unmodified = Rh+ refers to presence of D allele (Autosommal dominant) During childbirth, women exposed to some fetal blood. If woman is Rh- and fetus is Rh+ , she will become sensitized to Rh+ and start producing antibodies ( not a problem for first child) -But, if any future child is Rh+ maternal Anti-Rh antibodies can cross placenta and attack fetal blood (Erthroblastosis fetalis) ABO blood mismatching less of a concern because AB antigens are IgM , which doesn't readily cross placenta like anti-Rh IgG antibodies Administration of RhoGAM (Rh immunoglobulin) to Rh- mother (passive immunization) absorbs fetus Rh+ cells, preventing production of anti-Rh antibodies by mother

Nephron Function

Regulate the concentration of water and soluble substances like sodium salts by filtering the blood, reabsorbing what is needed and excreting the rest as urine thereby eliminating wastes from the body, regulating blood volume and BP, controls levels of electrolytes and metabolites, and regulates blood pH. Anything that makes it into the filtrate and is not reabsorbed is lost as urine Filtrate first enter Proximal Convulated Tubule (PCT): most salt reabsorbed, but filtrate remains isotonic to the interstitiuim (connective tissue surrounding nephron) until diluting segment -Then filtrate enters descending limb of loop of Henle (only permeable to water) -> medulla -> ascending limb (only permeable to salts, takes advantage of decreasing medullary osmolarity) Diluting segment: loop of Henle becomes thicker- only part that can produce urine more dilute than blood: eliminate excess water -Volume of filtrate has been significantly reduced, showing net reabsorption of large volume of water Filtrate then enters distal convoluted tubule (DCT) -Responds to aldosterone, promotes sodium reabsorption -Concentrates urine/decreases volume Final concentration of urine in collecting duct: responsive to aldosterone and ADH/vasopressin -As permeability of collecting duct increases -> water reabsorption increases = further concentration of urine -Reabsorbed water enters interstitum -> vasa rectca -> bloodstream -Collecting duct is fairly impermeable when body hydrated -ADH and aldosterone increase reabsorption of water in collecting duct = greater water retention *ADH only governs water reabsorption and this results in lower blood osmolarity *Aldosterone causes both salt and water reabsorption and does not change blood osmolarity -No more chance for reabsorption -> filtrate goes to renal pelvis

Excretory System

Regulation of blood pressure, blood osmolarity, acid-base balance, removal of nitrogenous waste Kidneys, ureters, bladder, urethra Functions: *kidney uses osmolarity gradient and selective permeability to filter, secrete, and reabsorb *Selective elimination allows body to control BP, blood osmolarity, and acid-base balance (kidneys increase/decrease H=/bicarbonate concentration, when pH too low in blood, kidneys excrete more H+ and increase reabsorption of bicarbonate and when pH too high kidneys excrete more bicarbonate and increase reabsorption of H+ BP: ADH, constriction of afferent arteriole = les blood entering glomeruli = renin release= increased BP Osmoregulation: blood osmolarity low = excess water excreted = solutes reabsorbed in higher concentrations -blood osmolarity high = more water reabsorption and solute excretion

What are the events that initiate muscle contraction, from neurotransmitter release to myosin binding with actin

Release of ACh from motor neuron -> activation of acetylcholine receptors in sarcolemma -> depolarization of sarcolemma -> spreading of signal to T tubules -> Release of Ca2+ from Sarcoplasmic reticulum -> binding of Ca2+ to troponin -> conformational shift in tropomyosin (bound to troponin) -> Exposure to myosin binding site -> binding myosin to actin

Lungs: Thermoregulation, immune function, pH control) -Role in cardiovascular, immune, real, and nervous system

Respiratory tract is highly vascular = lots of surface area Capillaries expand = more thermal energy dissipated Capillaries contract = less blood can pass through (conserve thermal energy) -Capillaries constrict and dilate to maintain proper body temp *lack of cilia = difficulty expelling mucus from lungs (walking pneumonia) *If an area of the lung becomes filled with mucus and inflammatory cells, the area will not be able to participate in gas exchange. Because no air will enter or leave the area, the concentration gradient will no longer exist, and neither oxygen nor carbon dioxide will be able to diffuse across the alveolar wall. Immune function: *First line of defense = nasal cavity (Small hairs/vibrissae trap particles ) -Nasal cavity also contains lysozyme (found in tears/saliva): attacks peptidoglycan walls of gram positive bacteria *Mucociliary Escalator: Cilia propel mucus up respiratory tract for expulsion -Lungs (especially alveoli) also contain lots of immune cells/macrophages -Macrophages can engulf/digest pathogens and signal to body that there is an invader -Mucosal surfaces contain IgA antibody to protect against pathogens -Mast cells populate lungs (have antibodies on surface/release chemicals/can cause allergic reactions) Control of pH: *Metabolic acidosis: Excess acid caused by mechanism besides hypoventilation (treat by increasing respiratory rate) *respiratory system plays role in pH balance through bicarbonate buffer system in blood (image shown) *Body wants pH between 7.35 and 7.45 (lower pH = higher H+ concentration (acidemia: acid sensing chemoreceptors outside blood-brain barrier send signals to brain to increase respiratory rat) -Increase H+ = generate more CO2 = increase in respiratory rate = More CO2 blown off *shift to left can be caused by increase in H+ concentration, elevating CO2 or by removal of CO2, allowing H+ to drop back to normal If blood is too basic (alkalemia), respiratory rate is slowed, more CO2 retained, buffer equation shifted to right to produce more H+ ions (acid) and bicarbonate ions (base) -> lower pH *When bicarbonate ions combine with free hydrogen ions and become carbonic acid, hydrogen ions are removed, moderating pH changes. Similarly, excess carbonic acid can be converted into carbon dioxide gas and exhaled through the lungs; this prevents too many free hydrogen ions from building up in the blood and dangerously reducing its pH; *Kidneys also help with pH regulation (slower response)

Cardiac Circulation

Return deoxygenated blood to right atrium via venae cavae (divided into superior and inferior) -Travels through tricuspid valve into right ventricle, where on contraction, it passes through pulmonary valve into pulmonary arteries (Travels into lungs and breaks into smaller capillaries that line the alveoli, participates in gas exchange, with CO2 leaving blood and O2 entering blood) -Blood trails into pulmonary venules into pulmonary veins, which carry blood to left side of heart -Oxygenated blood enters from left atrium, travels through bicuspid/mitral valve, enters left ventricle, where on contraction it passes through aortic valve into aorta -From aorta, it foes into arteries, then arterioles, then capillaries. Gas and nutrient exchange occur, then blood enter venues, which become larger veins. -Veins empty into either the SVC or IVC to return to right side of heart Summary: Right atrium to tricuspid valve to right ventricle to pulmonary valve to pulmonary artery to lungs to pulmonary veins to left atrium to mitral valve to left ventricle to aortic valve to aorta to arteries to arterioles to capillaries to venules to veins to venae cavae to right atrium Valves: Anogram to TPMA

Pathway of sperm through the male reproductive system

SEVEN UP: Seminiferous tubules Epididymis Vas Deferens Ejacultaroy duct Nothing Urethra Penis Primitive gonads develop into testes -located in the scrotum (external pouch below penis)(Cooler temperature for sperm development) -composed of seminiferous tubules (where sperm are produced and nourished by Sertoli cells) and interstitial cells of leydig (Secrete testosterone/androgens) Seminiferous tubules Epididymis (where flagella gains motility and are stored until ejaculation) Vas Deferens (aka dusts deferens)(sperm pass through here during ejaculation) Ejacultaroy duct (two ejaculatroy ducts fuse at prostate to form urethra) Nothing Urethra Penis -Sperm mixed with seminal fluid made by seminal vesicles (fructose for sperm), prostate gland (both prostate and seminal vesicles give fluid mild alkaline properties so sperm can survive acidity of female), and bulbourethral (Cowper's) glands (produce fluid that cleans out tracts and lubricates) -Combo of sperm and seminal fluid = semen *In males, reproductive and urinary systems share common pathway

Gross Structure of Myocytes

Sarcomeres are attached end-to-end to form myofibrils -Myofibrils are surrounded by sarcoplasmic reticulum (SR) = specialized Endoplasmic reticulum for muscle cells with high concentration of Ca2+ cells -Outside SR = sarcoplasm (modified cytoplasm) -Cell membrane of myocyte = sarcolemma (capable of propagating action potential/ distribute it to all sarcomeres in muscle using transverse tubules (T-tubules) that are oriented perpendicularly to form myofibrils) Each myocyte (muscle cell) contains many myofibrils in parallel = Also called muscle fiber (Many nuclei are in periphery of cell) -Muscle composed of parallel myocytes (Muscle fibers), which are composed of parallel myofibrils -Many sarcomeres in series = myofibril. Many myofibril arranged in parallel = muscle fiber/myocyte/muscle cell. Many muscle fibers arranged in parallel = muscle

A developing T lymphocyte, also called a thymocyte, is LEAST likely to be prevented from entering the circulation and becoming activated if its cell-surface receptor recognizes:

Self MHC and non-self antigen A thymocyte that recognizes a self-MHC should be able to become activated, given that MHC molecules are just carriers for the antigens themselves. If a thymocyte is not activated by a self-antigen, it is not selected against and can enter the circulation. T-cells mature in the thymus, where they undergo both positive and negative selection. Positive selection refers to maturing only cells that can respond to the presentation of antigen on MHC (cells that cannot respond to MHC undergo apoptosis because they will not be able to respond in the periphery). Negative selection refers to causing apoptosis in cells that are self-reactive (activated by proteins produced by the organism itself). The maturation of T-cells is facilitated by thymosin, a peptide hormone secreted by thymic cells. Once the T-cell has left the thymus, it is mature but naïve. Upon exposure to antigen, T-cells will also undergo clonal selection so that only those with the highest affnity for a given antigen proliferate.

Recognition of Self and Non-self

Self antigens: Proteins and carbs present on surface of every cell (Not foreign) Autoimmunity: Body attacks self antigen (MS, myasthenia graves, psoriasis, lupus, rheumatoid arthritis, Graves disease, Guillian-Barre syndrome) -Most autoimmune disease can be treated with glucocorticoids (modified cortisol), which has immunosuppressive qualities Another problem: Immune system identifies foreign antigen as dangerous when it is not: Allergic Reaction Hypersensitivty reactions = Autoimmunity + allergy *Type1 diabetes = autoimune destruction of c cells in pancreas = can't produce insulin

The anatomical pathway for sperm during development proceeds from the:

Seminiferous tubules to the epididymis

Functions of the Nervous System

Sensation and perception Motor function Cognition (thinking) and problem solving Executive function and planning Language comprehension and creation Memory Emotion and emotional expression Balance and coordiantion Reulsation of endocrine organs Regulation of heart rate, breathing rate, vascular resistance, temperature, and exocrine glans Divided into central and peripheral NS Central divided into brain and spinal cord Peripheral divided into somatic and autonomic Autonomic divided into sympathetic and parasympathetic

Three kinds of nerve cells in the nervous system

Sensory neurons (Also known as afferent neurons): Transmit sensory information from sensory receptors to the spinal cord and brain -afferent: Away from the source, towards the brain -Efferent: Exiting the brain, towards the source Motor neurons (AKA efferent neurons): transmit motor information from he brain and spinal cord to muscles and glands interneurons: Found between other neurons -Most numerous -Located predominantly in brain and spinal cord -Linked to reflexive behavior -Reflexes only require processing at the spinal cord (if scenario requires input from brain or brainstem = supra spinal circuits)

Which of the following accurately describes sensory neurons? A. Sensory neurons are afferent and enter the spinal cord on the dorsal side. B. Sensory neurons are efferent and enter the spinal cord on the dorsal side. C. Sensory neurons are afferent and enter the spinal cord on the ventral side. D. Sensory neurons are efferent and enter the spinal cord on the ventral side.

Sensory neurons are considered afferent (carrying signals from the periphery to the central nervous system) and enter the spinal cord on the dorsal side. Motor neurons are considered efferent (carrying signals from the central nervous system to the periphery) and exit the spinal cord on the ventral side.

Complement System (innate immunity)

Set of proteins in blood that can create holes in bacteria Can not be modified to regret specific organisms Can be activated through classical pathway (requires binding antibody to pathogen) or by alternative pathway (does not require antibody) Complement proteins punch holes in cell walls (osmotically unstable)

Hormones

Signaling molecules that are secreted directly into the bloodstream to distant target tissues -Bind to receptors, inducing change in gene expression or cellular functioinng -Can be classified by their chemical structure: Peptides, steroids, or amino acid derivatives

Nucleotide-Level Mutations

Single nucleotide or small number Point mutations occur when 1 nucleotide in DNA (A, C, T, or G) swapped for another -Can be silent, missense, or nonsense: -Silent: Nucleotide has no effect on final protein *Most commonly occurs when changed nucleotide is transcribed to be the third nucleotide (degeneracy/wobble in genetic code) -Missense: Change in nucleotide results in substitution of 1 amino acid for another in final protein -Nonsense: Change in nucleotide results in substituting a stop codon for an amino acid in final protein Frameshift Mutation: Nucleotides inserted or deleted from genome (insertion or deletion mutations) -mRNA transcribed from DNA always read in 3 letter code -> shifts reading frame

Negative Sense VS Positive Sense RNA Which of the following contains an RNA replicase enzyme? Retrovirus, DNA virus, negative sense RNA, or positive sense RNA

Single stranded RNA viruses may be positive sense (genome may be directly translated to functional proteins by the ribosomes of host cell like mRNA) or negative sense (RNA acts as template for synthesis of a complement strand which can be used as template for protein synthesis) -Negative sense RNA viruses carry an RNA replicase in the vision to ensure that the complementary strand is synthesized (Require synthesis of complementary RNA strand via ENA replicase which can be translated to form proteins) viral genomes may be made of either DNA or RNA, and may be single- or double-stranded in both cases. DNA viruses first need to be transcribed and then translated, but do not require RNA replicase in any capacity (A). Retroviruses (B) are enveloped, single-stranded RNA viruses in the family Retroviridae; usually, the virion contains two identical RNA molecules. These viruses carry an enzyme known as reverse transcriptase, which synthesizes DNA from single- stranded RNA. Single-stranded RNA viruses may be positive sense or negative sense. Positive sense implies that the genome may be directly translated to functional proteins by the ribosomes of the host cell, just like mRNA (D). Negative-sense RNA viruses are a bit more complicated: these viruses require synthesis of an RNA strand complementary to the negative-sense RNA strand, which can then be used as a template for protein synthesis. Negative-sense RNA viruses must carry an RNA replicase in the virion to ensure that the complementary strand is synthesized. Therefore, (C) is correct.

Archaea

Single-celled, no nucleus, Live in harsh conditions (Extremophiles) Visually similar to bacteria and contain a single, circular chromosome and divide by binary fission/budding but contain genes/metabolic pathways more similar to eukaryotes (both start translation with methionine, contain similar RNA polymerases, and associate their DNA with histones) Can use alternative sources of energy: Photosynthetic, chemosynthetic (Can generate energy from inorganic compounds, like sulfur and nitrogen based products (ammonia) -Hypothesized that archaea and eukaryotes share common ancestor

Virus

Size: 20 nm-300 nm Lack organelles and nucleus Composed of genetic material, protein coat (Capsid), and sometimes an envelope containing lipids/phospholipids and virus-specific proteins Envelope is sensitive, so enveloped viruses are easier to kill Obligate intracellular parasites: Must express and replicate genetic information within a host cell because they lack ribosomes to carry out protein synthesis (Uses cell to replicate and produce viral progeny called virions) Genetic information can be circular or linear, single or double stranded, and composed of DNA or RNA

Phrenic Nerve The phrenic nerve originates in the neck and innervates the diaphragm. An occasional complication of heart surgery is damage to the phrenic nerve. Which of the following best describes the phrenic nerve and the effect of damage to it?

Somatic Nerve, difficulty with inspiration (breathing in) Inhalation is an active process. We use our diaphragm as well as the external intercostal muscles (one of the layers of muscles between the ribs) to expand the thoracic cavity (diaphragm actively contracts). As the diaphragm flattens and the chest wall expands outward, the intrathoracic volume (the volume of the chest cavity) increases. Specifically, because the intrapleural space most closely abuts the chest wall, its volume increases first. Unlike inhalation, exhalation does not have to be an active process. Simple relaxation of the external intercostal muscles will reverse the processes we discussed in the last paragraph. As the diaphragm and external intercostals relax, the chest cavity decreases in volume. The diaphragm is under somatic control, even though breathing itself is under autonomic control.

Erythrocytes (RBC)

Specialized cell for O2 transport -O2 is non-polar, low solubility in aqueous environment Contains 250 million molecules of hemoglobin (each can bind 4 O2 molecules) (Each RBC can carry 1 billion O2 molecules) O2 and nutrients delivered to peripheral tissues, waste and CO2 picked up and delivered to lungs/liver/kidneys Erythrocytes are phagocytized in the spleen and liver after a certain period of time. RBC are biconcave (indented on both sides): Increases surface area/ gas exchange Lose nuclei, mitochondria, and other membrane bound stuff when they mature (Does not consume O2 before delivering it) -Do not carry out oxidative phosphorylation to generate ATP, use glycolysis for ATP (Lactic acid as main by-product) -Unable to divide (live for 120 days) because they dont have nuclei Hematocrit: measure of how much of blood sample consists of RBC (12-17.5 g/dL or 36-53%) -3.5-6 million per cubic millimeter of blood: Transport O2 and help transport CO2

Neurons

Specialized cells capable of transmitting electrical impulses and translating impulses into chemical signals Nucleus located cell body/soma Soma = location of ER and RIBOSOMES Dendrites: recieve impulses from other cells (transmitted to axon hillock, which plays important role in action potentials (transmission fo electrical impulses down the axon)) Axon: Long appendage that terminates in close proximity to target structure (Axons carry neural signals away from soma, dendrites carry signals toward soma) Most nerve fibers insulated in fatty membrane, myelin, to prevent signal loss/crossing signals / maintains elctrical signal within neuron/Increases speed of conduction Myelin produced by oligodendrocytes in CNS and Schwann cells in PNS -Demyelinating disorder/body attacks myelin = MS Nodes of Ranvier: Small breaks in myelin sheath with exposed axon membrane (critical for rapid signal conduction) Nevre terminal/synaptic bouton (knob): Enlarged and flattened to maximize transmission of signal to next neuron to ensure proper release of neurotransmitters (Chemicals that transmit info between neurons) Neurons aren't physically connected - small space (synaptic cleft) into which axon terminal portions release neurotransmitters that bind to dendrite of postsynaptic neuron Nerve terminal + synaptic cleft + post synaptic membrane = synapse

Speciation

Species: the largest group of organisms capable of breeding to form fertile offspring Formation of new species through evolution = speciation Isolation: Progeny of isolation of same species populations can no longer freely interbreed now considered separate species Reproductive isolation may occur either prezygotically or postzygotically -Prezygotic mechanisms: prevent formation of zygote completely (temporal isolation (breeding at different times), ecological isolation (living in different niches in same territory), behavioral isolation (lack of attraction), reproductive isolation (incompatibility of reproductive anatomy), or gametic isolation (intercourse can occur, but fertilization cant) -Postzygotic: Allow for gamete fusion, but yields nonviable or sterile offspring (hybrid inviability (formation of zygote that cant develop to term), hybrid sterility (forming hybrid offspring that cant reproduce = mule), and hybrid breakdown (forming first generation hybrid offspring that are viable and fertile, but second generation hybrid offspring are inviable/infertile)

Cell Specialization

Specification (cell reversibly designated as specific cell type, can still change mind), determination (commitment of cell to function in future, irreversibly committed), and differentiation (assumes structure, function, and biochemistry) Morphogens (molecules that cause differentiation/cause cells to proceed in a specific development, like Sonic HH, EGF, TGF-Beta) play role in determination Cells that haven't differentiated = stem cells Potency: Greatest potency = totipotent (can become anything), then pluripotent, then multipoint (stem cells found in adults, can differentiate into a few types) ES Stem cells retain pleuripotency (can form any cell type in body, just not placental structures)

Steroid Hormones vs Peptide hormones

Steroid: Made of lipids Made of 4 cholesterol rings Lipophilic but hydrophobic= freely diffuse past plasma membrane into cytoplasm Bind to intracellular receptors of target cells Primary signaling -> go directly inside the cell -Cortisol, aldosterone, progesterone -Because blood is water-based, lipid-derived hormones must travel to their target cell bound to a transport protein. Peptide: Example = insulin Bind to receptor on the surface of target cells Hydrophilic: Cant pass through plasma membrane, so they need secondary messengers to go into the cell since they can't (but, being hydrophilic lets them travel freely) -Includes tyrosine derivatives (like catecholamines, produced in adrenal medulla) Plasma membrane is bilayer with hydrophilic (polar) heads and LARGE region with hydrophobic tails *Small, non-polar molecules pass through membrane much quicker than small, polar molecules because polar molecules move through really slow because hydrophobic tails don't want to help hydrophilic molecules get by (but because they are small, they kinda sneak by) Large, non-polar can pass through, like benzene, but slow because even though it is non-polar (fast), it's big which makes it hard to push through Any charged molecule, like many amino acids, can't pass through because something charged means that it is very polar, and the hydrophobic tails want nothing to do with that

Fredrick Griffith Experiment (1928)

Studied bacteria that caused pneumonia 2 strains: Virulent (disease causing = killed mice: Smooth capsule = better able to evade immune system) and nonvirulent (no capsule = didn't kill mice) Killed virulent bacteria and injected it into mice = no death Combined dead virulent and living non virulent and injected into mice and mice died = Transforming principle Demonstrated transforming principle

Epidermis

Subdivided into 5 strata stratum basale (Deepest): Contains stem cells, responsible for proliferation of keratinocytes (predominant cells of skin) that produce keratin stratum spinosum (Squamous cell layer): Cells become connected, site of Langerhans cells stratum granulosum (Granular cell layer): Keratinocytes die and lose nuclei stratum lucidum: Only present in thick, hairless skin (soles of feet, palms), nearly transparent stratum corneum (Shallowest)(Horny cell layer): Up to several dozen layers of flattened keratinocytes, forming barrier Top to bottom: CLGSB Other cells of epidermis: -Calluses: Excessive keratin due to friction -Melanocytes: Dervied from neural crest cells and found in stratum basale, produce melanin (Pigment serves to protect skin from DNA damage by UV) -Langerhans cells: special macrophages in stratum spinosum, present antigens to T cells to activate immune system

For each of the following substances, determine whether it is a digestive enzyme of a hormone and briefly summarize its functions. Sucrase Secretin Dipeptidase Cholecystokinin (CCK) Enteropeptidase

Sucrase: Brush border Enzyme, digests sucrose into monosaccharides found on duodenal cells -hydrolyzes sucrose (a disaccharide) to form glucose and fructose (monosaccharides). Secretin: Hormone,increases pancreatic enzymes released into duodenum, regulates pH by reducing HCl and increasing bicarbonate, slows/decreases motility through tract (enterogastrone) Dipeptidase: Brush border Enzyme, breaks down dipeptides into free amino acids Cholecystokinin: Hormone, responds to chyme secretion, recruits secretions from gallbladder and pancreas, stimulates release of bile and pancreatic juices, promotes satiety Enteropeptidase: Enzyme: Activates trypsinogen, initiates activation cascade, Increases release of secretin and CCK *Brush Border enzyme: Group of enzymes present on luminal surface of cells lining the duodenum that break down larger biomolecules into monomers that are able to be absorbed *Does not require enteropeptidase

Summation

Summation: Additive effect of multiple signals Two types: Temporal: Multiple signals are integrated during relatively short period of time Spatial Summation: Additive effects based on number and location

Anterior Pituitary (FLAT PEG)

Synthesizes and secretes 7 different products (4 tropic (Cause release of another hormone at organ level) (FLAT) and 3 direct (PEG)) Follicle-stimulating hormone (FSH): Promotes development go ovarian follicles and spermatogenesis Luteinizing hormone (LH): promotes ovulation and testosterone production *LH and FSH release stimulated by GnRH, both act on gonads Adrenocorticotropic hormone (ACTH): Promotes synthesis and release of glucorticoids from adrenal cortex Thyroid stimulating hormone (TSH): Promotes synthesis and release of Triiodothronine and thyroxine Prolactin (unusual. because release of dopamine from hypothalamus decreases its secretion): Milk production Endorphins (Decrease perception of pain) Growth Hormone (GH) (Requires large quantities of glucose) (Stimulates breakdown of fatty acids) (stimulated by GHRH)(Excess GH= gigantism/acromegaly, deficit = dwarfism),

Problems in Early Development

Teratogens: Substances that interfere with development -Alcohol, drugs, viruses, environmental chemicals (polycyclic aromatic hydrocarbon) Too much sugar exposure is bad Folic acid deficiency = no complete closure of neural tube (Anencephaly) Failure of neural tube to close = spina bifida

Gonads

Testes secrete testosterone in response to LH and FSH -Sexual differentiation/development -Secondary sex characteristics Ovaries secrete estrogen and progesterone

What are the two circulatory vessels in a virus? What biomolecules are absorbed by each?

The 2 circulatory vessels are CAPILLARIES and LACTEALS. The capillary absorbs water-soluble nutrients like monosaccharides. amino acids, short-chain fatty acids, water-soluble vitamins, and water itself. The lacteal absorbs fat-soluble nutrients, like fats, cholesterol, and fat soluble molecules *In each villi there is a capillary bed for absorption of water soluble nutrients and a lacteal (lymphatic channel that takes up fats) *Lacteals (small lymphatic vessels) located at villus center in small intestine- Fats packaged into chylomicrons by intestinal mucosal cells enter the lacteal for transport *Lymphatic fluid with lots of chylomicrons = chyle (milky white appearance) *transports chylomicrons to the circulatory system. *Villi = small, finger-like projections that extend into the lumen of the small intestine. chorionic villi (penetrate endometrium, support fetal gas exchange)

Meiosis 1 vs Mitosis

The chromosome number is halved (reductional division) in meiosis 1, and the daughter cells have the haploid number of chromosomes (23 in humans) Meiosis 2 is similar to mitosis in that sister chromatids are separated from one another, so no change in policy is observed In mitosis, no crossing over occurs and homologous chromosomes do not pair. Goes from 2N to 2N and occurs in all diving cells Meiosis: Crossing over, homologous chromosomes align on opposite sides of metaphase plate, only in sex cells (2N to N) Synapsis and crossing over occurs during prophase I, and homologous chromosomes are separated during meiosis I (rather than sister chromatids, as in mitosis). During metaphase of mitosis, centromeres are present directly on the metaphase plate; during metaphase of meiosis I, there are no centromeres on the metaphase plate.

Exposure of which subendothelial compounds start the coagulation cascade? What protein helps stabilize the clot?

The coagulation cascade can be started by exposure of collagen and tissue factor to platelets and coagulation factors. Clot stabilized by fibrin Fibrin: The insoluble protein that forms bulk of blood clot

What is the chemical equation for the bicarbonate buffer system? What enzyme catalyzes this reaction?

The combining of Carbon dioxide and water is catalyzed by carbonic anhydrase

Primary Germ Layers

The ectoderm, mesoderm, and endoderm; the three tissue layers of an early embryo from which all later tissues and organs arise. Ectoderm: outer layer, integument (gives rise to skin/epidermis, hair, nails, epithelia of nose, mouth, lower anal canal, eye lens, nervous system (adrenal medulla), inner ear) -NERVOUS SYSTEM AND SKIN Mesoderm: Musculoskeletal, circulatory, excretory, gonas, connective tissue, layers of digestive and respiratory systems, adrenal cortex, urinary system, kidneys, bones, muscle Endoderm: Gives rise to linings of alimentary canal and of digestive and respiratory organs: epithelial linings of digestive and respiratory tracts, pancreas, thyroid, bladder, urinary tracts, liver *Induction: Ability of one group of cells to influence the fate of nearby cells (mediated by chemical substances) -Embryonic cell that undergoes induction is called responder

Neuralation

The formatino of the nervous system during weeks 5-8 of gestation. Neuralation begins when a section of the ectoderm invaginates rod of mesodermal cells (notochord) forms primitive spine While the neural tube forms from ectoderm, the notochord itself is mesodermal (Notochord forms mesoderm) Pinches off to form the neural groove, which ultimately forms the neural tube, from which the brain and spinal cord develop (CNS). At the tip of each neural fold = neural crest cells (form PNS)

Approximately 40% of somatotroph (growth hormone) cell adenomas are the result of mutations in the GNAS gene affecting GTPase activity. Given this information, which of the following is true regarding somatotroph cells? The mutation in the GNAS gene has no effect on the second messenger response in somatotroph cells Somatotroph cells secrete a hormone that binds to an intracellular receptor The hormone secreted by somatotroph cells is water soluble and can travel freely in the bloodstream The first messenger used in the signaling cascade in somatotroph cells is cyclic adenosine monophosphate

The hormone secreted by somatotroph cells is water soluble and can travel freely in the bloodstream Somatrophs release somatotropin (growth hormone), which is a peptide hormone. Peptide hormones are water soluble, so they can travel freely in the bloodstream. The question indicates that somatotrophs are cells that secrete growth hormone. Growth hormone is also known as somatotropin and the -in ending indicates it is peptide hormone. Peptide hormones cannot pass through the membrane and bind to an intracellular receptor, indicating that (B) is wrong. Both (A) and (D) are incorrect because of how the second messenger system works. GTPase activity is important to second messenger systems because G-protein coupled receptors play a role in releasing second messengers. First messengers will be the hormone that is acting on a cell, not cyclic adenosine monophosphate (cAMP) a common secondary messenger. Lastly, peptide hormones are water-soluble and therefore can travel freely in the blood stream. This indicates that (C) is correct.

Large Intestine

The last section of the GI tract, where water is absorbed and the remaining material is eliminated from the body -larger diameter but shorter length than SI -Divided into 3 sections: Cecum, colon, rectum Cecum: Accepts fluid from SI through ileocecal valve, site of attachment to appendix (small, fingerlike projection once thought to be vestigial (Organ/limb that has no apparent function but was probably functional at some point in past, may have role in gut bacteria) *The ileocecal valve lies between the ileum of the small intestine and the cecum of the large intestine. Colon: Divided into ascending, transverse, descending, and sigmoid - Absorbs water and salts (but SI actually absorbs more water) and from feces Rectum: Storage site for feces -Anus: Opening with 2 sphincters Urine and feces Expulsion: both have internal sphincter under autonomic control and external sphincter under somatic/voluntary control *Symbiotic relationship: Bacteria in gut produce vitamin K (clotting factors) and B7 (biotin, metabolic enzyme)

The speed at which action potentials move depends on:

The length and cross-sectional area of the axon Increased length = higher resistance slow conduction Greater cross sectional area allows for faster propagation due to decreased resistance The effect of cross sectional area is more significant than the effect of length Myelin maximized speed of transmission (good insulator) - Only permeable to ion movement at nodes of ranvier -Signal hops from node to node (called saltatory conduction) *Note: All action potentials within the same type of neuron have the same potential difference during depolarization. Increased intensity of stimulus does not result in increased potential difference of action potential, but rather increased frequency of firing

Vagus Nerve

The tenth cranial nerve; it innervates the palate, pharynx, larynx, heart, lungs, and abdominal viscera; responsible for maintaining homeostatic activity through the parasympathetic response

Natural Selection (Proposed by Darwin in 1859)(Same as Survival of the fittest)

Theory that certain characteristics/traits possessed by individuals within a species may help those individuals have greater reproductive success, thus passing on traits to offspring Theory built on several tenets: *Organisms produce offspring, few of which survive to reproduce *Chance variation within individuals in population may be heritable. If these variations give an organism advantage, its favorable *Individuals with more of the favorable variations are more likely to survive to reproduce; leads to increase in these traits in future generations (fitness: organisms fitness directly related to relative genetic contribution of individual to next generation) *Natural selection is not same as evolution, natural selection is simply a mechanism for evolution Mostly correct Modern Theories: -Modern synthesis model (Neo-darwinism): *Accounts fro mutation and recombination as mechanisms of variation and considers differential reproduction to be the mechanism for reproductive success Updated Darwin theory -> when mutation or recombination results in a change that is favorable, that change is more likely to be passed on, but the opposite is also true (differential reproduction) *Populations evolve, not individuals -Selection is for specific allele, which are passed on through formation of gametes; alleles for these favorable traits arise from mutations Modern theories shift to focus on inclusive fitness: Measure of an organisms success based on number of offspring, success of offspring, and ability of those offspring to support others (Darwin had just focused on number of offspring) (inclusive fitness promotes idea that altruistic behavior can improve fitness and success) -Eldredge and Gould: Punctuated equilibrium : Suggests that changes in some species occur in rapid bursts rather than evenly over time (based on fossils)

Genetic Recombination of Bacteria: Transformation, conjugation, and Transduction

Transformation: Involves uptake of "naked" DNA through the cell membrane Integration of foreign genetic material into host genome Does not require a virus Mostly comes from other bacteria lysing (spilling contents) into vicinity of bacterium capable of transformation Common for gram-negative rods Conjugation: Analagous to sexual reproduction in eukaryotes Requires sex pili on donor male (+) to recipient female (-) (physical bridge (conjugation bridge) between cells, rather than viral vector) -Donor male must contain sex factors to form plus (like F (fertility) factor in E.Coli -Allows for rapid acquisition of antibiotic resistance/virulanece factors -Sex factor is plasmid but can become integrated into host genome through transformation -Bridge usually breaks before entire copy of genome passed to recipient -Hfr = High frequency of recombination Transduction: Only genetic process that requires a vector (virus that carries genetic material from one bacterium to another) Transfer of genes from one bacterium to another using a viral vector -BACTERIOPHAGES: Viruses that infect bacteria -Can incorporate a segment of host DNA during assembly (Release trapped DNA into new host cell)

Prions and Viroids

Very small (subviral) particles that can cause disease under certain circumstances Prions: Nonliving, infectious proteins -Cause disease by triggering misfolding of other proteins, usually though the conversion of proteins from an alpha-helix structure to a beta-pleated sheet -Reduces solubility of protein/as well as ability of cell to degrade the protein (Eventually protein aggregates from, interfering with cell function) -Mad cow disease Viroids: Small pathogens consisting of very short circular single-stranded RNA that infect plants -Can bind to large number of RNA sequences and can silence genes in plant genome (prevents necessary protein synthesis, resulting in metabolic/ structural damaged cell -Mostly plant pathogens, can be in humans (Hepatitis B virus and hepatitis D virus)

Viral (Intracellular Pathogen) Infections

Virally infected cell will begin to produce interferons = reduce the permeability of nearby cells = decrease ability of virus to infect cell, reduce the rate of transcription and translation in these cells (decreasing ability of virus to multiply), and cause systemic symptoms (aching, fever, etc) Infected cells present intracellular proteins on their surface in conjunction with MHCI in a virally infected cell. CD8+ T cells will recognize the MHCI and antigen complex as foreign and will inject toxins into the cell to promote apoptosis. -Viruses can down regulate production/presetnation of MHC1, so NK cells (recognizing absence of MHC1) cause apoptosis *T-lymphocytes, which mature in the thymus, are the only specific defense against intracellular pathogens

When a sensory neuron receives a stimulus that brings it to threshold, it will do all of the following EXCEPT: A. become depolarized. B. transduce the stimulus to an action potential. C. inhibit the spread of the action potential to other sensory neurons. D. cause the release of neurotransmitters onto cells in the central nervous system.

When a sensory neuron receives a signal that is strong enough to bring it to threshold, one can assume that the receptor becomes depolarized, allowing it to transduce the stimulus to an action potential. The action potential will then be carried by sensory neurons to the central nervous system, where the cell will release neurotransmitters. Therefore, among the given choices, the only incorrect statement is found in choice (C). If a receptor is stimulated, it will promote the spread of the action potential to postsynaptic sensory neurons in the spinal cord, which can send the signal toward the brain.

Antagonistic pairs of muscles

When one contracts the other relaxes -Arm: bicep brachii contracts and triceps brachii relax = elbow flexed -tricep contracts and bicep relax = elbow extended -Contraction of one causes other to elongate Muscles can also be synergistic: Work together

Adenine Adenosine Triphosphate (ATP) Adipose

a purine base present in DNA and RNA; it forms hydrogen bonds with thymine and uracil A nucleotide molecule consisting of adenine, ribose, and 3 phosphate moieties; outer 2 phosphates are bound by high energy bonds Refers to fats tissue, fat storing tissue, fat within cells

Zymogen

an inactive enzyme precursor that is converted into an active enzyme

Binary Fission

asexual reproduction in bacteria Circular chromosome attaches to cell wall and replicates while cell continues to grow, eventually plasma membrane and cell wall grow inward and produce to identical daughter cells Requires fewer events than mitosis, can proceed faster

Thrombocytes (Platelets)

cell fragments from megakaryocytes in bone marrow Assists in blood clotting 150,000-400,000 per microliter of blood

Prophase 1

chromatin condenses into chromosomes. Homologous chromes -> crossing over (exchange genetic material) Spindle apparatus forms Nucleoli and nuclear membrane disappear Homologous chromosomes come together and intertwine in SYNAPSIS Each chromosome has two sister chromatids, and each pair of four chromatids = tetrad Homologous chromosomes held together by synaptonemal complex Chiasma: Homologs chromosomes point of contact -Exchange pieces of DNA (called crossing over) *Crossing over occurs between homologous chromosomes, not sister chromatids (Sister chromatids are identical, so crossing over wouldn't do anything)

Early Developmental Stages

cleavage, implantation, gastrulation, neurulation All embryos female by default: SRY gene makes male at 6-8 weeks post fertilization When sperm meets secondary oocyte in Fallopian tubes, binds oocyte and releases enzymes let sperm head penetrate corona radiata and zone pellucida Forms acrosomal apparatus After penetration through cell membrane, cortical reaction (release of calcium ions by ovum) occurs -> results in fertilization membrane Depolarize ovum membrane, prevents fertilization by multiple sperm and increases metabolic rate of diploid zygote (now impenetrable and depolarized): Called fertilization membrane Cleavage (Rapid mitotic division): Zygote travels to uterus for implantation First cleavage = creates embryo (nullifies unicellularity, total size unchanged, increase ratios of nuclear to cytoplasmic ratio and surface area to volume ratio) Two types of cleavage: Indeterminate (cell can still develop into complete organism) and determinate (Fates are already determined, committed to differentiating into certain type) Blastulation: Blastula: The early embryonic stage during which embryo is a hollow, fluid filled sphere of undifferentiated cells Embryo becomes solid mass of cells (morula), undergoes blastulation to form mammalian blastula (called blastocyst) -Blastocyst: A mammalian blastula consisting of the trophoblastic cells and an inner cell mass: hollow ball of cells with fluid filled inner cavity known as blastocoel Blastocoel: the fluid-filled central cavity of the blastula -Contains trophoblast (Embryonic cells that line the blastocoel and give rise to the chorion and the placenta/Outer circle) and inner cell mass (gives rise to organism) Implantation: Blastula moves to uterus, burrows in endometrium Trophoblast cells give rise to chorion (extraemryonic membrane that becomes placenta), form chorionic villi (penetrate endometrium, support fetal gas exchange) Embryo connected to placenta by umbilical cord (2 arteries (Carries deoxygenated blood and waste to placenta for exchange), one vein (carries oxygenated blood)) Embryo supported by yolk sac (site of early blood cell development) Gastrulation: Generation of three distinct cell layers Results in gastrula Membrane invagination into the blastocoel is called the archenteron (develops into gut) (opening called blastopore, develops into anus in deuterostomes (humans) and mouth in protostomes) *Blastopore: Opening of the archenteron to the external environment in the gastrula stage of embroynic development *Archenteron: Central cavity in gastrula stage of embryonic development (lined by endoderm and ultimately gives rise to the adult digestive tract)

Opsonization

coating antigen with antibody enhances phagocytosis (marked with antibody from B cell) antibodies attract other WBCs to phagocytize

Hershey-Chase Experiment (1952) One year before Watson-Crick Model

confirmed that DNA is the genetic material/ can independently carry genetic information because only radiolabeled DNA could be found in bacteriophage-infected bacteria -Cretaed bacteriophages with radiolabled DNA and protein (one with sulfur, found in protein but not DNA) -Another group contained radio labeled phosphorus (found in DNA not protein) -BActeriophages infected bacterium, injected material and left capsid outside -Sampled centrifuged material to separate material that remained outside cell from bacterial cells themselves -No radio labeled protein entered cells, but radiolabled DNA had *Used radio labeling sulfur to tag proteins because 2 amino acids (cysteine and methionine) contain sulfur in R group, while no nucleotides contain any sulfur

Foramen Ovale

connects the two atria in the fetal heart Moves blood from right atrium to the left atrium Shunt within fetal heart between right and left atria that allows circulation to largely bypass the developing lungs

Small Intestine

consists of duodenum, jejunum, and ileum Up to 7 meters long Duodenum responsible for most chemical digestion, absorption takes place in jejunum and ileum (large intestines mean function is to absorb water, but the SI actually absorbs larger volume) Food leaves stomach through pyloric sphincter to duodenum, where chyme causes release of BRUSH BORDER ENZYMES (break stuff into monomers) like disaccharidases (maltase (digests maltose), lactase, etc) and peptidases (breaks down proteins/peptides like dipeptidase and aminopeptidase)(Unlike carbs, which must be broken in to monosaccharides, proteins can be di or try peptides and still be absorbed into SI wall) -Hydrolyzing disaccharide = methane gas by-product, can also have osmotic effect (pulls water in, causes diarrhea) -Duodenum also secretes enteropeptidase: activates other enzymes/organs (like Procarboxypeptidases A and B) and secretes secretin and cholecystokinin (CCK) -Activates Trypsinogen (pancreatic protease) to trypsin -Secretin: Peptide hormone, enterogastrone (hormone that slows motility= increased digestion), causes pancreatic enzymes to be released, regulates pH by reducing HCl from parietal cells and increasing bicarbonate from pancreas -CCK: Also secreted in response to entry of chyme into duodenum, promotes satiety, acts on brain, increases release of bile and pancreatic juice (mix of enzymes in bicarbonate rich alkaline solution, helps neutralize acidic chyme, brings pH to 8.5) (Pepsin in stomach works best at pH 2, pancreatic enzymes in duodenum work best at pH 8.5 Small intestine lined with villi (small fingerlike projections from epithelial lining), each with several microvilli (increasing surface area for absorption) Simple sugars (glucose, fructose, etc) and amino acids absorbed by secondary active transport and facilitated dissuasion into epithelial cells lining SI -> move across epithelial cell membrane into intestinal capillaries -Absorbed molecules go to liver via hepatic portal circulation

Rhabdomyolysis

dissolution of striated muscle (caused by trauma, extreme exertion, or drug toxicity; in severe cases renal failure can result) -Rhabdo = striation -Myo = muscle -lysis = breakdown Products of skeletal muscle destruction can be toxic, like creatine kinase, myoglobin

Four Tissue Types

epithelial, connective, muscle, nervous Epithelial: Covers body and lines cavities, provides protection against invasion and desiccation, can be involved in absorption, secretion, and sensation -Tightly joined by underlying layer: Basement membrane -Usually constitute the parenchyma (functional parts of organs, like nephrons in kidneys, hepatocytes in liver...) -Often polarized (one side faces lumen (hollow inside of organ/tube) while other side interacts with underlying blood vessels/cells) -Simple epithelia have one layer of cells, stratified have multiple layers, and pseudostratified look like they have many layers but only have one -Cubodial cells are cube shaped, columnar cells are long and thin, and squamous cells are flat and scalelike *Squamous cells compose the external surface of the body and the lining of many internal organs. They are epithelial cells. Connective Tissue:Supports body and provides framework for epithelial cells to carry out their functions -Bone, cartilage, tendons, ligaments, adipose tissue, blood -Most cells in connective tissues produce and secrete materials like colleen and elastin to form the extracellular matrix -main contributors to stroma (support structure)

Blood Pressure

force per unit area exerted on wall of blood vessel by blood Measured by sphygmomanometer (Measure gauge pressure in systemic circulation = pressure above/beyond atmospheric pressure/760 mmHg at sea level) Expressed as ratio of systolic (ventricular contraction) to diastolic (ventricular relaxation) pressures -Pressure gradually drops form arterial to venous circulation, with largest drop across arterioles (capillaries are thin-walled and unable to withstand pressure of arterial side) *The greatest amount of resistance is provided by the arterioles, which also results in the greatest drop in blood pressure. Arterioles are highly muscular and have the ability to contract and dilate in order to affect blood pressure. -Normal BP: between 90/60 and 120/80 -the pressure in the aorta is usually about 120 or 80 mmHg, depending on whether the heart is in systole (ventricular contraction) or diastole, whereas the pressure in the superior vena cava is near zero. -Ohm's Law (V=IR) can be translated to Delta P = CO X TPR for pressure differential across circulation = Cardiac output X total peripheral (vascular) resistance Resistance is based on three factors: Resisitivity, length, and cross sectional area -Longer blood vessel/smaller cross-sectional area = more pressure -With exception to 3 portal systems, all systemic capillary beds are in parallel with each other, so opening bed will decrease vascular resistance and increase cardiac output BP is regulated using baroreceptors (specialized neurons that detect changes in the mechanical focus on the walls of the blood vessels) in walls of vasculature -BP too low = stimulate Sympathetic NS = vasoconstriction -In addition, chemoreceptors can sense osmolarity of blood too high (dehydration), promoting release of antidiuretic hormone (ADH/vasopressin = peptide hormone made in hypothalamus and stored in posterior pituitary) = increase reabsorption of water = increase in blood volume and pressure (also dilutes blood) -ADH to raise BP, ANP to lower it

Brush-Border Enzymes

group of enzymes present on the luminal surface of cells lining the duodenum that break down larger biomolecules into monomers that are able to be absorbed

Anaphase 1

homologous chromosomes separate and are pulled to opposite ends of the cell Disjunction (Mendel's first law of segregation)

Metaphase 1

homologous pairs (tetrads) line up in center, Spindle fibers attach to the chromosmes. *In mitosis, each chromosome is lined up on metaphase plate by 2 spindle fibers (one from each pole), in meiosis homologous chromosomes are lined up across from each other at the plate and held by ONE spindle fiber

Sexual development

hypothalamus restricts the making of (gonadotropin-releasing hormone) GnRH up until puberty - and lifted at puberty by hypothalamus at puberty GnRH is made by hypothalamus --> triggers anterior pituitary to synthesize/release LH (luteinizing hormone) and FSH (follicle stimulating hormone) --> trigger production of sex hormones Male: Fetal period (9 weeks after fertilization until birth), presence of Y chromosome leads to androgen production -FSH stimulate Sertoli cells to trigger sperm maturation, LH causes interstitial cells to produce testosterone -Testosterone = negative feedback on hypothalamus Female: Estrogen secreted in response to FSH: Develops/maintains reproductive system -Estrogen also thickens lining of uterus (endometrium) each month in prepartion of zygote -Progesterone: secreted by corpus lute (Remians of ovarian follicles) in response to LH -Estrogen establishes endometrium (involved in initial thickening), progesterone (development and maintenance) protects endometrium By the end of first trimester of pregnancy, progesterone is supplied by placenta, corpus lute atrophies and stops

Flagella

long, whiplike structures that can be used for propulasion Bacteria can have one, two, or many Ability of cell to detect chemical stimuli and move toward/away from them = chemotaxis Flagella composed of filament (hollow, helical structure composed of flagellin), basal body (anchors flagellum to cytoplasmic membrane/serves as motor (Rotates up to 300 hz), and hook (connects filament and basal body)

Agranulocytes (leukocytes)

monocytes, lymphocytes (B and T cells: Responsible for antibody production, immune system modulation, and targeted killing of infected cells) -No granules -Specific immune response/target specific pathogens -Primary response or long term memory (Vaccine: exposure to weakened pathogen/ antigenic protein (protein that can be recognized by immune system) -Lymphocyte maturation takes place in bone marrow (B cells: Antibody generation) or thymus (T cells: kill virally infected cells/activate immune cells) -Monocytes: Phagocytize foreign matter in bloodstream (become macrophages when they enter organ/tissue)

A patient presents in the ER with a blood pH of 7.25. A possible reason for this is: a panic attack causing the patient to hyperventilate myasthenia gravis causing diaphragm weakness. chloride deficiency causing excessive bicarbonate anion reabsorption in the proximal convoluted tubule severe vomiting and the loss of stomach fluids.

myasthenia gravis causing diaphragm weakness. This could increase CO2 concentrations by decreasing breathing rate and cause a decrease in blood pH. The respiratory system plays a role in pH balance through the bicarbonate buffer system in the blood: CO2(g) + H2O(l) ⇋ H2CO3(aq) ⇋ H+(aq) + HCO⁻3(aq) The body attempts to maintain a pH between 7.35 and 7.45. When the pH is lower, and hydrogen ion concentration is higher (acidemia), acid-sensing chemoreceptors just outside the blood-brain barrier send signals to the brain to increase the respiratory rate. Further, an increasing hydrogen ion concentration will cause a shift in the bicarbonate buffer system, generating additional carbon dioxide. To answer this question, we must evaluate how each of the situations affects the buffer system. Muscle weakness caused by myasthenia gravis would decrease the breathing rate causing a retention of CO2 and decrease in blood pH, making choice (B) the correct answer. Hyperventilation will cause a decrease in CO2 which will increase blood pH (A). Shifting the buffer system to the right by increasing bicarbonate concentrations will also increase blood pH (C). Losing stomach acid through vomiting would cause the blood pH to increase as well (D). Again, (B) is correct.

Albumin

protein synthesized in the liver that maintains the oncotic pressure of the blood and serves as a carrier for many drugs and hormones

Modes of Natural Selection (3 patterns of selection)

stabilizing selection: Loss of extremes Keeps phenotypes within specific range by selecting against extremes (human birth weight: maintained within narrow band) directional selection: Movement towards one extreme or the other -Adaptive pressure can lead to emergence and dominance of an initially extreme phenotype (Example: plate with only a few bacteria that are resistant to bacteria, then antibiotic introduced, so only resistant survive) -Natural selection is the history of differential survivorship over time -Emergence of mosquitoes resistant to DDT disruptive selection: movement towards both extremes with loss of norm (speciation may occur) -Two extreme phenotypes are selected over the norm -Darwin and the finches: Saw either all large or all small beaks with no intermediate size, he hypothesized original ancestor had medium sized beak -Facilitated by existence of polymorphisms (naturally occurring differences in form between members of same population, like light and dark color in same species) -Adaptive radiation: The evolutionary process by which one species gives rise to several species, each specialized for different niches describes rapid rise of a number of different species from a common ancestor -> allows for various species to occupy different niches (specific environment, including habitat, available resources, and predators) *Adaptive radiation is favored by environmental changes or isolation of small groups of ancestral species

Golgi Apparatus

stack of membrane-bound sacs in the cell that modifies, sorts, and packages proteins from the endoplasmic reticulum Contents secreted via exocytosis

Second Messenger

substance that is mobilized within a cell after the binding of a hormone to its receptor

Vital Capacity

the maximum amount of air that can be moved in a single respiratory cycle =Tidal volume + inspiratory and expiratory reserves volumes Difference between max and min amounts of air that lungs can hold Not the same as TLC The total amount of air the lungs can hold is called the total lung capacity (TLC)

Prophase 2

the nuclear envelope breaks down, nucleoli disappear, centrioles migrate to opposite poles, and the spindle apparatus begins to forms


Conjuntos de estudio relacionados

Fluid & Electrolytes Prep U Questions

View Set

Medical Law and Ethics Quiz 5-8 *

View Set

The Computer and Technology Revolutions.

View Set

Computer Applications Cumulative Exam (90%)

View Set

Investment Companies - Practice Questions

View Set

PSYC-305 Final Exam Review Part 3

View Set

Chapter 2.1: Drawing InQuisitive

View Set

leadership test 3 practice questions

View Set